Chapter 19: Nursing Management of Pregnancy at Risk

Réussis tes devoirs et examens dès maintenant avec Quizwiz!

The nurse is preparing discharge instructions for a pregnant patient experiencing preterm rupture of membranes. What should the nurse include in this teaching? Select all that apply.

1. Avoid douching. 2. Measure oral temperature twice a day.

A primigravida 28-year-old client is noted to have Rh negative blood and her husband is noted to be Rh positive. The nurse should prepare to administer RhoGAM after which diagnostic procedure?

Amniocentesis

A nurse in the maternity triage unit is caring for a client with a suspected ectopic pregnancy. Which nursing intervention should the nurse perform first?

Assess the client's vital signs.

A client has been admitted to the hospital with a diagnosis of severe preeclampsia. Which nursing intervention is the priority?

Confine the client to bed rest in a darkened room.

This syndrome is a variant of preeclampsia/eclampsia syndrome that occurs in 10% to 20% of clients whose diseases are labeled as severe.

HELLP syndrome

elvated liver may indicate

HELLP syndrome

Placenta Previa

Painless bleeding condition that occurs in the last two trimesters of pregnancy

Clonus

The presence of rhythmic involuntary contractions, most often at the foot or ankle

A nurse is monitoring a client with PROM who is in labor and observes meconium in the amniotic fluid. What does the observation of meconium indicate?

fetal distress related to hypoxia

The nurse is assessing a client at 12 weeks' gestation at a routine prenatal visit who reports something doesn't feel right. Which assessment findings should the nurse prioritize?

gestational hypertension, hyperemesis gravidarum, absence of FHR

A nurse is providing care to a multiparous client. The client has a history of cesarean births. The nurse anticipates the need to closely monitor the client for which condition?

placenta accreta

A nursing instructor identifies which factor as increasing the chances of infection when coupled with prolonged labor? multiple births number of previous pregnancies maternal age premature rupture of membranes

premature rupture of membranes

A pregnant client is brought to the health care facility with signs of premature rupture of the membranes (PROM). Which conditions and complications are associated with PROM? Select all that apply. preterm labor placental abruption (abruptio placentae) placenta previa prolapsed cord spontaneous abortion (miscarriage)

prolapsed cord placental abruption (abruptio placentae) preterm labor

A woman has presented to the emergency department with symptoms that suggest an ectopic pregnancy. Which finding would lead the nurse to suspect that the fallopian tube has ruptured?

referred pain

A pregnant woman has arrived to the office reporting vaginal bleeding. Which finding during the assessment would lead the nurse to suspect an inevitable abortion?

strong abdominal cramping

A nurse is assessing a client diagnosed with mild preeclampsia. The nurse suspects that the client has developed severe preeclampsia based on which finding?

urine output of less than 400 mL/24 hours

what is Placenta abruption or abruptio placentae

when the placenta becomes detached prematurely.

A pregnant patient with a history of premature cervical dilatation undergoes cervical cerclage. Which outcome indicates that this procedure has been successful?

The client delivers a full-term fetus at 39 weeks' gestation.

Methotrexate

a folic acid antagonist that inhibits cell division in the developing embryo, is most commonly used to treat ectopic pregnancy.

Rh incompatibility can be prevented with the

use of RHogam

Which medication would the nurse prepare to administer if prescribed as treatment for an unruptured ectopic pregnancy? methotrexate promethazine ondansetron oxytocin

methotrexate

Which medication would the nurse prepare to administer if prescribed as treatment for an unruptured ectopic pregnancy?

methotrexate

Which assessment findings, experienced by the client at 36 weeks' gestation, would the nurse document as diagnostic signs of severe preeclampsia? Select all that apply. +1 proteinuria blood pressure of 164/110 mm Hg Elevated serum creatinine elevated liver enzymes edema

+1 proteinuria blood pressure of 164/110 mm Hg Elevated serum creatinine elevated liver enzymes

Approximately _____ of all women with diabetes will develop hydramnios during their pregnancy

18%

Hydramnios occurs in approximately_____ of all pregnancies and is associated with fetal anomalies of development.

3%

Hyperrelexia (brisk reflexes)

A common presenting symptom of preeclampsia and is the result of an irritable cortex

The nurse is comforting and listening to a young couple who just suffered a miscarriage. When asked why this happened, which reason should the nurse share as a common cause?

Chromosomal abnormality

The nurse is comforting and listening to a young couple who just suffered a miscarriage. When asked why this happened, which reason should the nurse share as a common cause? Maternal smoking Lack of prenatal care Chromosomal abnormality The age of the mother

Chromosomal abnormality

A woman at 34 weeks' gestation presents to labor and delivery with vaginal bleeding. Which finding from the obstetric examination would lead to a diagnosis of placental abruption (abruptio placentae)? -Onset of vaginal bleeding was sudden and painful -Fetus is in a breech position -Sonogram shows the placenta covering the cervical os -Uterus is soft between contractions

Onset of vaginal bleeding was sudden and painful Sudden onset of abdominal pain and vaginal bleeding with a rigid uterus that does not relax are signs of a placental abruption (abruptio placentae). The other findings are consistent with a diagnosis of placenta previa.

What makes the diagnosis of gestational hypertension different from the diagnosis of preeclampsia? a) Severity of hypertension b) Proteinuria c) Ketonuria d) The hypertension of gestation disappears after delivery. The hypertension of preeclampsia does not.

Proteinuria Explanation: Gestational hypertension is the current term used to describe elevated blood pressure (greater than or equal to 140/90 mm/Hg) that develops for the first time during pregnancy without the presence of protein in the urine.

n. With an inevitable abortion,

passage of the products of conception may occur.

A woman pregnant with twins comes to the clinic for an evaluation. While assessing the client, the nurse would be especially alert for signs and symptoms for which potential problem? chorioamnionitis post-term labor preeclampsia oligohydramnios

preeclampsia

Preeclampsia without Severe Features Blood Pressure? Seizures/coma? Hyperreflexia? Other signs and symptoms?

Blood pressure: >140/90 mm Hg after 20 weeks' gestation Seizures/coma? NO Hyperreflexia? NO Other signs and symptoms? N/A

Vaginal spotting, nausea, and breast tenderness are typical findings of

early pregnancy

A woman who is Rh negative asks the nurse how many children she will be able to have before Rh incompatibility causes them to die in utero. The nurse's best response would be that:

as long as she receives RhoGAM, there is no limit.

A high-risk pregnant client is determined to have gestational hypertension. The nurse suspects that the client has developed severe preeclampsia based on which finding?

blurred vision

A woman with severe preeclampsia is receiving magnesium sulfate. The woman serum magnesium level is 9.0mEq/L. Which finding would the nurse most likely note?

diminshed reflexes

After an examination, a client has been determined to have an unruptured ectopic pregnancy. Which medication would the nurse anticipate being prescribed?

methotrexate

The onset of seizure activity indicates

eclampsia

A client who has been hospitalized with type 2 diabetes is to be discharged. Which tips for eating out should the nurse share?

"Select fresh fruit for dessert" Tips for eating out include the following: Choose unsweetened juice, use oil and vinegar or fresh lemon juice instead of regular salad dressings, order plain roasted meat without gravy or sauce, and select fresh fruit for dessert.

Rh Imcompatibility

A condition that develops when a women with Rh-negative blood type is exposed to Rh-positive blood cells and subsequently develops circulation titers of Rh antibodies

A nurse suspects that a pregnant client may be experiencing a placental abruption based on assessment of which finding? Select all that apply. dark red vaginal bleeding insidious onset rigid uterus absence of pain absent fetal heart tones

Absent fetal heart tones Dark red vaginal bleeding Rigid uterus

Preterm premature rupture of membranes (PPROM

defined as the rupture of the membranes prior to the onset of labor in a woman who is less than 37 weeks' gestation.

A nurse is conducting a refresher program for a group of perinatal nurses. Part of the program involves a discussion of HELLP. The nurse determines that the group needs additional teaching when they identify which aspect as a part of HELLP?

elevated lipoproteins

A pregnant client has been admitted with reports of brownish vaginal bleeding. On examination there is an elevated hCG level, absent fetal heart sounds, and a discrepancy between the uterine size and the gestational age. The nurse interprets these findings to suggest which condition?

gestational trophoblastic disease

The nurse is preparing the plan of care for a woman hospitalized for hyperemesis gravidarum. Which interventions would the nurse most likely include? Select all that apply.

maintaining NPO status for the first day or two administering antiemetic agents obtaining baseline blood electrolyte levels monitoring intake and output

The nurse is caring for a pregnant client with fallopian tube rupture. Which intervention is the priority for this client? Monitor the client's beta-hCG level. Monitor the client's vital signs and bleeding. Monitor the fetal heart rate (FHR). Monitor the mass with transvaginal ultrasound.

monitor the client's vital signs and bleeding

A nurse is caring for a client who wants to consume monounsaturated fats in his diet. What should the nurse recommend the client include in the diet?

olive oil low-fat margarine, soybean oil, and corn oil are all examples of polyunsaturated fats.

PROM (premature rupture of membranes)

refers to a woman who is beyond 37 weeks' gestation, has presented with spontaneous rupture of the membranes, and is not in labor

A woman with a recent incomplete abortion is to receive therapeutic misoprostol. The nurse understands that the rationale for administering this drug to?

ensure passage of all the products of conception

Amniocentesis would be appropriate for treatment of ?

polyhydramnios

When assessing a pregnant woman with vaginal bleeding, the nurse would suspect a threatened abortion based on which finding? slight vaginal bleeding cervical dilation strong abdominal cramping passage of fetal tissue

slight vaginal bleeding

Misoprostol

used to stimulate uterine contractions and evacuate the uterus after an incomplete abortion to ensure passage of all the products of conception.

The nurse is teaching a client who is diagnosed with preeclampsia how to monitor her condition. The nurse determines the client needs more instruction after making which statement?

"If I have changes in my vision, I will lie down and rest."

A nurse is caring for a client who had hyperlipidemia as a result of low levels of high-density lipoproteins. What suggestion should the nurse offer to increase HDL level?

"have a regular exercise routine" Moderate alcohol consumption, regular exercise, and a low-trans-fat and low-carbohydrate diet can help to raise HDL levels.

Oligohydramnios

A decreased amount of amniotic fluid (<500 mL) between 32 and 36 weeks' gestation

oligohydramnios

A decreased amount of amniotic fluid (<500mL) between 32 and 36 weeks' gestation.

A nurse is caring for a pregnant adolescent client, who is in her first trimester, during a visit to the maternal child clinic. Which important area should the nurse address during assessment of the client? A. Sexual development of the client B. Whether sex was consensual C. Options for birth control in the future D. Knowledge of child development

D. Knowledge of child development

A 25-year-old client at 22 weeks' gestation is noted to have proteinuria and dependent edema on her routine prenatal visit. Which additional assessment should the nurse prioritize and alert the RN or health care provider?

Initial BP 100/70 mm Hg; current BP 140/90 mm Hg

mild preeclampsia

Mild facial edema or hand edema occurs

A woman at 34 weeks' gestation presents to labor and delivery with vaginal bleeding. Which finding from the obstetric examination would lead to a diagnosis of placental abruption (abruptio placentae)? Fetus is in a breech position Sonogram shows the placenta covering the cervical os Uterus is soft between contractions Onset of vaginal bleeding was sudden and painful

Onset of vaginal bleeding was sudden and painful

A woman in labor suddenly reports sharp fundal pain accompanied by slight dark red vaginal bleeding. The nurse should prepare to assist with which situation? Premature separation of the placenta Preterm labor that was undiagnosed Placenta previa obstructing the cervix Possible fetal death or injury

Premature separation of the placenta

A client at 27 weeks' gestation is admitted to the OB unit afer reporting headaches and edema of her hands. Review of the prenatal notes reveals BP consistently above 136/90 mm Hg. The nurse anticipates the health care provider will order magneisum sulfate to accomplish which primary goal?

Prevent maternal seizures

A 24-year-old client is brought to the emergency department complaining of severe abodminal pain, vaginal bleeding, and fatigue. The nurse notes on assessment cool, clammy skin, confusion, and vital signs: HR 130, RR 28, and BP 98/60 mm Hg. Which action should the nurse prioritize?

Rule out shock.

A pregnant woman with preeclampsia is to receive magnesium sulfate IV. Which of the following assessments would be most important prior to administering a new dose? a) Anxiety level b) Patellar reflex c) Blood pressure d) Pulse rate

b) Patellar reflex Rationale: A symptom of magnesium sulfate toxicity is loss of deep tendon reflexes. Assessing for one of these before administration is assurance the drug administration will be safe.

just know this part 2

electrolyte replacement therapy as indicated. Once the nausea and vomiting subside, oral food and fluids are gradually reintroduced. Total parenteral nutrition or a feeding tube is used to prevent malnutrition only if the client does not improve with these interventions.

A pregnant woman with preeclampsia is to receive magnesium sulfate IV. Which assessment would be most important prior to administering a new dose?

patellar reflex A symptom of magnesium sulfate toxicity is loss of deep tendon reflexes. Assessing for one of these before administration is assurance the drug administration will be safe.

A nurse is teaching a group of pregnant woman about bleeding that can occur early in pregnancy. The nurse determines that additional teaching is needed when the group identifies which condition as a common cause?

placenta previa

Slight vaginal bleeding early in pregnancy and a closed cervical os are associated with a

threatened abortion.

A client at 11 weeks' gestation experiences pregnancy loss. The client asks the nurse if the bleeding and cramping that occurred during the miscarriage were caused by working long hours in a stressful environment. What is the most appropriate response from the nurse?

"I can understand your need to find an answer to what caused this. Let's talk about this further." Explanation: Talking with the client may assist her to explore her feelings. She and her family may search for a cause for a spontaneous early bleeding so they can plan for future pregnancies. Even with modern technology and medical advances, however, a direct cause cannot usually be determined.

the diagnosis of HELLP is

: low hematocrit, elevated LDH, elevated AST, elevated ALT, elevated BUN, elevated bilirubin level, elevated uric acid and creatinine levels, and low platelet coun

grade 3 (severe)

? absent to moderate bleeding (more than 1,500 mL), more than 50% separation, profound shock, dark vaginal bleeding, agonizing abdominal pain, decreased blood pressure, significant tachycardia, and development of disseminated intravascular coagulopathy.

c. gestational hypertension

A novice nurse asks to be assigned to the least complex antepartum client. Which condition would necessitate the least complex care requirements? a. abruptio placenta b. preecalmpsia c. gestational hypertension d. placenta previa

A pregnant patient is being admitted for severe preeclampsia. In which room location should the nurse place this patient?

In the back private room With severe preeclampsia, hospitalization is required so that bed rest can be enforced and the patient can be observed more closely. A patient with severe preeclampsia is admitted to a private room so that rest is undisturbed. Noises such as a baby crying, elevator doors opening and closing, and conversation from the nurse's station is sufficient to trigger a seizure. A private room will help reduce the likelihood of seizure development.

Medications Related to Abortions Mifepristone (RU-486)

Indication: Acts as progesterone antagonist, allowing prostaglandins to stimulate uterine contractions; causes the endometrium to slough; may be followed by administration of misoprostol within 48 hours Nursing Actions: -Monitor for headache, vomiting, diarrhea, and heavy bleeding. -Anticipate administration of antiemetic prior to use to reduce nausea and vomiting. -Encourage client to use acetaminophen to reduce discomfort from cramping.

A woman at 34 weeks' gestation presents to labor and delivery with vaginal bleeding. Which finding from the obstetric examination would lead to a diagnosis of placental abruption (abruptio placentae)?

Onset of vaginal bleeding was sudden and painful Explanation: Sudden onset of abdominal pain and vaginal bleeding with a rigid uterus that does not relax are signs of a placental abruption (abruptio placentae). The other findings are consistent with a diagnosis of placenta previa

A woman at 28 weeks' gestation has been hospitalized with moderate bleeding that is now stabilizing. The nurse performs a routine assessment and notes the client sleeping, lying on the back, and electronic fetal heart rate (FHR) monitor showing gradually increasing baseline with late decelerations. Which action will the nurse perform first?

Reposition the client to left side The fetus is showing signs of fetal distress. The immediate treatment is putting the client in a side-lying position to ensure adequate perfusion to the fetus. After placing the client on the side, the nurse should re-assess the FHR and determine if oxygen, IV fluids, and calling the health care provider are needed.

The nurse is caring for a client who has a multifetal pregnancy. What topic should the nurse prioritize during health education?

Signs of preterm labor The client with a multifetal pregnancy must be made aware of the risks posed by preterm labor. There is no corresponding increase in the risk for hypertension or blood incompatabilities. Parenting skills are secondary to physiologic needs at this point.

A pregnant patient with a history of premature cervical dilatation undergoes cervical cerclage. Which outcome indicates that this procedure has been successful?

The client delivers a full-term fetus at 39 weeks' gestation. Premature cervical dilatation is when the cervix dilates prematurely and cannot retain a fetus until term. After the loss of one child because of premature cervical dilatation, a surgical operation termed cervical cerclage can be performed to prevent this from happening in a second pregnancy. This procedure is the use of purse-string sutures placed in the cervix to strengthen the cervix and prevent it from dilating until the end of pregnancy. Evidence that this procedure is effective would be the client delivering a full-term fetus at 39 weeks' gestation. Spontaneous rupture of the membranes could indicate that the procedure was not successful. Vaginal bleeding could indicate another health problem or that the procedure was not successful. This procedure does not impact the patient's respirations or amount of abdominal pain while pregnant. These manifestations could indicate another health problem with the pregnancy.

a. Assess the client's temperature

When caring for a client with premature rupture of membranes (PROM), the nurse observes an increase in the client's pulse. What should the nurse do next? a. Assess the client's temperature b. Monitor the client for preterm labor c. Assess for cord compression d. Monitor the fetus for respiratory distress

The nurse is required to assess a pregnant client who is reporting vaginal bleeding. Which nursing action is the priority?

assessing the amount & color of the bleeding When the woman arrives and is admitted, assessing her vital signs, the amount and color of the bleeding, and current pain rating on a scale of 1 to 10 are the priorities. Assessing the signs of shock, monitoring uterine contractility, and determining the amount of funneling are not priority assessments when a pregnant woman complaining of vaginal bleeding is admitted to the hospital.

When administering magnesium sulfate to a woman with severe preeclampsia, which finding would alert the nurse to the development of magnesium toxicity? a) Seizures b) Elevated liver enzymes c) Diminished reflexes d) Serum magnesium level of 6.5 mEq/L

c) Diminished reflexes Rationale: Diminished or absent reflexes occur when a client develops magnesium toxicity. Elevated liver enzymes are unrelated to magnesium toxicity and may indicate the development of HELLP syndrome. The onset of seizure activity indicates eclampsia. A serum magnesium level of 6.5 mEq/L would fall within the therapeutic range of 4 to 7 mEq/L.

You Selected:

frequent urination Polyphagia, polyuria, polydipsia, and weight loss are cardinal signs of type 1 diabetes mellitus. Other signs include shortened attention span, lowered frustration tolerance, fatigue, dry skin, blurred vision, sores that are slow to heal, and flushed skin.

A novice nurse asks to be assigned to the least complex antepartum client. Which condition would necessitate the least complex care requirements?

gestational hypertension Explanation: Hypertensive disorders represent the most common complication of pregnancy. Gestational hypertension is elevated blood pressure without proteinuria, other signs of preeclampsia, or preexisting hypertension. Placental abruption (abruptio placentae), a separation of the placenta from the uterine wall; placenta previa (placenta covering the cervical os); and preeclampsia are high-risk, potentially life-threatening conditions for the fetus and mother during labor and birth.

A nurse is providing care to a client who has been diagnosed with a common benign form of gestational trophoblastic disease. The nurse identifies this as: hydatidiform mole. ectopic pregnancy. hydramnios. placenta accrete.

hydatidiform mole

Which measure would the nurse include in the plan of care for a woman with prelabor rupture of membranes if her fetus's lungs are mature?

labor induction With prelabor rupture of membranes (PROM) in a woman whose fetus has mature lungs, induction of labor is initiated. Reducing physical activity, observing for signs of infection, and giving corticosteroids may be used for the woman with PROM when the fetal lungs are immature.

Which medication would the nurse prepare to administer if prescribed as treatment for an unruptured ectopic pregnancy? oxytocin promethazine ondansetron methotrexate

methotrexate

During a routine prenatal visit, a client is found to have proteinuria and a blood pressure rise to 140/90 mm Hg. The nurse recognizes that the client has which condition? mild preeclampsia gestational hypertension severe preeclampsia eclampsia

mild preeclampsia

While assessing a pregnant woman, the nurse suspects that the client may be at risk for hydramnios. Which information would the nurse use to support this suspicion? Select all that apply. reports of shortness of breath difficulty obtaining fetal heart rate history of diabetes fundal height below that for expected gestational age identifiable fetal parts on abdominal palpation

reports of shortness of breath difficulty obtaining fetal heart rate history of diabetes

A woman's baby is HIV positive at birth. She asks the nurse if this means the baby will develop AIDS. Which statement would be the nurse's best answer? "The antibodies may be those transferred across the placenta; the baby may not develop AIDS." "She already has AIDS. That's what being HIV positive means." "HIV antibodies do not cross the placenta; this means the baby will develop AIDS." "HIV is transmitted at birth; having a cesarean birth prevented transmission."

"The antibodies may be those transferred across the placenta; the baby may not develop AIDS."

spontaneous

A ___________________ abortion refers to the loss of a fetus resulting from natural causes - that is, not elective or therapeutically induced by a procedure.

A pregnant woman has arrived to the office reporting vaginal bleeding. Which finding during the assessment would lead the nurse to suspect an inevitable abortion?

Strong abdominal cramping

The nurse is caring for a woman at 32 weeks gestation with severe preeclampsia. Which assessment finding should the nurse prioritize after the administration of hydralazine to this client?

Tachycardia

The nurse is required to assess a pregnant client who is reporting vaginal bleeding. Which nursing action is the priority? monitoring uterine contractility assessing signs of shock determining the amount of funneling assessing the amount and color of the bleeding

assessing the amount and color of the bleeding

A nurse is providing care to a multiparous client. The client has a history of cesarean births. The nurse anticipates the need to closely monitor the client for which condition? placenta accreta placenta abruption preeclampsia oligohydramnios

placenta accreta

The nurse is teaching a client who is diagnosed with preeclampsia how to monitor her condition. The nurse determines the client needs more instruction after making which statement? "If I have changes in my vision, I will lie down and rest." "I will weigh myself every morning after voiding before breakfast." "I will count my baby's movements after each meal." "If I have a severe headache, I'll call the clinic."

"If I have changes in my vision, I will lie down and rest."

A pregnant women calls the clinic to report a small amount of painless vaginal bleeding. What response by the nurse is best?

"Please come in how for evaluation from your healthcare provider" Bleeding during pregnancy is always a deviation from normal and should be evaluated carefully. It may be life-threatening or it may be something that is not a threat to the mother and/or fetus. Regardless, it needs to be evaluated quickly and carefully. Telling the client it may be harmless is a reassuring statement, but does not suggest the need for urgent evaluation. Having the mother lay on her left side and drink water is indicated for cramping.

A woman you care for has an Rh-negative blood type. Following the birth of her infant, you administer her Rho(D) (D immune globulin). The purpose of this is to a) Prevent fetal RH blood formation. b) Stimulate maternal D immune antigens. c) Promote maternal D antibody formation. d) Prevent maternal D antibody formation.

...

A client in her 20th week of gestation develops HELLP syndrome. What are features of HELLP syndrome? Select all that apply.

1. hemolysis 2. elevated liver enzymes 3. low platelet count

A client reporting she recently had a positive pregnancy test has reported to the emergency department stating one-sided lower abdominal pain. The health care provider has prescribed a series of tests. Which test will provide the most definitive confirmation of an ectopic pregnancy?

Abdominal ultrasound Explanation: An ectopic pregnancy refers to the implantation of the fertilized egg in a location other than the uterus. Potential sites include the cervix, uterus, abdomen, and fallopian tubes. The confirmation of the ectopic pregnancy can be made by an ultrasound, which would confirm that there was no uterine pregnancy. A quantitative hCG level may be completed in the diagnostic plan. hCG levels in an ectopic pregnancy are traditionally reduced. While this would be an indication, it would not provide a positive confirmation. The qualitative hCG test would provide evidence of a pregnancy, but not the location of the pregnancy. A pelvic exam would be included in the diagnostic plan of care. It would likely show an enlarged uterus and cause potential discomfort to the client but would not be a definitive finding.

A nurse is caring for a young woman who is in her 10th week of gestation. She comes into the clinic reporting vaginal bleeding. Which assessment finding best correlates with a diagnosis of hydatidiform mole?

Dark red, "clumpy" vaginal discharge Women with hydatidiform mole ("molar pregnancy") often pass blood clots or watery brown/dark red discharge from the vagina in the first trimester. If a complete molar pregnancy continues into the second trimester undetected, other signs and symptoms appear. The woman often presents with complaints of dark to bright red vaginal bleeding and pelvic pain. Infrequently, she will report passage of grapelike vesicles.

The nurse is caring for an Rh-negative nonimmunized client at 14 weeks' gestation. What information would the nurse provide to the client?

Obtain Rho(D) immune globulin at 28 weeks' gestation. The current recommendation is that every Rh-negative nonimmunized woman receives Rho(D) immune globulin at 28 weeks' gestation and again within 72 hours after giving birth. Consuming a well-balanced nutritional diet and avoiding sexual activity until after 28 weeks will not help to prevent complications of blood incompatibility. Transvaginal ultrasound helps to validate the position of the placenta and will not help to prevent complications of blood incompatibility.

A woman at 34 weeks' gestation presents to labor and delivery with vaginal bleeding. Which finding from the obstetric examination would lead to a diagnosis of placental abruption (abruptio placentae)? Fetus is in a breech position Onset of vaginal bleeding was sudden and painful Sonogram shows the placenta covering the cervical os Uterus is soft between contractions

Onset of vaginal bleeding was sudden and painful

A client at 27 weeks' gestation is admitted to the OB unit afer reporting headaches and edema of her hands. Review of the prenatal notes reveals BP consistently above 136/90 mm Hg. The nurse anticipates the health care provider will order magneisum sulfate to accomplish which primary goal? Decrease blood pressure Decrease protein in urine Prevent maternal seizures Reverse edema

Prevent maternal seizures

A client at 27 weeks' gestation is admitted to the obstetric unit after reporting headaches and edema of her hands. Review of the prenatal notes reveals blood pressure consistently above 136/90 mm Hg. The nurse anticipates the health care provider will prescribe magnesium sulfate to accomplish which primary goal?

Prevent maternal seizures Explanation: The primary therapy goal for any client with preeclampsia is to prevent maternal seizures. Use of magnesium sulfate is the drug therapy of choice for severe preeclampsia and is only used to manage and attempt to prevent progression to eclampsia. Magnesium sulfate therapy does not have as a primary goal of decreasing blood pressure, decreasing protein in the urine, or reversing edema.

A patient is admitted to labor and delivery for management of severe preeclampsia. An IV infusion of magnesium sulfate is started. What is the primary goal for magnesium sulfate therapy? a) Reverse edema b) Decrease protein in urine c) Decrease blood pressure d) Prevent maternal seizures

Prevent maternal seizures Explanation: The primary therapy goal for any preeclamptic patient is to prevent maternal seizures. Use of magnesium sulfate is the drug therapy of choice for severe preeclampsia and is only used to manage and attempt to prevent progression to eclampsia. Magnesium sulfate therapy does not have as its primary goal a decrease in blood pressure, a decrease in protein in the urine, nor the reversal of edema.

A 44-year-old client has lost several pregnancies over the last 10 years. For the past 3 months, she has had fatigue, nausea, and vomiting. She visits the clinic and takes a pregnancy test; the results are positive. Physical examination confirms a uterus enlarged to 13 weeks' gestation; fetal heart tones are heard. Ultrasound reveals that the client is experiencing some bleeding. Considering the client's prenatal history and age, what does the nurse recognize as the greatest risk for the client at this time? a) Pregnancy loss. b) Preterm labor. c) Premature birth. d) Hypertension.

a) Pregnancy loss. Rationale: The client's advanced maternal age (pregnancy in a woman 35 years or older) increases her risk for pregnancy loss. Hypertension, preterm labor, and prematurity are risks as this pregnancy continues. Her greatest risk at 13 weeks' gestation is losing this pregnancy.

A nurse is caring for a client who just experienced a miscarriage in her first trimester. When asked by the client why this happened, which is the best response from the nurse?

abnormal fetal development The most frequent cause of miscarriage in the first trimester of pregnancy is abnormal fetal development, due either to a teratogenic factor or to a chromosomal aberration. In other miscarriages, immunologic factors may be present or rejection of the embryo through an immune response may occur. Another common cause of early miscarriage involves implantation abnormalities. Miscarriage may also occur if the corpus luteum on the ovary fails to produce enough progesterone to maintain the decidua basalis.

A woman who is Rh negative asks the nurse how many children she will be able to have before Rh incompatibility causes them to die in utero. The nurse's best response would be that: no more than three children is recommended. she will have to ask her primary care provider. as long as she receives Rho(D) immune globulin, there is no limit. only her next child will be affected.

as long as she receives Rho(D) immune globulin, there is no limit.

A client has been admitted to the hospital with a diagnosis of severe pre-eclampsia. Which of the following is the priority nursing? a) Check for vaginal bleeding every 15 minutes b) Keep the client on her side so that secretions can drain from her mouth c) Confine the client to bed rest in a darkened room d) Administer oxygen by face mask

c) Confine the client to bed rest in a darkened room Rationale: With severe pre-eclampsia, most women are hospitalized so that bed rest can be enforced and a woman can be observed more closely than she can be on home care. Darken the room if possible because a bright light can also trigger seizures. The other interventions listed pertain to a client who has experienced a seizure and has thus progressed to eclampsia.

Which of the following would the nurse include in the plan of care for a woman with premature rupture of membranes if her fetus's lungs are mature? a) Administration of corticosteroids b) Observation for signs of infection c) Labor induction d) Reduction in physical activity level

c) Labor induction Rationale: With premature rupture of membranes in a woman whose fetus has mature lungs, induction of labor is initiated. Reducing physical activity, observing for signs of infection, and giving corticosteroids may be used for the woman with PROM when the fetal lungs are immature.

A woman in her 20s has experienced a spontaneous abortion (miscarriage) at 10 weeks' gestation and asks the nurse at the hospital what went wrong. She is concerned that she did something that caused her to lose her baby. The nurse can reassure the woman by explaining that the most common cause of miscarriage in the first trimester is related to which factor?

chromosomal defects in the uterus Fetal factors are the most common cause of early miscarriages, with chromosomal abnormalities in the fetus being the most common reason. This client fits the criteria for early spontaneous abortion since she was only 10 weeks' pregnant and early miscarriage occurs before 12 weeks.

While examining a newborn, a nurse observes salmon patches on the nape of the neck and on the eyelids. Which is the most likely cause of these skin abnormalities? -bruising from the birth process -an immature autoregulation of blood flow -an allergic reaction to the soap used for the first bath -concentration of immature blood vessels

concentration of immature blood vessels A concentration of immature blood vessels causes salmon patches. Bruising does not look like salmon patches but would be more bluish-purple in appearance. Harlequin sign is a result of immature autoregulation of blood flow and is commonly seen in low-birth-weight newborns. An allergic reaction would be more generalized and would not be salmon-colored.

A client is diagnosed with gestational hypertension and is receiving magnesium sulfate. The nurse determines that the medication is at a therapeutic level based on which finding? urinary output of 20 mL per hour deep tendons reflexes 2+ difficulty in arousing respiratory rate of 10 breaths/minute

deep tendons reflexes 2+

A nurse is assessing a pregnant client for the possibility of preexisting conditions that could lead to complications during pregnancy. The nurse suspects that the woman is at risk for hydramnios based on which preexisting condition?

diabetes Explanation: Approximately 18% of all women with diabetes will develop hydramnios during their pregnancy. Hydramnios occurs in approximately 2% of all pregnancies and is associated with fetal anomalies of development.

A nurse is caring for a client with poorly controlled type 1 diabetes. The nurse understands that the client may experience acute life-threatening complications caused by a high blood glucose concentration. Which of the following complications should the nurse expect?

diabetic ketoacidosis People with type 1 diabetes are susceptible to diabetic ketoacidosis. Hyperosmolar hyperglycemic nonketotic syndrome occurs most commonly in people with type 2 diabetes.

A woman is being closely monitored and treated for severe preeclampsia with magnesium sulfate. Which finding would alert the nurse to the development of magnesium toxicity in this client? serum magnesium level of 6.5 mEq/L diminished reflexes elevated liver enzymes seizures

diminished reflexes

A woman is being closely monitored and treated for severe preeclampsia with magnesium sulfate. Which finding would alert the nurse to the development of magnesium toxicity in this client? seizures serum magnesium level of 6.5 mEq/L diminished reflexes elevated liver enzymes

diminished reflexes

A nurse is conducting a refresher program for a group of perinatal nurses. Part of the program involves a discussion of HELLP. The nurse determines that the group needs additional teaching when they identify which aspect as a part of HELLP? hemolysis liver enzyme elevation low platelet count elevated lipoproteins

elevated lipoproteins

The nurse is required to assess a client for HELLP syndrome. Which are the signs and symptoms of this condition? Select all that apply.

epigastric pain upper right quadrant pain hyperbilirubinemia The signs and symptoms of HELLP syndrome are nausea, malaise, epigastric pain, upper right quadrant pain, demonstrable edema, and hyperbilirubinemia. Blood pressure higher than 160/110 mm Hg and oliguria are the symptoms of severe preeclampsia rather than HELLP syndrome.

A nurse is monitoring a client with PROM who is in labor and observes meconium in the amniotic fluid. What does the observation of meconium indicate?

fetal distress related to hypoxia When meconium is present in the amniotic fluid, it typically indicates fetal distress related to hypoxia. Meconium stains the fluid yellow to greenish brown, depending on the amount present. A decreased amount of amniotic fluid reduces the cushioning effect, thereby making cord compression a possibility. A foul odor of amniotic fluid indicates infection. Meconium in the amniotic fluid does not indicate CNS involvement.

A nurse is taking a history of a client at 5 weeks' gestation in the prenatal clinic; however, the client is reporting dark brown vaginal discharge, nausea, and vomiting. Which diagnosis should the nurse suspect? placenta previa hyperemesis gravidarum gestational trophoblastic disease pregnancy-induced depression

gestational trophoblastic disease

A pregnant client has been admitted with reports of brownish vaginal bleeding. On examination there is an elevated hCG level, absent fetal heart sounds, and a discrepancy between the uterine size and the gestational age. The nurse interprets these findings to suggest which condition? ectopic pregnancy placenta previa gestational trophoblastic disease abruption of placenta

gestational trophoblastic disease

A client in her 20th week of gestation develops HELLP syndrome. What are features of HELLP syndrome? Select all that apply. leukocytosis hemolysis elevated liver enzymes low platelet count hyperthermia

hemolysis elevated liver enzymes low platelet count

A nurse is providing care to a client who has been diagnosed with a common benign form of gestational trophoblastic disease. The nurse identifies this as:

hydatidiform mole.

A nurse is providing care to a client who has been diagnosed with a common benign form of gestational trophoblastic disease. The nurse identifies this as: hydramnios. hydatidiform mole. placenta accrete. ectopic pregnancy.

hydatidiform mole.

A woman is admitted with a diagnosis of ectopic pregnancy. For which procedure should the nurse prepare?

immediate surgery

A woman is admitted with a diagnosis of ectopic pregnancy. For which procedure should the nurse prepare? internal uterine monitoring immediate surgery bed rest for the next 4 weeks intravenous administration of a tocolytic

immediate surgery

A 25-year-old client at 22 weeks' gestation is noted to have proteinuria and dependent edema on her routine prenatal visit. Which additional assessment should the nurse prioritize and convey to the RN or health care provider? initial BP 120/80mm Hg; current BP 130/88 mm Hg initial BP 100/70 mm Hg; current BP 140/90 mm Hg initial BP 140/85 mm Hg; current BP 130/80 mm Hg initial BP 110/60 mm Hg; current BP 112/86 mm Hg

initial BP 100/70 mm Hg; current BP 140/90 mm Hg Explanation: A proteinuria of trace to 1+ and a rise in blood pressure to above 140/90 mm Hg is a concern the client may be developing preeclampsia. The blood pressures noted in the other options are not indicative of developing preeclampsia. The edema would not necessarily be indicative of preeclampsia; however, edema of the face and hands would be a concerning sign for severe preeclampsia.

Which measure would the nurse include in the plan of care for a woman with prelabor rupture of membranes if her fetus's lungs are mature? -reduction in physical activity level -observation for signs of infection -administration of corticosteroids -labor induction

labor induction With prelabor rupture of membranes (PROM) in a woman whose fetus has mature lungs, induction of labor is initiated. Reducing physical activity, observing for signs of infection, and giving corticosteroids may be used for the woman with PROM when the fetal lungs are immature.

Some women experience a rupture of their membranes before going into true labor. A nurse recognizes that a woman who presents with preterm premature rupture of membranes (PPROM) has completed how many weeks of gestation? -less than 37 weeks -less than 38 weeks -less than 39 weeks -less than 40 weeks

less than 37 weeks Preterm premature rupture of membranes (PPROM) is defined as the rupture of the membranes prior to the onset of labor in a woman who is less than 37 weeks' gestation. PROM (premature rupture of membranes) refers to a woman who is beyond 37 weeks' gestation, has presented with spontaneous rupture of the membranes, and is not in labor.

What would be the physiologic basis for a placenta previa?

low placental implantation The cause of placenta previa is usually unknown, but for some reason the placenta is implanted low instead of high on the uterus.

A client in her 20th week of gestation develops HELLP syndrome. What are features of HELLP syndrome? Select all that apply.

low platelet count elevated liver enzymes hemolysis The HELLP syndrome is a syndrome involving hemolysis (microangiopathic hemolytic anemia), elevated liver enzymes, and a low platelet count. Hyperthermia and leukocytosis are not features of HELLP syndrome.

A nurse is reviewing the medical record of a pregnant client diagnosed with placenta previa. The physical exam reveals that the placenta is implanted near the internal os but does not reach it. The nurse interprets this as which type of placenta previa?

low-lying

Which medication will the nurse anticipate the health care provider will prescribe as treatment for an unruptured ectopic pregnancy? oxytocin promethazine ondansetron methotrexate

methotrexate

A 32-year-old gravida 3 para 2 at 36 weeks' gestation comes to the obstetric department reporting abdominal pain. Her blood pressure is 164/90 mm Hg, her pulse is 100 beats per minute, and her respirations are 24 per minute. She is restless and slightly diaphoretic with a small amount of dark red vaginal bleeding. What assessment should the nurse make next?

palpate the fundus and check fetal heart rate The classic signs of placental abruption (abruptio placentae) are pain, dark red vaginal bleeding, a rigid, board-like abdomen, hypertonic labor, and fetal distress.

A client with a history of cervical insufficiency is seen for reports of pink-tinged discharge and pelvic pressure. The primary care provider decides to perform a cervical cerclage. The nurse teaches the client about the procedure. Which client response indicates that the teaching has been effective? "The cervix is glued shut so no amniotic fluid can escape." "Staples are put in the cervix to prevent it from dilating." "Purse-string sutures are placed in the cervix to prevent it from dilating." "A cervical cap is placed so no amniotic fluid can escape."

"Purse-string sutures are placed in the cervix to prevent it from dilating."

A 32-year-old gravida 3 para 2 at 36 weeks' gestation comes to the obstetric department reporting abdominal pain. Her blood pressure is 164/90 mm/Hg, her pulse is 100 beats per minute, and her respirations are 24 per minute. She is restless and slightly diaphoretic with a small amount of dark red vaginal bleeding. What assessment should the nurse make next? a) Obtain a voided urine specimen and determine blood type. b) Check deep tendon reflexes. c) Palpate the fundus and check fetal heart rate. d) Measure fundal height.

...

During pregnancy a woman's blood volume increases to accommodate the growing fetus to the point that vital signs may remain within normal range without showing signs of shock until the woman has lost what percentage of her blood volume

40%

b. pregnancy loss

A 44-year-old client has lost several pregnancies over the last 10 years. For the past 3 months, she has had fatigue, nausea, and vomiting. She visits the clinic and takes a pregnancy test; the results are positive. Physical examination confirms a uterus enlarged to 13 weeks' gestation; fetal heart tones are heard. Ultrasound reveals that the client is experiencing some bleeding. Considering the client's prenatal history and age, what does the nurse recognize as the greatest risk for the client at this time? a. premature birth b. pregnancy loss c. preterm labor d. hypertension

c. "I can understand your need to find an answer to what caused this. Let's talk about this further."

A client at 11 weeks' gestation experiences pregnancy loss. The client asks the nurse if the bleeding and cramping that occurred during the miscarriage were caused by working long hours in a stressful environment. What is the most appropriate response from the nurse? a. "It is hard to know why a woman bleeds during early pregnancy." b. "Your spontaneous bleeding is not work-related." c. "I can understand your need to find an answer to what caused this. Let's talk about this further." d. "Something was wrong with the fetus."

a. Assess the client's vital signs.

A nurse in the maternity triage unit is caring for a client with a suspected ectopic pregnancy. Which nursing intervention should the nurse perform first? a. Assess the client's vital signs. b. Provide emotional support to the client and significant other. c. Obtain a surgical consent from the client. d. Administer oxygen to the client.

b. "Come to the health facility with any vaginal material passed."

A woman of 16 weeks' gestation telephones the nurse because she has passed some "berry-like" blood clots and now has continued dark brown vaginal bleeding. Which action would the nurse instruct the woman to do? a. "Come to the health care facility if uterine contractions begin." b. "Come to the health facility with any vaginal material passed." c. "Continue normal activity, but take the pulse every hour." d. "Maintain bed rest, and count the number of perineal pads used."

a. ensures passage of all the products of conception

A woman with an incomplete abortion is to receive misoprostol. The woman asks the nurse, "Why am I getting this drug?" The nurse responds to the client, integrating understanding that this drug achieves which effect? a. ensures passage of all the products of conception b. halts the progression of the abortion c. suppresses the immune response to prevent isoimmunization d. alleviates strong uterine cramping

A primigravida 28-year-old client is noted to have Rh negative blood and her husband is noted to be Rh positive. The nurse should prepare to administer RhoGAM after which diagnostic procedure?

Amniocentesis Amniocentesis is a procedure requiring a needle to enter into the amniotic sac. There is a risk of mixing of the fetal and maternal blood which could result in blood incompatibility. A contraction test, a nonstress test, and biophysical profile are not invasive, so there would be no indication for Rho(D) immune globulin to be administered.

A nurse in the maternity triage unit is caring for a client with a suspected ectopic pregnancy. Which nursing intervention should the nurse perform first?

Assess the client's vital signs. A suspected ectopic pregnancy can put the client at risk for hypovolemic shock. The assessment of vital signs should be performed first, followed by any procedures to maintain the ABCs. Providing emotional support would also occur, as would obtaining a surgical consent, if needed, but these are not first steps.

A woman in labor suddenly reports sharp fundal pain accompanied by slight dark red vaginal bleeding. The nurse should prepare to assist with which situation?

Premature separation of the placenta Premature separation of the placenta begins with sharp fundal pain, usually followed by dark red vaginal bleeding. Placenta previa usually produces painless bright red bleeding. Preterm labor contractions are more often described as cramping. Possible fetal death or injury does not present with sharp fundal pain. It is usually painless.

The following hourly assessments are obtained by the nurse on a patient with preeclampsia receiving Magnesium Sulfate: 97.3, P88, R10, blood pressure 148/110. What other priority physical assessment by the nurse should be implemented to assess for potential toxicity? a) Lung sounds b) Magnesium sulfate level c) Reflexes d) Oxygen saturation

Reflexes Correct Explanation: Reflex assessment is part of the standard assessment for patients on magnesium sulfate. The first change when developing magnesium toxicity may be a decrease in reflex activity. The health care provider needs to be notified immediately. A change in lung sounds and oxygen saturation are not indicative of magnesium sulfate toxicity. Hourly blood draws to gain information on the magnesium sulfate level are not indicated.

The nurse is caring for a woman at 32 weeks' gestation with severe preeclampsia. Which assessment finding should the nurse prioritize after the administration of hydralazine to this client?

Tachycardia Explanation: Hydralazine reduces blood pressure but is associated with adverse effects such as palpitation, tachycardia, headache, anorexia, nausea, vomiting, and diarrhea. It does not cause gastrointestinal bleeding, blurred vision (halos around lights), or sweating. Magnesium sulfate may cause sweating.

The nurse is caring for a client who has remained in stable condition at 37 weeks' gestation. The client's condition suddenly changes. Which assessment change should the nurse prioritize? -Vaginal bleeding and no pain -Uterine contractions with vaginal mucus -Fundal height and fetal heart rate -Size and contour of the abdomen

Vaginal bleeding and no pain Placenta previa includes bright red and painless vaginal bleeding, which is different from the dark red bleeding of placental abruption (abruptio placenta) accompanied by severe pain. This differentiates the two conditions. Uterine contractions with vaginal mucus may be indications of the start of labor with the mucus plug being discharged. The fetal heart rate, fundal height, and contour of the abdomen are normal components that are assessed during the labor process.

just know this part 1

When hospitalization is necessary, oral food and fluids are withheld to allow the gut to rest. Antiemetic agents are ordered to help control nausea and vomiting. The woman is likely to be dehydrated, so the nurse would obtain baseline blood electrolyte levels and administer intravenous fluid .

A primipara at 36 weeks' gestation is being monitored in the prenatal clinic for risk of preeclampsia. Which sign or symptom should the nurse prioritize?

a dipstick value of 2+ for protein The increasing amount of protein in the urine is a concern the preeclampsia may be progressing to severe preeclampsia. The woman needs further assessment by the health care provider. Dependent edema may be seen in a majority of pregnant women and is not an indicator of progression from preeclampsia to eclampsia. Weight gain is no longer considered an indicator for the progression of preeclampsia. A systolic blood pressure increase is not the highest priority concern for the nurse, since there is no indication what the baseline blood pressure was.

A client in her 20th week of gestation develops HELLP syndrome. Which of the following should the nurse consider as features of HELLP syndrome? Select all that apply. a) Elevated liver enzymes b) Leukocytosis c) Low platelet count d) Hemolysis e) Hyperthermia

a) Elevated liver enzymes c) Low platelet count d) Hemolysis Rationale: The HELLP syndrome is a syndrome involving hemolysis (microangiopathic hemolytic anemia), elevated liver enzymes, and a low platelet count. Hyperthermia and leukocytosis are not features of HELLP syndrome.

A pregnant woman is admitted to the hospital with a diagnosis of placenta previa. Which action would be the priority for this woman on admission? • performing a vaginal examination to assess the extent of bleeding • helping the woman remain ambulatory to reduce bleeding • assessing fetal heart tones by use of an external monitor • assessing uterine contractions by an internal pressure gauge

assessing fetal heart tones by use of an external monitor

A pregnant woman is admitted to the hospital with a diagnosis of placenta previa. Which action would be the priority for this woman on admission?

assessing fetal heart tones by use of an external monitor Not disrupting the placenta is a prime responsibility in caring for a patient with placenta previa, so an external fetal monitor would be used. An internal monitor, a vaginal examination, and remaining ambulatory could all disrupt the placenta and thus are contraindicated.

The nurse is required to assess a pregnant client who is reporting vaginal bleeding. Which nursing action is the priority? monitoring uterine contractility assessing signs of shock determining the amount of funneling assessing the amount and color of the bleeding

assessing the amount and color of the bleeding Explanation: When the woman arrives and is admitted, assessing her vital signs, the amount and color of the bleeding, and current pain rating on a scale of 1 to 10 are the priorities. Assessing the signs of shock, monitoring uterine contractility, and determining the amount of funneling are not priority assessments when a pregnant woman complaining of vaginal bleeding is admitted to the hospital.

A woman of 16 weeks' gestation telephones you because she has passed some "berry-like" blood clots and now has continued dark brown vaginal bleeding. Which of the following would you instruct the woman to do? a) "Come to the health care facility if uterine contractions begin." b) "Come to the health facility with any vaginal material passed." c) "Maintain bed rest and count the number of perineal pads used." d) "Continue normal activity, but take your pulse every hour."

b) "Come to the health facility with any vaginal material passed." Rationale: This is a typical time in pregnancy for gestational trophoblastic disease to present. Asking the woman to bring any material passed vaginally would be important so it can be assessed for this.

A pregnant woman is admitted to the hospital with a diagnosis of placenta previa. Which of the following would be the priority for this woman on admission? a) Performing a vaginal examination to assess the extent of bleeding b) Helping the woman remain ambulatory to reduce bleeding c) Assessing fetal heart tones by use of an external monitor d) Assessing uterine contractions by an internal pressure gauge

c) Assessing fetal heart tones by use of an external monitor Rationale: Not disrupting the placenta is a prime responsibility. An internal monitor, a vaginal examination, and remaining ambulatory could all do this and thus are contraindicated.

A woman with severe preeclampsia is receiving magnesium sulfate. The woman serum magnesium level is 9.0mEq/L. Which finding would the nurse most likely note?

diminished reflexes Diminished or absent reflexes occur when a client develops magnesium toxicity, serum levels greater than 8.0 mEq/L. Elevated liver enzymes are unrelated to magnesium toxicity and may indicate the development of HELLP syndrome. The onset of seizure activity indicates eclampsia. A serum magnesium level of 6.5 mEq/L would fall within the therapeutic range of 4 to 7 mEq/L.

A woman is being closely monitored and treated for severe preeclampsia with magnesium sulfate. Which finding would alert the nurse to the development of magnesium toxicity in this client? diminished reflexes elevated liver enzymes seizures serum magnesium level of 6.5 mEq/L

diminished reflexes Explanation: Diminished or absent reflexes occur when a client develops magnesium toxicity. Elevated liver enzymes are unrelated to magnesium toxicity and may indicate the development of HELLP syndrome. The onset of seizure activity indicates eclampsia. A serum magnesium level of 6.5 mEq/L would fall within the therapeutic range of 4 to 7 mEq/L.

A home health care nurse is visiting a pregnant client with preeclampsia who is being managed at home. The nurse is reviewing the situations for which the client should contact the nurse. The nurse determines that the client demonstrates understanding when identifying which situation(s) as needing to be reported? Select all that apply. blurred vision excessive heartburn increased urination dizziness sinus headache

dizziness excessive heartburn blurred vision

A client comes to a clinic asking for advice on how to prevent diabetes. What suggestion is BEST for the nurse to give the client? `

eat an adequate intake of high fiber foods A high-fiber diet can slow the absorption of sugar and help to improve blood sugar levels.

A 28-year-old woman presents in the emergency department with severe abdominal pain. She has not had a normal period for 2 months, but she reports that that is not abnormal for her. She has a history of endometriosis. What might the nurse suggest to the primary care provider as a possible cause of the client's abdominal pain?

ectopic pregnancy

A woman with an incomplete abortion is to receive misoprostol. The woman asks the nurse, "Why am I getting this drug?" The nurse responds to the client, integrating understanding that this drug achieves which effect? halts the progression of the abortion ensures passage of all the products of conception alleviates strong uterine cramping suppresses the immune response to prevent isoimmunization

ensures passage of all the products of conception

A woman at 9 weeks' gestation was unable to control the nausea and vomiting of hyperemesis gravidarum through conservative measures at home. With nausea and vomiting becoming severe, the woman was omitted to the obstetrical unit. Which action should the nurse prioritize?

establish IV for rehydration With severe nausea and vomiting the client may be dehydrated upon coming to hospital for assistance, so establishing an IV line is the priority intervention. This will also allow for hydration, and if needed, the administration of an antiemetic to bypass the gastrointestinal tract. Although the nurse will explain the NPO status to the client (so that vomiting may be brought under control) and the likelihood of being placed on bed rest with bathroom privileges, these teaching are not the priority.

A nurse is providing care to a client who has been diagnosed with a common benign form of gestational trophoblastic disease. The nurse identifies this as: hydatidiform mole. ectopic pregnancy. placenta accrete. hydramnios.

hydatidiform mole. Explanation: Gestational trophoblastic disease comprises a spectrum of neoplastic disorders that originate in the placenta. The two most common types are hydatidiform mole (partial or complete) and choriocarcinoma. Hydatidiform mole is a benign neoplasm of the chorion in which the chorionic villi degenerate and become transparent vesicles containing clear, viscid fluid. Ectopic pregnancy, placenta accreta, and hydramnios fall into different categories of potential pregnancy

The woman with severe preeclampsia should be

in complete bed rest, in a dark and quiet room to avoid stimulation

What would be the physiologic basis for a placenta previa?

low placental implantation The cause of placenta previa is usually unknown, but for some reason the placenta is implanted low instead of high on the uterus.

The nurse is preparing the plan of care for a woman hospitalized for hyperemesis gravidarum. Which interventions would the nurse most likely include? Select all that apply.

maintaining NPO status for the first day or two administering antiemetic agents obtaining baseline blood electrolyte levels monitoring intake and output When hospitalization is necessary, oral food and fluids are withheld to allow the gut to rest. Antiemetic agents are ordered to help control nausea and vomiting. The woman is likely to be dehydrated, so the nurse would obtain baseline blood electrolyte levels and administer intravenous fluid and electrolyte replacement therapy as indicated. Once the nausea and vomiting subside, oral food and fluids are gradually reintroduced. Total parenteral nutrition or a feeding tube is used to prevent malnutrition only if the client does not improve with these interventions.

Which medication will the nurse anticipate the health care provider will prescribe as treatment for an unruptured ectopic pregnancy? ondansetron promethazine methotrexate oxytocin

methotrexate

A pregnant client with severe preeclampsia has developed HELLP syndrome. In addition to the observations necessary for preeclampsia, what other nursing intervention is critical for this client?

observation for bleeding Explanation: Because of the low platelet count associated with this condition, women with HELLP syndrome need extremely close observation for bleeding, in addition to the observations necessary for preeclampsia. Maintaining a patent airway is a critical intervention needed for a client with eclampsia while she is having a seizure. Administration of a tocolytic would be appropriate for halting labor. Monitoring for infection is not a priority intervention in this situation.

After a regular prenatal visit, a pregnant client asks the nurse to describe the differences between placental abruption (abruptio placentae) and placenta previa. Which statement will the nurse include in the teaching?

placenta previa is an abnormally implanted placenta that is too close to the cervix Placenta previa is a condition of pregnancy in which the placenta is implanted abnormally in the lower part of the uterus and is the most common cause of painless, bright red bleeding in the third trimester. Placental abruption is the premature separation of a normally implanted placenta that pulls away from the wall of the uterus either during pregnancy or before the end of labor. Placental abruption can result in concealed or apparent dark red bleeding and is painful. Immediate intervention is required for placental abruption.

The nurse is facilitating a nursing class regarding the difference between type 1 and type 2 diabetes. Which type of diabetes has the risk factors of family history, increasing age, obesity, physical inactivity, ethnicity, and a history of gestational diabetes?

type 2 Risk factors for type 2 diabetes include family history, increasing age, obesity, physical inactivity, ethnicity, and a history of gestational diabetes. Family history, diet, and environmental factors are risk factors for type 1 diabetes. Studies have found an increased risk in children whose parents have type 1 diabetes, and this risk increases with maternal age.

The nurse is teaching a prenatal class on potential problems during pregnancy to a group of expectant parents. The risk factors for placental abruption (abruptio placentae) are discussed. Which comment validates accurate learning by the parents? "If I develop this complication, I will have bright red vaginal bleeding," "Since I am over 30, I run a much higher risk of developing this problem." "I need a cesarean section if I develop this problem." "Placental abruption is quite painful and I will need to let the doctor know if I begin to have abdominal pain."

"Placental abruption is quite painful and I will need to let the doctor know if I begin to have abdominal pain."

c. Twin-to-twin transfusion syndrome (TTTS)

A young mother gives birth to twin boys who shared the same placenta. What serious complication are they at risk for? a. ABO incompatibility b. TORCH syndrome c. Twin-to-twin transfusion syndrome (TTTS) d. HELLP syndrome

A 28-year-old client with a history of endometriosis presents to the emergency department with severe abdominal pain and nausea and vomiting. The client also reports her periods are irregular with the last one being 2 months ago. The nurse prepares to assess for which possible cause for this client's complaints?

Ectopic pregnancy

A woman at 37 weeks gestation presents to the labor and delivery area with symptoms of abruptio placentae. Which action should the nurse prioritize?

Ensure large bore IV access is obtained

A woman with a recent incomplete abortion is to receive therapeutic misoprostol. The nurse understands that the rationale for administering this drug is to:

Ensure passage of all the products of conception

C. to prevent progression of preeclampsia into seizures.

When administering magnesium sulfate to a client with preeclampsia, the nurse explains to her that this drug is given to: a. Reduce blood pressure b. Increase the progress of labor c. Prevent seizures d. Lower blood glucose levels

The nurse is preparing discharge instructions for several clients after their admission for emergent care of a pregnancy complication. The nurse will stress the importance of frequent office visits to the client with:

a molar pregnancy Molar pregnancies can indicate the possibility of developing malignancy. The woman will need close observation and follow-up for a year, every 1 to 2 weeks for hCG levels to detect cancer. A follow-up visit after an ectopic pregnancy or a complete spontaneous abortion (miscarriage) are typically scheduled at 6 weeks, not monthly. A woman who is Rh negative does not need a follow-up visit because of her Rh status, but would be scheduled as per routine postpartum visits.

A pregnant woman is admitted to the hospital with a diagnosis of placenta previa. Which action would be the priority for this woman on admission? assessing uterine contractions by an internal pressure gauge assessing fetal heart tones by use of an external monitor helping the woman remain ambulatory to reduce bleeding performing a vaginal examination to assess the extent of bleeding

assessing fetal heart tones by use of an external monitor

What makes the diagnosis of gestational hypertension different from the diagnosis of preeclampsia? a) Ketonuria b) Proteinuria c) Severity of hypertension d) The hypertension of gestation disappears after delivery. The hypertension of preeclampsia does not.

b) Proteinuria Rationale: Gestational hypertension is the current term used to describe elevated blood pressure (greater than or equal to 140/90 mm/Hg) that develops for the first time during pregnancy without the presence of protein in the urine.

The nurse is comforting and listening to a young couple who just suffered a spontaneous abortion (miscarriage). When asked why this happened, which reason should the nurse share as a common cause?

chromosomal abnormality The most common cause for the loss of a fetus in the first trimester is associated with a genetic defect or chromosomal abnormality. There is nothing that can be done and the mother should feel no fault. The nurse needs to encourage the parents to speak with a health care provider for further information and questions related to genetic testing. Early pregnancy loss is not associated with maternal smoking, lack of prenatal care, or the age of the mother.

A pregnant client in her 22nd week of gestation arrives at the healthcare facility with complaints of excessive vaginal bleeding and absence of fetal movements. She is diagnosed as having second trimester fetal loss. Which of the following would the nurse anticipate as the cause of second trimester fetal loss? a) Ectopic pregnancy b) Congenital malformations c) Placenta previa d) Cervical incompetence

d) Cervical incompetence Rationale: The nurse should identify cervical incompetence as the cause for second trimester fetal loss. Cervical incompetence is a condition where there is painless cervical dilatation and results in second trimester fetal loss or can progress to preterm premature rupture of membranes. Ectopic pregnancy, congenital malformations and placenta previa are not involved in causing second trimester fetal loss. Ectopic pregnancy usually leads to first trimester fetal loss. Placenta previa is a condition in which there is implantation of the placenta to the lower uterine segment. Congenital malformations result in first trimester fetal loss.

A woman is being closely monitored and treated for severe preeclampsia with magnesium sulfate. Which finding would alert the nurse to the development of magnesium toxicity in this client? diminished reflexes elevated liver enzymes seizures serum magnesium level of 6.5 mEq/L

diminished reflexes

A woman with severe preeclampsia is receiving magnesium sulfate. The woman's serum magnesium level is 9.0 mEq/L. Which finding would the nurse most likely note?

diminished reflexes

A nurse is conducting a refresher program for a group of perinatal nurses. Part of the program involves a discussion of HELLP. The nurse determines that the group needs additional teaching when they identify which aspect as a part of HELLP?

elevated lipoproteins

A nurse is conducting a refresher program for a group of perinatal nurses. Part of the program involves a discussion of HELLP. The nurse determines that the group needs additional teaching when they identify which aspect as a part of HELLP? elevated lipoproteins hemolysis liver enzyme elevation low platelet count

elevated lipoproteins Explanation: The acronym HELLP represents hemolysis, elevated liver enzymes, and low platelets. This syndrome is a variant of preeclampsia/eclampsia syndrome that occurs in 10% to 20% of clients whose diseases are labeled as severe.

A woman at 10 weeks gestation comes to the clinic for an evaluation. Which assessment finding should the nurse prioritize? report of frequent mild nausea blood pressure of 120/84 mm Hg history of bright red spotting 6 weeks ago fundal height measurement of 18 cm

fundal height measurement of 18 cm

A client tells that nurse in the doctor's office that her friend developed high blood pressure on her last pregnancy. She is concerned that she will have the same problem. What is the standard of care for preeclampsia?

have her blood pressure checked at every prenatal visit Preeclampsia and eclampsia are common problems for pregnant clients and require regular blood pressure monitoring at all prenatal visits. Antihypertensives are not prescribed unless the client is already hypertensive. Monitoring for headaches and swelling is a good predictor of a problem but doesn't address prevention—nor does it predict who will have hypertension. Taking aspirin has shown to reduce the risk in women who have moderate to high risk factors, but has shown no effect on those women with low risk factors.

two types of gestional tropholastic disease

hydatidiform mole (partial or complete) and choriocarcinoma

A client is seeking advice for his pregnant wife who is experiencing mild elevations in blood pressure. In which position should a nurse recommend the pregnant client rest?

lateral recumbent position The nurse should encourage a client with mild elevations in blood pressure to rest as much as possible in the lateral recumbent position to improve uteroplacental blood flow, reduce blood pressure, and promote diuresis. The nurse should maintain the client with severe preeclampsia on complete bed rest in the left lateral lying position. Keeping the head of the bed slightly elevated will not help to improve the condition of the client with mild elevations in blood pressure.

Severe preeclampsia

may develop suddenly and bring with it high blood pressure of more than 160/110 mm Hg, proteinuria of more than 5 g in 24 hours, oliguria of less than 400 mL in 24 hours, cerebral and visual symptoms, and rapid weight gain

A pregnant woman with preeclampsia is to receive magnesium sulfate IV. Which assessment should the nurse prioritize before administering a new dose? heart rate anxiety level blood pressure patellar reflex

patellar reflex

Cerclage

treatment for cervical insufficiency.

The nurse is teaching a prenatal class on potential problems during pregnancy to a group of expectant parents. The risk factors for placental abruption (abruptio placentae) are discussed. Which comment validates accurate learning by the parents? "If I develop this complication, I will have bright red vaginal bleeding," "Placental abruption is quite painful and I will need to let the doctor know if I begin to have abdominal pain." "I need a cesarean section if I develop this problem." "Since I am over 30, I run a much higher risk of developing this problem."

"Placental abruption is quite painful and I will need to let the doctor know if I begin to have abdominal pain."

A client with a history of cervical insufficiency is seen for reports of pink-tinged discharge and pelvic pressure. The primary care provider decides to perform a cervical cerclage. The nurse teaches the client about the procedure. Which client response indicates that the teaching has been effective? "Purse-string sutures are placed in the cervix to prevent it from dilating." "Staples are put in the cervix to prevent it from dilating." "A cervical cap is placed so no amniotic fluid can escape." "The cervix is glued shut so no amniotic fluid can escape."

"Purse-string sutures are placed in the cervix to prevent it from dilating."

A 16-year-old client gave birth to a 12 weeks' gestation fetus last week. The client has come to the office for follow-up and while waiting in an examination room notices that on the schedule is written her name and "follow-up of spontaneous abortion." The client is upset about what is written on the schedule. How can the nurse best explain this terminology?

"Spontaneous abortion is a more specific term used to describe a spontaneous miscarriage, which is a loss of pregnancy before 20 weeks. This term does not imply that you did anything to affect the pregnancy." Abortion is a medical term for any interruption of a pregnancy before a fetus is viable, but it is better to speak of these early pregnancy losses as spontaneous abortions to avoid confusion with intentional terminations of pregnancies. The other responses are correct, but they do not provide the client with the most complete and reassuring answer.

d. Respiratory rate

A client at 37 weeks' gestation presents to the emergency department with a BP 150/108 mm Hg, 1+ pedal edema, 1+ proteinuria, and normal deep tendon reflexes. Which assessment should the nurse prioritize as the client is administered magnesium sulfate IV? a. Urine protein b. Ability to sleep c. Hemoglobin d. Respiratory rate

a. elevated hCG levels d. absence of fetal heart sound e. hyperemesis gravidarum

A client visits a health care facility reporting amenorrhea for 10 weeks, fatigue, and breast tenderness. Which assessment findings should the nurse prioritize for immediate intervention? (Select all that apply.) a. elevated hCG levels b. dyspareunia c. whitish discharge from the vagina d. absence of fetal heart sound e. hyperemesis gravidarum

a. Assess deep tendon reflexes

A client with preeclampsia is receiving magnesium sulfate to suppress or control seizures. Which nursing intervention should a nurse perform to determine the effectiveness of therapy? a. Assess deep tendon reflexes b. Monitor intake and output c. Assess client's mucous membrane d. Assess client's skin turgor

b. proteinuria c. hypereflexia d. blurring of vision

A nursing instructor is conducting a session exploring the signs and symptoms of eclampsia to a group of student nurses. The instructor determines the session is successful after the students correclty choose which signs indicating eclampsia? (Select all that apply.) a. auditory hallucinations b. proteinuria c. hypereflexia d. blurring of vision e. hyperglycemia

a. Prolapsed cord b. Abruptio placenta e. Preterm labor

A pregnant client is brought to the health care facility with signs of premature rupture of the membranes (PROM). Which conditions and complications are associated with PROM? (Select all that apply.) a. Prolapsed cord b. Abruptio placenta c. Spontaneous abortion d. Placenta previa e. Preterm labor

A woman with severe preeclampsia is receiving magnesium sulfate. The woman serum magnesium level is 9.0mEq/L. Which finding would the nurse most likely note?

A: diminished reflexes Diminished or absent reflexes occur when a client develops magnesium toxicity, serum levels greater than 8.0 mEq/L. Elevated liver enzymes are unrelated to magnesium toxicity and may indicate the development of HELLP syndrome. The onset of seizure activity indicates eclampsia. A serum magnesium level of 6.5 mEq/L would fall within the therapeutic range of 4 to 7 mEq/L.

A nurse is caring for a client with hyperemesis gravidarum. Which nursing action is the priority for this client?

Administer IV normal saline with vitamins and electrolytes. Explanation: The first choice for fluid replacement is generally normal saline with vitamins and electrolytes added. If the client does not improve after several days of bed rest, "gut rest," IV fluids, and antiemetics, then total parenteral nutrition or percutaneous endoscopic gastrostomy tube feeding is instituted to prevent malnutrition.

A pregnant patient is diagnosed with preterm labor. What should the nurse teach the patient to help prevent the reoccurrence of preterm labor? Select all that apply.

Drink 8 to 10 glasses of fluid each day. Report any signs of ruptured membranes. Remain on bed rest except to use the bathroom. To reduce the onset of preterm labor, the nurse should instruct the patient to drink 8 to 10 glasses of fluid each day to remain hydrated. The patient should also report any signs of ruptured membranes and remain on bed rest unless using the bathroom. Should uterine contractions begin, the patient should be instructed to lie on either the right or left side to increase blood return to the uterus. The patient should not engage in any activity other than bed rest with bathroom privileges.

A 28-year-old client with a history of endometriosis presents to the emergency department with severe abdominal pain and nausea and vomiting. The client also reports her periods are irregular with the last one being 2 months ago. The nurse prepares to assess for which possible cause for this client's complaints? Healthy pregnancy Ectopic pregnancy Molar pregnancy Placenta previa

Ectopic pregnancy

The nurse is preparing the plan of care for a woman hospitalized for hyperemesis gravidarum. Which interventions would the nurse most likely include? Select all that apply. administering antiemetic agents monitoring intake and output maintaining NPO status for the first day or two preparing the woman for insertion of a feeding tube obtaining baseline blood electrolyte levels

Everything but preparing for insertion of feeding tube

Medications Related to Abortions Misoprostol (Cytotec)

Indication: Stimulates uterine contractions to terminate a pregnancy and to evacuate the uterus after abortion to ensure passage of all the products of conception Nursing Implications: -Monitor for side effects such as diarrhea, abdominal pain, nausea, vomiting, and dyspepsia. -Assess vaginal bleeding, and report any increased bleeding, pain, or fever. -Monitor for signs and symptoms of shock, such as tachycardia, hypotension, and anxiety.

The nurse is caring for a pregnant client with fallopian tube rupture. Which intervention is the priority for this client? Monitor the fetal heart rate (FHR). Monitor the client's vital signs and bleeding. Monitor the mass with transvaginal ultrasound. Monitor the client's beta-hCG level.

Monitor the client's vital signs and bleeding.

A 25-week-gestation client presents with a blood pressure of 152/99, pulse 78, no edema, and urine negative for protein. What would the nurse do next? Notify the health care provider Provide health education Assess the client for ketonuria Document the client's blood pressure

Notify the health care provider

A woman at 34 weeks' gestation presents to labor and delivery with vaginal bleeding. Which finding from the obstetric examination would lead to a diagnosis of placental abruption (abruptio placentae)? Onset of vaginal bleeding was sudden and painful Fetus is in a breech position Sonogram shows the placenta covering the cervical os Uterus is soft between contractions

Onset of vaginal bleeding was sudden and painful Explanation: Sudden onset of abdominal pain and vaginal bleeding with a rigid uterus that does not relax are signs of a placental abruption (abruptio placentae). The other findings are consistent with a diagnosis of placenta previa.

A 32-year-old gravida 3 para 2 at 36 weeks' gestation comes to the obstetric department reporting abdominal pain. Her blood pressure is 164/90 mm/Hg, her pulse is 100 beats per minute, and her respirations are 24 per minute. She is restless and slightly diaphoretic with a small amount of dark red vaginal bleeding. What assessment should the nurse make next?

Palpate the fundus and check fetal heart rate.

A 32-year-old gravida 3 para 2 at 36 weeks' gestation comes to the obstetric department reporting abdominal pain. Her blood pressure is 164/90 mm/Hg, her pulse is 100 beats per minute, and her respirations are 24 per minute. She is restless and slightly diaphoretic with a small amount of dark red vaginal bleeding. What assessment should the nurse make next?

Palpate the fundus, and check fetal heart rate.

A woman in labor suddenly reports sharp fundal pain accompanied by slight dark red vaginal bleeding. The nurse should prepare to assist with which situation? Preterm labor that was undiagnosed Possible fetal death or injury Placenta previa obstructing the cervix Premature separation of the placenta

Premature separation of the placenta

A woman in labor suddenly reports sharp fundal pain accompanied by slight dark red vaginal bleeding. The nurse should prepare to assist with which situation?

Premature separation of the placenta Explanation: Premature separation of the placenta begins with sharp fundal pain, usually followed by dark red vaginal bleeding. Placenta previa usually produces painless bright red bleeding. Preterm labor contractions are more often described as cramping. Possible fetal death or injury does not present with sharp fundal pain. It is usually painless.

The following hourly assessments are obtained by the nurse on a client with preeclampsia receiving magnesium sulfate: 97.3oF (36.2oC), HR 88, RR 12 breaths/min, BP 148/110 mm Hg. What other priority physical assessments by the nurse should be implemented to assess for potential toxicity? Reflexes Lung sounds Oxygen saturation Magnesium sulfate level

Reflexes

A nurse is providing education to a woman at 28 weeks' gestation who has tested positive for gestational diabetes mellitus (GDM). What would be important for the nurse to include in the client teaching? She is at increased risk for type 1 diabetes mellitus after her baby is born. She is at increased risk for type 2 diabetes mellitus after her baby is born. Her baby is at increased risk for type 1 diabetes mellitus. Her baby is at increased risk for neonatal diabetes mellitus.

She is at increased risk for type 2 diabetes mellitus after her baby is born.

A patient is admitted to labor and delivery for management of severe preeclampsia. An IV infusion of magnesium sulfate is started. What is the primary goal for magnesium sulfate therapy? a) Prevent maternal seizures b) Reverse edema c) Decrease blood pressure d) Decrease protein in urine

a) Prevent maternal seizures Rationale: The primary therapy goal for any preeclamptic patient is to prevent maternal seizures. Use of magnesium sulfate is the drug therapy of choice for severe preeclampsia and is only used to manage and attempt to prevent progression to eclampsia. Magnesium sulfate therapy does not have as its primary goal a decrease in blood pressure, a decrease in protein in the urine, nor the reversal of edema.

A woman in labor is at risk for abruptio placentae. Which of the following assessments would most likely lead you to suspect that this has happened? a) Sharp fundal pain and discomfort between contractions. b) Painless vaginal bleeding and a fall in blood pressure. c) An increased blood pressure and oliguria. d) Pain in a lower quadrant and increased pulse rate.

a) Sharp fundal pain and discomfort between contractions. Rationale: An abruptio placentae refers to premature separation of the placenta from the uterus. As the placenta loosens, it causes sharp pain. Labor begins with a continuing nagging sensation. Painless vaginal bleeding and a fall in blood pressure are indicative of placenta previa. Pain in a lower quadrant and increased pulse rate are indicative of an ectopic pregnancy. Hypertension and oliguria are indicative of preeclampsia.

A client in her first trimester has just experienced a miscarriage. The nurse knows that which of the following is the most likely cause of the miscarriage? a) Lack of sufficient progesterone produced by the corpus luteum b) Abnormal fetal development c) Rejection of the embryo through an immune response d) Implantation abnormality

b) Abnormal fetal development Rationale: The most frequent cause of miscarriage in the first trimester of pregnancy is abnormal fetal development, due either to a teratogenic factor or to a chromosomal aberration. In other miscarriages, immunologic factors may be present or rejection of the embryo through an immune response may occur. Another common cause of early miscarriage involves implantation abnormalities. Miscarriage may also occur if the corpus luteum on the ovary fails to produce enough progesterone to maintain the decidua basalis.

A pregnant client with multiple gestation arrives at the maternity clinic for a regular antenatal check up. The nurse would be aware of the client's risk of perinatal complications including which of the following? a) Maternal hypotension b) Congenital anomalies c) Post-term birth d) Fetal non-immune hydrops

b) Congenital anomalies Rationale: Multiple gestation involves two or more fetuses. The perinatal complications associated with multiple pregnancy include preterm birth, maternal hypertension and congenital anomalies. Fetal non-immune hydrops occurs in the infection of pregnant clients with parvovirus. Post-term birth, maternal hypotension and fetal non-immune hydrops are not seen as complications of multiple pregnancy.

A pregnant client with severe pre-eclampsia has developed the HELLP syndrome. In addition to the observations necessary for pre-eclampsia, what other nursing intervention is critical for this patient? a) Administration of a tocolytic, if prescribed b) Observation for bleeding c) Monitoring for infection d) Maintaining a patent airway

b) Observation for bleeding Rationale: Because of the low platelet count associated with this condition, women with the HELLP syndrome need extremely close observation for bleeding, in addition to the observations necessary for pre-eclampsia. Maintaining a patent airway is a critical intervention needed for a patient with eclampsia while she is having a seizure. Administration of a tocolytic would be appropriate for halting labor. Monitoring for infection is not a priority intervention in this situation.

A client in her 38th week of gestation is admitted into the labor and birth unit with painless bleeding from the vagina. The client is diagnosed with placenta previa. When reviewing the client's history, which of the following would the nurse identify as a risk factor for placenta previa? a) Maternal age more than 30 years b) Previous cesarean birth c) Primigravida status d) Living in coastal areas

b) Previous cesarean birth Rationale: A previous cesarean birth is a risk factor for developing placenta previa. This is due to the damage caused to the endometrial tissue. Multiparity, and not a primigravida status, predisposes to placenta previa. Maternal age over 35 years, and not just more than 30 years, is considered another risk factor. Placenta previa is more common among those living in high altitudes, and not among those living in coastal areas.

You are caring for a patient with preeclampsia. You know that you need to auscultate this patient's lung sounds every two hours. Why would you do this? a) Pulmonary emboli b) Pulmonary edema c) Pulmonary atelectasis d) Pulmonary hypertension

b) Pulmonary edema Rationale: In the hospital, monitor blood pressure at least every four hours for mild preeclampsia and more frequently for severe disease. In addition, it is important to auscultate the lungs every two hours. Adventitious sounds may indicate, developing pulmonary edema.

A woman who is 31 weeks pregnant presents at the emergency room with bright red vaginal bleeding. She says the onset of the bleeding was sudden and she has no pain. The nurse is most likely to assist the physician or technician with which exam? a) A blood transfusion b) An abdominal ultrasound c) A transvaginal ultrasound d) A digital cervical exam

c) A transvaginal ultrasound Rationale: The use of a transvaginal ultrasound is the diagnostic test of choice; it is 100% accurate in prediction of placenta previa, while abdominal ultrasound is only 95% accurate. A digital cervical exam is contraindicated in this patient and the scenario described does not indicate the need for a blood transfusion.

A pregnant woman is diagnosed with abruptio placentae. When reviewing the woman's medical record, which of the following would the nurse expect to find? a) Bright red vaginal bleeding b) Fetal heart rate within normal range c) Firm, rigid uterus on palpation d) Absence of pain

c) Firm, rigid uterus on palpation Rationale: The uterus is firm to rigid to the touch with abruptio placentae; it is soft and relaxed with placenta previa. Bleeding associated with abruptio placentae occurs suddenly and is usually dark in color. Bleeding also may not be visible. Bright red vaginal bleeding is associated with placenta previa. Fetal distress or absent fetal heart rate may be noted with abruptio placentae. The woman with abruptio placentae usually experiences constant uterine tenderness on palpation.

A woman is admitted with a diagnosis of ectopic pregnancy. For which of the following would you anticipate beginning preparation? a) Bed rest for the next 4 weeks. b) Intravenous administration of a tocolytic. c) Immediate surgery. d) Internal uterine monitoring.

c) Immediate surgery. Rationale: Ectopic pregnancy means an embryo has implanted outside the uterus, usually in the fallopian tube. Surgery is usually necessary to remove the growing structure before the tube ruptures or repair the tube if rupture has already occurred. Bed rest will not correct the problem of an ectopic pregnancy. Administering a tocolytic is not indicated, nor is internal uterine monitoring. This makes options A, B, and D incorrect

A woman in labor has sharp fundal pain accompanied by slight vaginal bleeding. Which of the following would be the most likely cause of these symptoms? a) Possible fetal death or injury. b) Placenta previa obstructing the cervix. c) Premature separation of the placenta. d) Preterm labor that was undiagnosed.

c) Premature separation of the placenta. Rationale: Premature separation of the placenta begins with sharp fundal pain, usually followed by vaginal bleeding. Placenta previa usually produces painless bleeding; Preterm labor contractions are more often described as cramping. Possible fetal death or injury does not present with sharp fundal pain. It is usually painless.

Rhogam (Rh immune globulin) will be ordered for an RH - mother undergoing which of the following tests? a) Biophysical profile b) Contraction test c) Non-stress test d) Amniocentesis

d) Amniocentesis Rationale: Amniocentesis is a procedure requiring a needle to enter into the amniotic sac. There is a risk of mixing of the fetal and maternal blood which could result in blood incompatibility. A contraction test, a non-stress test, and biophysical profile are not invasive, so there would be no indication for Rhogam to be administered.

A 28-year-old woman presents in the emergency room with severe abdominal pain. She has not had a normal period for 2 months but she reports that that is not abnormal for her. She has a history of endometriosis. What might the nurse suggest to the physician as a possible cause of the patient's abdominal pain? a) Healthy pregnancy b) Placenta previa c) Molar pregnancy d) Ectopic pregnancy

d) Ectopic pregnancy Rationale: Ectopic pregnancy can present with severe unilateral abdominal pain. Given the history of the client , the amount of pain, the possibility of ectopic pregnancy needs to be considered. A healthy pregnancy would not present with severe abdominal pain unless the patient were term and she was in labor. With a molar pregnancy the woman typically presents between 8 to 16 weeks' gestation with complaints of painless (usually) brown to bright red vaginal bleeding. Placenta previa typically presents with painless, bright red bleeding that begins with no warning.

A woman is being admitted to your hospital unit for severe preeclampsia. When deciding on where to place her, which of the following areas would be most appropriate? a) Near the elevator so she can be transported quickly b) Near the nurse's station so she can be observed closely c) By the nursery so she can maintain hope she will have a child d) In the back hallway where there is a quiet, private room

d) In the back hallway where there is a quiet, private room Rationale: A sudden noise can trigger a seizure in a severely preeclamptic woman. Room placement, therefore, should not be near noise, such as the nursery, the elevator, or nurse's station.

A patient is admitted at 22 weeks gestation with advanced cervical dilatation to 5 centimeters, cervical insufficiency, and a visible amniotic sac at the cervical opening. What is the primary goal for this patient at this point? a) Notification of social support for loss of pregnancy b) Education on causes of cervical insufficiency for the future c) Bed rest to maintain pregnancy as long as possible d) Deliver vaginally

ed rest to maintain pregnancy as long as possible Correct Explanation: At 22 weeks gestation, the fetus is not viable. The woman would be placed on bed rest, total, with every attempt made to halt any further progression of dilatation as long as possible. You would not want to deliver this fetus vaginally at this stage of gestation. It is not your responsibility to notify the patient's social support of a possible loss of the pregnancy. It is not appropriate at this time to educate the mother on causes of cervical insufficiency for future pregnancies.

A client has come to the office for a prenatal visit during her 22nd week of gestation. On examination, it is noted that her blood pressure has increased to 138/90 mm Hg. Her urine is negative for proteinuria. The nurse recognizes which factor as the potential cause? gestational hypertension chronic hypertension HELLP preeclampsia

gestational hypertension

What would be the physiologic basis for a placenta previa?

low placental implantation Explanation: The cause of placenta previa is usually unknown, but for some reason the placenta is implanted low instead of high on the uterus.

A pregnant woman has arrived to the office reporting vaginal bleeding. Which finding during the assessment would lead the nurse to suspect an inevitable abortion? strong abdominal cramping slight vaginal bleeding closed cervical os no passage of fetal tissue

strong abdominal cramping

A woman at 8 weeks' gestation is admitted for ectopic pregnancy. She is asking why this has occurred. The nurse knows that which factor is a known risk factor for ectopic pregnancy? high number of pregnancies multiple gestation pregnancy use of oral contraceptives use of IUD for contraception

use of IUD for contraception

Ectopic Preganancy

Pregnancy in which the fertilized ovum implants outside the uterine cavity

First

The most common cause for _____ trimester abortions is fetal genetic abnormalities, usually unrelated to the mother

when does placenta previa occur?

later weeks of gestation

characterized by hypertension without proteinuria after 20 weeks of gestation resolving by 12 weeks postpartum

gestational hypertension

Rh (D) immunoglobulin is used to

suppress the immune response and prevent isoimmunization.

A woman of 16 weeks' gestation telephones the nurse because she has passed some "berry-like" blood clots and now has continued dark brown vaginal bleeding. Which action would the nurse instruct the woman to do? "Come to the health facility with any vaginal material passed." "Continue normal activity, but take the pulse every hour." "Maintain bed rest, and count the number of perineal pads used." "Come to the health care facility if uterine contractions begin."

"Come to the health facility with any vaginal material passed."

A woman of 16 weeks' gestation telephones the nurse because she has passed some "berry-like" blood clots and now has continued dark brown vaginal bleeding. Which action would the nurse instruct the woman to do?

"Come to the health facility with any vaginal material passed." This is a typical time in pregnancy for gestational trophoblastic disease to present. Asking the woman to bring any material passed vaginally would be important so it can be assessed for this.

A pregnant client at 32 weeks' gestation calls the clinic and informs the nurse that she thinks her membranes are leaking. She states that some clear fluid has run down her leg. What is the best response by the nurse?

"It is best for you to visit a hospital immediately. They can use a nitrazine strip to determine if it is amniotic fluid.

After a regular prenatal visit, a pregnant client asks the nurse to describe the differences between abruptio placentae and placenta previa. Which statement should the nurse include in the teaching?

"Placenta previa causes painless, bright red bleeding during pregnancy due to an abnormally implanted placenta that is too close to or covers the cervix; abruptio placenta is associated with dark red painful bleeding caused by premature separation of the placenta from the wall of the uterus before the end of labor."

After a regular prenatal visit, a pregnant client asks the nurse to describe the differences between abruptio placentae and placenta previa. Which statement should the nurse include in the teaching?

"Placenta previa causes painless, bright red bleeding during pregnancy due to an abnormally implanted placenta that is too close to or covers the cervix; abruptio placentae is associated with dark red painful bleeding caused by premature separation of the placenta from the wall of the uterus before the end of labor."

A client with a history of cervical insufficiency is seen for reports of pink-tinged discharge and pelvic pressure. The primary care provider decides to perform a cervical cerclage. The nurse teaches the client about the procedure. Which client response indicates that the teaching has been effective? "Staples are put in the cervix to prevent it from dilating." "The cervix is glued shut so no amniotic fluid can escape." "Purse-string sutures are placed in the cervix to prevent it from dilating." "A cervical cap is placed so no amniotic fluid can escape."

"Purse-string sutures are placed in the cervix to prevent it from dilating."

A client in her first trimester arrives at the emergency room with reports of severe cramping and vaginal spotting. On examination, the physician informs her that no fetal heart sounds are evident and the fetus is no longer viable. He says that he must perform a dilatation and curettage to ensure that all of the products of conception are removed. The client looks frightened and confused and says that she does not believe in abortion. Which of the following should the nurse say to the client? a) "The pregnancy is already lost; the procedure is simply to clean out the uterus to prevent further complications." b) "The pregnancy must be terminated to protect your health and life." c) "You have experienced an incomplete miscarriage. This procedure is simply to remove the membranes and placenta that are retained in the uterus." d) "The choice is up to you, but the physician is recommending an abortion."

...

grade 1 (mild)

? minimal bleeding (less than 500 mL), 10% to 20% separation, tender uterus, no coagulopathy, signs of shock or fetal distress;

c. "Purse-string sutures are placed in the cervix to prevent it from dilating."

A client with a history of cervical insufficiency is seen for reports of pink-tinged discharge and pelvic pressure. The primary care provider decides to perform a cervical cerclage. The nurse teaches the client about the procedure. Which client response indicates that the teaching has been effective? a. "Staples are put in the cervix to prevent it from dilating." b. "A cervical cap is placed so no amniotic fluid can escape." c. "Purse-string sutures are placed in the cervix to prevent it from dilating." d. "The cervix is glued shut so no amniotic fluid can escape."

b. Fetal distress related to hypoxia

A nurse is monitoring a client with PROM who is in labor and observes meconium in the amniotic fluid. What does this indicate? a. Cord compression b. Fetal distress related to hypoxia c. Infection d. Central nervous system "CNS" involvement

b. Dyspepsia d. Hypotension e. Tachycardia

A nurse is monitoring a client with spontaneous abortion who has been prescribed misoprostol. Which symptoms are common adverse effects associated with misoprostol? (Select all that apply.) a. Constipation b. Dyspepsia c. Headache d. Hypotension e. Tachycardia

a. hydatidiform mole.

A nurse is providing care to a client who has been diagnosed with a common benign form of gestational trophoblastic disease. The nurse identifies this as: a. hydatidiform mole. b. ectopic pregnancy. c. placenta accrete. d. hydramnios.

a. strong abdominal cramping

A pregnant woman has arrived to the office reporting vaginal bleeding. Which finding during the assessment would lead the nurse to suspect an inevitable abortion? a. strong abdominal cramping b. no passage of fetal tissue c. slight vaginal bleeding d. closed cervical os

a. use of IUD for contraception

A woman at 8 weeks' gestation is admitted for ectopic pregnancy. She is asking why this has occurred. The nurse knows that which factor is a known risk factor for ectopic pregnancy? a. use of IUD for contraception b. use of oral contraceptives c. multiple gestation pregnancy d. high number of pregnancies

d. Chromosomal defects in the fetus

A woman in her 20s has experienced a miscarriage at 10 weeks' gestation and asks the nurse at the hospital what went wrong. She is concerned that she did something that caused her to lose her baby. The nurse can reassure the woman by explaining that the most common cause of spontaneous miscarriage in the first trimester is related to which factor? a. Advanced maternal age b. Faulty implantation c. Exposure to chemicals or radiation d. Chromosomal defects in the fetus

After an examination, a client has been determined to have an unruptured ectopic pregnancy. Which medication would the nurse anticipate being prescribed?

methotrexate

A client with preeclampsia is receiving magnesium sulfate to suppress or control seizures. Which nursing intervention should a nurse perform to determine the effectiveness of therapy? A. Assess deep tendon reflexes B. Monitor intake and output C. Assess the client's mucous membrane D. Assess client's skin turgor

A. Assess deep tendon reflexes

A nurse is caring for a client with CVD who has just delivered. What nursing interventions should the nurse perform when caring for this client? SATA A. Assess for shortness of breath B. Assess for a moist cough C. Assess for edema and note any pitting D. Auscultate heart sounds for abnormalities E. Monitor the client's hemoglobin and hematocrit

A. Assess for shortness of breath C. Assess for edema and note any pitting D. Auscultate the heart sounds for abnormalities

When caring for a client with premature rupture of membranes (PROM), the nurse observes an increase in the client's pulse. What should the nurse do next? A. Assess the client's temperature B. Monitor the client for preterm labor C. Assess for cord compression D. Monitor the fetus for respiratory distress

A. Assess the client's temperature

What important instruction should the nurse give a pregnant client with tuberculosis? A. Maintain adequate hydration B. Avoid direct sunlight C. Avoid red meat D. Wear light, cotton clothes

A. Maintain adequate hydration

During the assessment of a laboring client, the nurse learns the client has cardiovascular disease (CVD). Which assessment would be priority for the newborn? A. Respiratory function B. Heart rate C. Temperature D. Urine output

A. Respiratory function

A nurse is caring for a newborn with fetal alcohol spectrum disorder. What characteristic of the fetal alcohol spectrum disorder should the nurse assess for in the newborn? A. Small head circumference B. Decreased blood glucose level C. Abnormal breathing pattern D. Wide eyes

A. Small head circumference

The nurse is caring for a pregnant client who is in her 30th week of gestation and has genital heart disease. Which should the nurse recognize as a symptom of cardiac decompensation with this client? A. Swelling of the face B. Dry, rasping cough C. Slow, labored respiration D. Elevated temperature

A. Swelling of the face

A nurse in the maternity triage unit is caring for a client with a suspected ectopic pregnancy. Which nursing intervention should the nurse perform first? Administer oxygen to the client. Provide emotional support to the client and significant other. Assess the client's vital signs. Obtain a surgical consent from the client.

Assess the client's vital signs.

Which medication would the nurse prepare to administer if prescribed as treatment for an unruptured ectopic pregnancy?

A: Methotrexate Methotrexate, a folic acid antagonist that inhibits cell division in the developing embryo, is most commonly used to treat ectopic pregnancy. Oxytocin is used to stimulate uterine contractions and would be inappropriate for use with an ectopic pregnancy. Promethazine and ondansetron are antiemetics that may be used to treat hyperemesis gravidarum

Rhogam (Rh immune globulin) will be ordered for an RH - mother undergoing which of the following tests? a) Biophysical profile b) Contraction test c) Non-stress test d) Amniocentesis

Amniocentesis Correct Explanation: Amniocentesis is a procedure requiring a needle to enter into the amniotic sac. There is a risk of mixing of the fetal and maternal blood which could result in blood incompatibility. A contraction test, a non-stress test, and biophysical profile are not invasive, so there would be no indication for Rhogam to be administered.

A nurse is caring for a pregnant client. The initial interview reveals that the client is accustomed to drinking coffee at regular intervals. For which increased risk should the nurse make the client aware? A. Heart Disease B. Anemia C. Rickets D. Scurvy

B. Anemia

A nurse is caring for a pregnant client with asthma. Which intervention would the nurse perform first? A. Monitoring temperature frequently B. Assessing oxygen saturation C. Monitoring frequency of headache D. Assessing for feeling nauseated

B. Assessing oxygen saturation

A client is seeking advise for his pregnant wife who is experiencing mild elevations in blood pressure. In which positions should a nurse recommend the pregnant client rest? A. Supine position B. Lateral recumbent position C. Left lateral lying position D. Head of the bed slightly elevated

B. Lateral recumbent position

What is the role of the nurse during the preconception counseling of a pregnant client with chronic hypertension? A. Stressing the avoidance of dairy products B. Stressing the positive benefits of a healthy lifestyle C. Stressing the increased use of Vitamin D supplements D. Stressing regular walks and exercise

B. Stressing the positive benefits of a healthy lifestyle

The nurse recognizes that documenting accurate blood pressures is vital in the diagnosing of preeclampsia and eclampsia. The nurse suspects preeclampsia based on which finding?

BP of 140/90 mm Hg on two occasions 6 hours apart

A nurse has been assigned to assess a pregnant client for abruptio placenta. For which classic manifestation of this condition should the nurse assess? A. Painless bright red vaginal bleeding B. Increased fetal movement C. "Knife-like" abdominal pain with vaginal bleeding D. Generalized vasospasm

C. "Knife-like" abdominal pain with vaginal bleeding

A 28-year-old client with a history of endometriosis presents to the emergency department with severe abdominal pain and nausea and vomiting. The client also reports her periods are irregular with the last one being 2 months ago. The nurse prepares to assess for which possible cause for this client's complaints? Placenta previa Ectopic pregnancy Molar pregnancy Healthy pregnancy

Ectopic pregnancy

A nurse is caring for a pregnant client with gestational diabetes. Which meal should the nurse recommend for this client? A. Baked chicken, green beans, and chocolate cake B. Pizza, corn, and orange slices C. Baked turkey, brown rice, and strawberries D. Steak, baked potato with butter, and ice cream

C. Baked turkey, brown rice, and strawberries

A nurse is caring for a pregnant client with eclamptic seizure. Which is a characteristic of eclampsia? A. Muscle rigidity is followed by facial twitching B. Respirations are rapid during the seizure C. Coma occurs after the seizure D. Respiration fails after the seizure

C. Coma occurs after the seizure

Gestational Hypertension

Characterized by hypertension without proteinuria after 20 weeks' gestation and a return of the blood pressure to normal postpartum

The nurse is comforting and listening to a young couple who just suffered a miscarriage. When asked why this happened, which reason should the nurse share as a common cause? Maternal smoking Lack of prenatal care The age of the mother Chromosomal abnormality

Chromosomal abnormality

A nurse is assessing pregnant clients for the risk of placenta previa. Which of the following clients faces the greatest risk for this condition? A. A 23-year-old multigravida client B. A client with a history of alcohol abuse C. A client with a structurally defective cervix D. A client who had a myomectomy to remove fibroids

D. A client who has a myomectomy to remove fibroids

A nurse is caring for a client with hyperemesis gravidarum. Which nursing action is the priority for this client? A. Administer total parental nutrition B. Administer an antiemetic C. Set up for a percutaneous endoscopic gastrostomy D. Administer IV NS with vitamins and electrolytes

D. Administer IV NS with vitamins and electrolytes

A nursing is caring for a pregnant client with heart disease in a labor unit. Which intervention is most important in the first 48 hours postpartum? A. Limiting sodium intake B. Inspecting the extremities for edema C. Ensuring that the client consumes a high-fiber diet D. Assessing for cardiac decompensation

D. Assessing for cardiac decompensation

The nurse is required to assess a pregnant client who is reporting vaginal bleeding. Which nurse action is the priority? A. Monitoring uterine contractility B. Assessing signs of shock C. Determining the amount of funneling D. Assessing the amount and color of the bleeding

D. Assessing the amount and color of the bleeding

A 28-year-old client with a history of endometriosis presents to the emergency department with severe abdominal pain and nausea and vomiting. The client also reports her periods are irregular with the last one being 2 months ago. The nurse prepares to assess for which possible cause for this client's complaints? Placenta previa Healthy pregnancy Ectopic pregnancy Molar pregnancy

Ectopic pregnancy

A nurse is caring for a young woman who is in her 10th week of gestation. She comes into the clinic reporting vaginal bleeding. Which assessment finding best correlates with a diagnosis of hydatidiform mole?

Dark red, "clumpy" vaginal discharge

Reducing physical activity, observing for signs of infection, and giving corticosteroids may be used for the woman with

PROM when the fetal lungs are immature.

A woman in labor suddenly reports sharp fundal pain accompanied by slight dark red vaginal bleeding. The nurse should prepare to assist with which situation?

Premature separation of the placenta

A woman with an incomplete abortion is to receive misoprostol. The woman asks the nurse, "Why am I getting this drug?" The nurse responds to the client, integrating understanding that this drug achieves which effect?

ensures passage of all the products of conception

A nurse is conducting a presentation for a group of pregnant women about conditions that can occur during pregnancy and that place the woman at highrisk. When discussing blood incompatibilities, which measure would the nurse explain as most effective in preventing isoimmunization during pregnancy?

RHoGAM administration to Rh-negative women

A nurse is conducting a presentation for a group of pregnant women about conditions that can occur during pregnancy and that place the woman at high-risk. When discussing blood incompatibilities, which measure would the nurse explain as most effective in preventing isoimmunization during pregnancy? cerclage blood typing of mothers with type A or B blood Rho(D) immune globulin administration to Rh-negative women amniocentesis

Rho(D) immune globulin administration to Rh-negative women

A nurse is working as part of a community group focusing efforts on preventing isoimmunization during pregnancy at the local women's health clinic. Which measure would the group encourage?

RhoGAM administration to Rh-negative women

A 44-year-old client has lost several pregnancies over the last 10 years. For the past 3 months, she has had fatigue, nausea, and vomiting. She visits the clinic and takes a pregnancy test; the results are positive. Physical examination confirms a uterus enlarged to 13 weeks' gestation; fetal heart tones are heard. Ultrasound reveals that the client is experiencing some bleeding. Considering the client's prenatal history and age, what does the nurse recognize as the greatest risk for the client at this time? premature birth hypertension spontaneous abortion (miscarriage) preterm labor

Spontaneous abortion (miscarriage) Explanation: The client's advanced maternal age (pregnancy in a woman 35 years or older) increases her risk for spontaneous abortion (miscarriage). Hypertension, preterm labor, and prematurity are risks as this pregnancy continues. Her greatest risk at 13 weeks' gestation is losing this pregnancy.

clonus

The presence of rhythmic involuntary contractions, most often at the foot or ankle.

Vaginal bleeding during pregnancy is always a deviation from the normal.

True Vaginal bleeding during pregnancy is always a deviation from the normal, is always potentially serious, may occur at any point during pregnancy, and is always frightening. It must always be carefully investigated because it can impair both the outcome of the pregnancy and the woman's life or health.

a. methotrexate

Which medication would the nurse prepare to administer if prescribed as treatment for an unruptured ectopic pregnancy? a. methotrexate b. promethazine c. oxytocin d. ondansetron

Vaginal bleeding during pregnancy is always a deviation from the normal. a) True b) False

a) True Rationale: Vaginal bleeding during pregnancy is always a deviation from the normal, is always potentially serious, may occur at any point during pregnancy, and is always frightening. It must always be carefully investigated because it can impair both the outcome of the pregnancy and the woman's life or health.

A young mother delivers twin boys who shared the same placenta. What serious complication are they at risk for? a) TORCH syndrome b) ABO incompatability c) Twin-to-twin transfusion syndrome (TTTS) d) HELLP syndrome

c) Twin-to-twin transfusion syndrome (TTTS) Rationale: When twins share a placenta, a serious condition called twin-to-twin transfusion syndrome (TTTS) can occur.

A pregnant client at 20 weeks' gestation arrives at the health care facility reporting excessive vaginal bleeding and no fetal movements. Which assessment finding would the nurse anticipate in this situation? congenital malformations placenta previa ectopic pregnancy cervical insufficiency

cervical insufficiency

severe preclampsia

characterized by a blood pressure of 160/110 mm Hg.

Gestational trophoblastic disease

comprises a spectrum of neoplastic disorders that originate in the placenta.

The three most common causes of hemorrhage during the first half of pregnancy are

e spontaneous abortion, ectopic pregnancy, and GTD

A 28-year-old woman presents in the emergency department with severe abdominal pain. She has not had a normal period for 2 months, but she reports that that is not abnormal for her. She has a history of endometriosis. What might the nurse suggest to the primary care provider as a possible cause of the client's abdominal pain?

ectopic pregnancy Ectopic pregnancy can present with severe unilateral abdominal pain. Given the history of the client and the amount of pain, the possibility of ectopic pregnancy needs to be considered. A healthy pregnancy would not present with severe abdominal pain unless the client were term and she was in labor. With a molar pregnancy the woman typically presents between 8 to 16 weeks' gestation reporting painless (usually) brown to bright red vaginal bleeding. Placenta previa typically presents with painless, bright red bleeding that begins with no warning.

A client has come to the office for a prenatal visit during her 22nd week of gestation. On examination, it is noted that her blood pressure has increased to 138/90 mm Hg. Her urine is negative for proteinuria. The nurse recognizes which factor as the potential cause?

gestational hypertension

A client in her 20th week of gestation develops HELLP syndrome. What are features of HELLP syndrome? Select all that apply. -hyperthermia -hemolysis -elevated liver enzymes -leukocytosis -low platelet count

hemolysis elevated liver enzymes low platelet count The HELLP syndrome is a syndrome involving hemolysis (microangiopathic hemolytic anemia), elevated liver enzymes, and a low platelet count. Hyperthermia and leukocytosis are not features of HELLP syndrome.

A client in her 20th week of gestation develops HELLP syndrome. What are features of HELLP syndrome? Select all that apply.

hemolysis elevated liver enzymes low platelet count The HELLP syndrome is a syndrome involving hemolysis (microangiopathic hemolytic anemia), elevated liver enzymes, and a low platelet count. Hyperthermia and leukocytosis are not features of HELLP syndrome.

A nurse is providing care to a client who has been diagnosed with a common benign form of gestational trophoblastic disease. The nurse identifies this as:

hydatidiform mole. Explanation: Gestational trophoblastic disease comprises a spectrum of neoplastic disorders that originate in the placenta. The two most common types are hydatidiform mole (partial or complete) and choriocarcinoma. Hydatidiform mole is a benign neoplasm of the chorion in which the chorionic villi degenerate and become transparent vesicles containing clear, viscid fluid. Ectopic pregnancy, placenta accreta, and hydramnios fall into different categories of potential pregnancy complications.

What would be the physiologic basis for a placenta previa?

low placental implantation

Which medication would the nurse prepare to administer if prescribed as treatment for an unruptured ectopic pregnancy?

methotrexate Methotrexate, a folic acid antagonist that inhibits cell division in the developing embryo, is most commonly used to treat ectopic pregnancy. Oxytocin is used to stimulate uterine contractions and would be inappropriate for use with an ectopic pregnancy. Promethazine and ondansetron are antiemetics that may be used to treat hyperemesis gravidarum.

A woman in labor has sharp fundal pain accompanied by slight vaginal bleeding. What is the most likely cause of these symptoms?

premature separation of the placenta Premature separation of the placenta begins with sharp fundal pain, usually followed by vaginal bleeding. Placenta previa usually produces painless bleeding. Preterm labor contractions are more often described as cramping. Possible fetal death or injury does not present with sharp fundal pain. It is usually painless.

A woman in labor is at risk for abruptio placentae. Which assessment would most likely lead the nurse to suspect that this has happened?

sharp fundal pain and discomfort between contractions

A 44-year-old client has lost several pregnancies over the last 10 years. For the past 3 months, she has had fatigue, nausea, and vomiting. She visits the clinic and takes a pregnancy test; the results are positive. Physical examination confirms a uterus enlarged to 13 weeks' gestation; fetal heart tones are heard. Ultrasound reveals that the client is experiencing some bleeding. Considering the client's prenatal history and age, what does the nurse recognize as the greatest risk for the client at this time? spontaneous abortion (miscarriage) hypertension premature birth preterm labor

spontaneous abortion (miscarriage)

Infection

A foul odor of amniotic fluid indicates ______

d. Premature separation of the placenta

A woman in labor suddenly reports sharp fundal pain accompanied by slight dark red vaginal bleeding. The nurse should prepare to assist with which situation? a. Possible fetal death or injury b. Preterm labor that was undiagnosed c. Placenta previa obstructing the cervix d. Premature separation of the placenta

A client has been admitted to the hospital with a diagnosis of severe preeclampsia. Which nursing intervention is the priority?

Confine the client to bed rest in a darkened room. With severe preeclampsia, most women are hospitalized so that bed rest can be enforced and a woman can be observed more closely than she can be on home care. The nurse should darken the room if possible because a bright light can also trigger seizures. The other interventions listed pertain to a client who has experienced a seizure and has thus progressed to eclampsia.

Spontaneous miscarriage occurs in 5% to 10% of all pregnancies.

False Spontaneous miscarriage occurs in 15% to 20% of all pregnancies and arises from natural causes.

A nurse is caring for a client undergoing treatment for ectopic pregnancy. Which symptom is observed in a client if rupture or hemorrhaging occurs before the ectopic pregnancy is successfully treated?

Phrenic nerve irritation

The nurse is caring for a woman at 32 weeks gestation with severe preeclampsia. Which assessment finding should the nurse prioritize after the administration of hydralazine to this client? Gastrointestinal bleeding Halos around lights Tachycardia Sweating

Tachycardia

Which statement is false regarding bathing the newborn? -To reduce the risk of heat loss, the bath should be performed by the nurse, not the parents, within 2 to 4 hours of birth. -Bathing should not be done until the newborn is thermally stable. -While bathing the newborn, the nurse should wear gloves. -Mild soap should be used on the body and hair but not on the face.

To reduce the risk of heat loss, the bath should be performed by the nurse, not the parents, within 2 to 4 hours of birth. Bathing the newborn is not necessary for thermal stability. It can be postponed until the parents are able to do it.

Fetal distress or absent fetal heart rate may be noted,usually experiences constant uterine tenderness on palpation

abruptio placenta

A pregnant woman is admitted to the hospital with a diagnosis of placenta previa. Which action would be the priority for this woman on admission?

assessing fetal heart tones by use of an external monitor

preeclampsia vision symptoms

blurred vision and blind spots

A woman with an incomplete abortion is to receive misoprostol. The woman asks the nurse, "Why am I getting this drug?" The nurse responds to the client, integrating understanding that this drug achieves which effect?

ensures passage of all the products of conception Misoprostol is used to stimulate uterine contractions and evacuate the uterus after an abortion to ensure passage of all the products of conception. Rho(D) immune globulin is used to suppress the immune response and prevent isoimmunization.

A novice nurse asks to be assigned to the least complex antepartum client. Which condition would necessitate the least complex care requirements? abruptio placenta preecalmpsia placenta previa gestational hypertension

gestational hypertension

A client with type 1 diabetes has received diet instruction as part of the treatment plan. The nurse determines a need for additional instruction when the client makes which of the following statements?

"I may consume alcohol without food" People with diabetes who choose to drink should limit their intake to one drink per day or less for women and two drinks per day or less for men. Alcohol should be consumed with food to reduce the risk of hypoglycemia.

A client in her 20th week of gestation develops HELLP syndrome. What are features of HELLP syndrome? Select all that apply.

-hemolysis -elevated liver enzymes -low platelet count

Which of the following would the nurse most likely include when planning the care for a woman requiring hospitalization for hyperemesis gravidarum? Select all that apply. a) Preparing the woman for insertion of a feeding tube b) Obtaining baseline blood electrolyte levels c) Administering antiemetic agents d) Monitoring intake and output e) Maintaining NPO status for the first day or two

Maintaining NPO status for the first day or two • Administering antiemetic agents • Obtaining baseline blood electrolyte levels • Monitoring intake and output Explanation: When hospitalization is necessary, oral food and fluids are withheld to allow the gut to rest. Antiemetic agents are ordered to help control nausea and vomiting. The woman is likely to be dehydrated, so the nurse would obtain baseline blood electrolyte levels and administer intravenous fluid and electrolyte replacement therapy as indicated. Once the nausea and vomiting subside, oral food and fluids are gradually reintroduced. Total parenteral nutrition or a feeding tube is used to prevent malnutrition only if the client does not improve with these interventions.

A pregnant patient is developing HELLP syndrome. During labor, which order should the nurse question?

Prepare for epidural anesthesia. In the HELLP syndrome, patients develop low platelet counts. With a low platelet count, injections such as epidural anesthesia are contraindicated. This is the order that the nurse should question. The patient's urine output should be assessed every hour because renal failure is a complication of this syndrome. Positioning on the left side during labor will help blood flow to the uterus. Assessing blood pressure every 15 minutes is appropriate for the patient with this syndrome.

A woman develops HELLP syndrome. During labor, which of the following orders would you question? a) Assess her blood pressure every 15 minutes. b) Prepare her for epidural anesthesia. c) Assess the urine output every hour. d) Urge her to lie on her left side during labor.

Prepare her for epidural anesthesia. Explanation: A consequence of the HELLP syndrome is poor blood coagulation. Epidural anesthesia is not recommended when blood coagulation is in doubt.

A patient is admitted to labor and delivery for management of severe preeclampsia. An IV infusion of magnesium sulfate is started. What is the primary goal for magnesium sulfate therapy? a) Decrease blood pressure b) Prevent maternal seizures c) Reverse edema d) Decrease protein in urine

Prevent maternal seizures Correct Explanation: The primary therapy goal for any preeclamptic patient is to prevent maternal seizures. Use of magnesium sulfate is the drug therapy of choice for severe preeclampsia and is only used to manage and attempt to prevent progression to eclampsia. Magnesium sulfate therapy does not have as its primary goal a decrease in blood pressure, a decrease in protein in the urine, nor the reversal of edema

A client at 37 weeks' gestation presents to the emergency department with a BP 150/108 mm Hg, 1+ pedal edema, 1+ proteinuria, and normal deep tendon reflexes. Which assessment should the nurse prioritize as the client is administered magnesium sulfate IV?

Respiratory rate

The nurse is monitoring a pregnant patient who is receiving intravenous magnesium sulfate for eclampsia. During the last assessment, the nurse was unable to elicit a patellar reflex. What should the nurse do?

Stop the current infusion.

A young mother gives birth to twin boys who shared the same placenta. What serious complication are they at risk for?

Twin-to-twin transfusion syndrome (TTTS) When twins share a placenta, a serious condition called twin-to-twin transfusion syndrome (TTTS) can occur.

Cervical Insufficiency (Incompetent Cervix)

Weak, structurally defective cervix that spontaneously dilates in the absence of contractions in the second trimester, resulting in the loss of the pregnancy

c. low placental implantation

What would be the physiologic basis for a placenta previa? a. a loose placental implantation b. a uterus with a midseptum c. low placental implantation d. a placenta with multiple lobes

A nurse is conducting a refresher program for a group of perinatal nurses. Part of the program involves a discussion of HELLP. The nurse determines that the group needs additional teaching when they identify which aspect as a part of HELLP?

elevated lipoproteins Explanation: The acronym HELLP represents hemolysis, elevated liver enzymes, and low platelets. This syndrome is a variant of preeclampsia/eclampsia syndrome that occurs in 10% to 20% of clients whose diseases are labeled as severe

A nurse suspects that a client is developing HELLP syndrome. The nurse notifies the health care provider based on which finding? disseminated intravascular coagulation (DIC) hyperglycemia elevated liver enzymes elevated platelet count

elevated liver enzymes

A woman with an incomplete abortion is to receive misoprostol. The woman asks the nurse, "Why am I getting this drug?" The nurse responds to the client, integrating understanding that this drug achieves which effect? -ensures passage of all the products of conception -alleviates strong uterine cramping -suppresses the immune response to prevent isoimmunization -halts the progression of the abortion

ensures passage of all the products of conception Misoprostol is used to stimulate uterine contractions and evacuate the uterus after an abortion to ensure passage of all the products of conception. Rho(D) immune globulin is used to suppress the immune response and prevent isoimmunization. Page 664

A nurse is assigned to care for a client with diabetes who has a diet high in saturated fats. What information should the nurse provide to the client about the detrimental effect of a high-fat diet?

it increases the risk of diabetic complications Foods rich in saturated fat have been proven clinically to raise levels of bad cholesterol in the blood. The recommendation based on strong evidence is to limit saturated fat to 7% of total calories.

What would be the physiologic basis for a placenta previa? a uterus with a midseptum a loose placental implantation low placental implantation a placenta with multiple lobes

low placental implantation

What would be the physiologic basis for a placenta previa? a uterus with a midseptum a loose placental implantation a placenta with multiple lobes low placental implantation

low placental implantation

The nurse is caring for a pregnant client with fallopian tube rupture. Which intervention is the priority for this client?

monitor the client's vital signs and bleeding A nurse should closely monitor the client's vital signs and bleeding (peritoneal or vaginal) to identify hypovolemic shock that may occur with tubal rupture. Beta-hCG level is monitored to diagnose an ectopic pregnancy or impending abortion. Monitoring the mass with transvaginal ultrasound and determining the size of the mass are done for diagnosing an ectopic pregnancy. Monitoring the FHR does not help to identify hypovolemic shock.

The nurse is developing a plan of care for a woman who is pregnant with twins. The nurse includes interventions focusing on which area because of the woman's increased risk?

preeclampsia Women with multiple gestations are at high risk for preeclampsia, preterm labor, hydramnios, hyperemesis gravidarum, anemia, and antepartal hemorrhage. There is no association between multiple gestations and the development of chorioamnionitis.

A 20-year-old client arrives at the emergency department complaining of frequent excessive urination, excessive thirst, and an increased appetite. Which disorder will the nurse most likely consider?

type 1 diabetes Frequent excessive urination, excessive thirst, and an increased appetite are classic symptoms of type 1 diabetes.

A client at 11 weeks' gestation experiences pregnancy loss. The client asks the nurse if the bleeding and cramping that occurred during the miscarriage were caused by working long hours in a stressful environment. What is the most appropriate response from the nurse?

"I can understand your need to find an answer to what caused this. Let's talk about this further."

A client with diabetes mellitus has received instructions about sick-day management. Which statement by the client would indicate that the client needs further instructions?

"I should increase the amount of carbohydrate intake" Unless blood glucose levels are higher than 250 mg/dL, the client should eat the usual amount of carbohydrate, dividing it into smaller meals and snacks if necessary.

The nurse is teaching a client who is diagnosed with preeclampsia how to monitor her condition. The nurse determines the client needs more instruction after making which statement?

"If I have changes in my vision I will lie down and rest" Changes in the visual field may indicate the client has moved from preeclampsia to severe preeclampsia and is at risk for developing a seizure due to changes in cerebral blood flow. The client would require immediate assessment and intervention. Gaining weight is not necessarily a sign of worsening preeclampsia. The other choices are instructions which the client may be given to follow.

A 16-year-old client gave birth to a 12 weeks' gestation fetus last week. The client has come to the office for follow-up and while waiting in an examination room notices that on the schedule is written her name and "follow-up of spontaneous abortion." The client is upset about what is written on the schedule. How can the nurse best explain this terminology?

"Spontaneous abortion is a more specific term used to describe a spontaneous miscarriage, which is a loss of pregnancy before 20 weeks. This term does not imply that you did anything to affect the pregnancy."

A client suffering a miscarriage at 12 weeks' gestations is very upset that the health care provider has ordered a D&C. How should the nurse respond after the client states she didn't have a D&C the time she lost a previous baby at 5 weeks' gestation?

"This procedure is needed to adequately remove all the fetal tissue."

A client in her first trimester arrives at the emergency room with reports of severe cramping and vaginal spotting. On examination, the health care provider informs her that no fetal heart sounds are evident and orders a dilatation and curettage. The client looks frightened and confused and states that she does not believe in abortion. Which statement by the nurse is best?

"Unfortunately, the pregnancy is already lost. The procedure is to clear the uterus to prevent further complications."

A student nurse asks the instructor what percentage of clinically recognized pregnancies end in miscarriages during the first trimester. Which response from the nurse is the most accurate? 5% to 10% 15% to 20% 21% to 30% 31% to 40%

15% to 20%

A client newly diagnosed with type 1 diabetes asks the nurse when is the best time to inject regular insulin. What should the nurse respond?

30 minutes before eating For optimum effect, regular insulin should be injected 30 minutes before eating.

A pregnant client is admitted to a health care unit with disseminated intravascular coagulation (DIC). Which prescription is the nurse most likely to receive regarding the therapy for such a client? Administer a ratio of 1 unit of blood to 4 units of frozen plasma. Administer cryoprecipitate and platelets. Aim at keeping the client's hematocrit above 20%. Give each unit of blood to raise the hematocrit by 3 g/dL.

Administer cryoprecipitate and platelets.

C. The woman should avoid noxious stimuli such as strong flavors, odors, or perfumes because they might trigger nausea and vomiting.

After teaching a woman about hyperemesis gravidarum and how it differs from the typical nausea and vomiting of pregnancy, which statement by the woman indicates that the teaching was successful? a. "I can expect the nausea to last through my second trimester." b. "I should drink fluids with my meals instead of in between them." c. "I need to avoid strong odors, perfumes, or flavors." d. "I should lie down after I eat for about 2 hours."

A 28-year-old client and her current partner present for the first antenatal OB appointment. The client has no children but does question a possible miscarriage 2 years ago; however, she never sought medical attention because she felt fine. Labs reveal both client and partner are Rh negative. Which action should the nurse prioritize? Assess client for anti-D antibodies Arrange for an amniocentesis Continue with routine procedures and tasks Perform direct Coombs test

Assess client for anti-D antibodies

A pregnant woman is admitted to the hospital with a diagnosis of placenta previa. Which action would be the priority for this woman on admission?

Assessing fetal heart tones by use of an external monitor

A pregnant client has been diagnosed with gestational diabetes. Which are risk factors for developing gestational diabetes? SATA A. Maternal age less than 18 years B. Genitourinary tract abnormalities C. Obesity D. Hypertension E. Previous large for gestational age (LGA) infant

C. Obesity D. Hypertension E. Previous large for gestational age (LGA) infant

What would be the physiologic basis for a placenta previa? • a loose placental implantation • low placental implantation • a placenta with multiple lobes • a uterus with a midseptum

low placental implantation

A woman has been diagnosed as having pregnancy-induced hypertension. Which of the following is the most typical symptom of this? a) Susceptibility to infection b) Blood pressure elevation c) Weight loss d) Increased perspiration

Blood pressure elevation Explanation: The symptom of hypertension of pregnancy is blood pressure elevation

What special interventions would the nurse implement in a client who is carrying twin fetuses?

Demonstrate to the client how to perform fetal movement counts after 32 weeks.

Current research indicates that supplementation with what before pregnancy may reduce the risk of placental abruption?

Folic acid

A patient is admitted with a diagnosis of ectopic pregnancy. For what should the nurse anticipate preparing the patient?

Immediate surgery An ectopic pregnancy is one in which implantation occurred outside the uterine cavity, usually within the fallopian tube. As the embryo grows, the fallopian tube can rupture. The therapy for ruptured ectopic pregnancy is laparoscopy to ligate the bleeding vessels and to remove or repair the damaged fallopian tube. There is no reason to begin uterine monitoring. The patient does not need to be on bed rest for 4 weeks. A tocolytic is not needed because the patient is not in labor.

Which of the following changes in B/P assessment findings during the second trimester indicate the highest risk for preeclampsia. a) Initial BP 140/85, current BP 130/80. b) Initial BP 110/60, current BP 112/86. c) Initial BP 120/80, current BP 130/88. d) Initial BP 100/70, current BP 140/90.

Initial BP 100/70, current BP 140/90. Explanation: A rise in blood pressure to above 140/90 is a concern the patient may be developing preeclampsia. The B/Ps noted in options A, C, and D are not indicative of developing preeclampsia, so these are incorrect responses.

The nurse is caring for a pregnant client with severe preeclampsia. Which nursing intervention should a nurse perform to institute and maintain seizure precautions in this client?

Keep the suction equipment readily available.

The nurse is caring for a pregnant client with severe preeclampsia. Which nursing intervention should a nurse perform to institute and maintain seizure precautions in this client? Provide a well-lit room. Keep head of bed slightly elevated. Place the client in a supine position. Keep the suction equipment readily available.

Keep the suction equipment readily available.

Spontaneous Abortion

Refers to the loss of a fetus resulting from natural causes- that is not elective or therapeutically induced by a procedure (usually before the 20th week of gestation)

The following hourly assessments are obtained by the nurse on a client with preeclampsia receiving magnesium sulfate: 97.3oF (36.2oC), HR 88, RR 1, BP 148/110 mm Hg. What other priority physical assessments by the nurse should be implemented to assess for potential toxicity? Lung sounds Oxygen saturation Reflexes Magnesium sulfate level

Reflexes

A nurse is conducting a presentation for a group of pregnant women about conditions that can occur during pregnancy and that place the woman at high-risk. When discussing blood incompatibilities, which measure would the nurse explain as most effective in preventing isoimmunization during pregnancy?

Rho(D) immune globulin administration to Rh-negative women Rh incompatibility can be prevented with the use of Rho(D) immune globulin. Hemolysis associated with ABO incompatibility is limited to mothers with type O blood and their fetuses with type A or B blood. Amniocentesis would be appropriate for treatment of polyhydramnios, not isoimmunization. Cerclage is a treatment for cervical insufficiency.

A 25-year-old pregnant client has just been diagnosed with hyperemesis gravidarum. Which instruction should the nurse prioritize during a teaching session?

Take your anti-nausea medicine around the clock. Antiemetics, if prescribed, are usually more effective when given on a regular, around-the-clock schedule versus as-needed (PRN) dosing. Increasing fluid intake may exacerbate nausea. It would be better if the woman did not drink a lot of fluids when she ate. She should continue with a balanced, nutritious diet, consulting a dietitian if needed. Doing her own cooking will not assist with developing a tolerance to the odors but will help her eliminate odors which might trigger her.

b. "Placental abruption is quite painful and I will need to let the doctor know if I begin to have abdominal pain."

The nurse is teaching a prenatal class on potential problems during pregnancy to a group of expectant parents. The risk factors for placental abruption are discussed. What comment validates accurate learning by the parents? a. "Since I am over 30, I run a much higher risk of developing this problem." b. "Placental abruption is quite painful and I will need to let the doctor know if I begin to have abdominal pain." c. "I need a cesarean section if I develop this problem." d. "If I develop this complication, I will have bright red vaginal bleeding,"

A 17-year-old client comes to the clinic because of irregular menstrual bleeding and facial acne. The client is overweight, despite exercising daily, and has excessive hair growth on the chin and abdomen. The nurse explains to the client that blood will be drawn for which purposes? Select all that apply.

To screen for insulin resistance To obtain fasting cholesterol levels To measure androgen level There is no single blood test to diagnose polycystic ovary syndrome (PCOS). However, a blood test to screen for insulin resistance, elevated androgen level, and elevated cholesterol level are used to identify clients with PCOS. There is no blood test to screen for hypertension.

A young mother gives birth to twin boys who shared the same placenta. What serious complication are they at risk for? -Twin-to-twin transfusion syndrome (TTTS) -HELLP syndrome -TORCH syndrome -ABO incompatibility

Twin-to-twin transfusion syndrome (TTTS) When twins share a placenta, a serious condition called twin-to-twin transfusion syndrome (TTTS) can occur.

A student nurse asks the instructor what percentage of clinically recognized pregnancies end in miscarriages during the first trimester. The most accurate response from the registered nurse is which of the following? a) 11% to 20% b) 21% to 30% c) 5% to 10% d) 31% to 40%

a) 11% to 20% Rationale: During the first trimester, 10% to 20% of all clinically recognized pregnancies end in miscarriage.

Rho(D) immune globulin will be prescribed for an Rh negative mother undergoing which test?

amniocentesis Amniocentesis is a procedure requiring a needle to enter into the amniotic sac. There is a risk of mixing of the fetal and maternal blood which could result in blood incompatibility. A contraction test, a nonstress test, and biophysical profile are not invasive, so there would be no indication for Rho(D) immune globulin to be administered.

A woman who is Rh negative asks you how many children she will be able to have before Rh incompatibility causes them to die in utero. Your best response would be that a) no more than three children is recommended. b) she will have to ask her physician. c) only her next child will be affected. d) as long as she receives RhIG, there is no limit.

as long as she receives RhIG, there is no limit. Correct Explanation: Because RhIG supplies passive antibodies, it prevents the woman from forming antibodies. Without antibodies that could affect the fetus, the woman could have as many children as she wants.

Which of the following is CORRECT regarding nutrition recommendations for controlling diabetes complications?

both A and B are correct For microvascular complications, the recommendation based on strong evidence is to reduce protein intake to 0.8 to 1.0 g/kg for people with early-stage kidney disease, and the recommendation for CVD is to control hemoglobin A1c as close to normal as possible without significant hypoglycemia.

A client with severe preeclampsia is receiving magnesium sulfate as part of the treatment plan. To ensure the client's safety, which compound would the nurse have readily available? ferrous sulfate potassium chloride calcium gluconate calcium carbonate

calcium gluconate

A woman of 16 weeks' gestation telephones the nurse because she has passed some "berry-like" blood clots and now has continued dark brown vaginal bleeding. Which action would the nurse instruct the woman to do?

come to the health facility with any vaginal material passed This is a typical time in pregnancy for gestational trophoblastic disease to present. Asking the woman to bring any material passed vaginally would be important so the material can be assessed for this.

A woman with an incomplete abortion is to receive misoprostol. The woman asks the nurse, "Why am I getting this drug?" The nurse responds to the client, integrating understanding that this drug achieves which effect? suppresses the immune response to prevent isoimmunization halts the progression of the abortion ensures passage of all the products of conception alleviates strong uterine cramping

ensures passage of all the products of conception

A nurse is monitoring a client with PROM who is in labor and observes meconium in the amniotic fluid. What does the observation of meconium indicate? central nervous system (CNS) involvement fetal distress related to hypoxia infection cord compression

fetal distress related to hypoxia

A pregnant woman is diagnosed with placental abruption (abruptio placentae). When reviewing the woman's physical assessment in her medical record, which finding would the nurse expect?

firm, ridged uterus on palpation The uterus is firm-to-rigid to the touch with abruptio placentae. It is soft and relaxed with placenta previa. Bleeding associated with abruptio placentae occurs suddenly and is usually dark in color. Bleeding also may not be visible. A gradual onset of symptoms is associated with placenta previa. Fetal distress or absent fetal heart rate may be noted with abruptio placentae. The woman with abruptio placentae usually experiences constant uterine tenderness on palpation.

A pregnant woman is diagnosed with placental abruption (abruptio placentae). When reviewing the woman's physical assessment in her medical record, which finding would the nurse expect?

firm, rigid uterus on palpation Explanation: The uterus is firm-to-rigid to the touch with abruptio placentae. It is soft and relaxed with placenta previa. Bleeding associated with abruptio placentae occurs suddenly and is usually dark in color. Bleeding also may not be visible. A gradual onset of symptoms is associated with placenta previa. Fetal distress or absent fetal heart rate may be noted with abruptio placentae. The woman with abruptio placentae usually experiences constant uterine tenderness on palpation.

A client has come to the office for a prenatal visit during her 22nd week of gestation. On examination, it is noted that her blood pressure has increased to 138/90 mm Hg. Her urine is negative for proteinuria. The nurse recognizes which factor as the potential cause?

gestational hypertension

A client in her 20th week of gestation develops HELLP syndrome. What are features of HELLP syndrome? Select all that apply. hyperthermia hemolysis elevated liver enzymes leukocytosis low platelet count`

hemolysis elevated liver enzymes low platelet count

A pregnant 36-year-old woman has presented to the emergency department with vaginal bleeding. While reviewing the client's history, the nurse suspects placenta previa when which risk factors are found in her record? Select all that apply.

infertility treatment smoking advancing maternal age previous induced surgical abortion

The nurse is seeing a client who after experiencing symptoms of hyperglycemia, immediately stopped eating foods high in carbohydrates. The client recognizes that this strategy will prevent which of the following?

ketoacidosis People with diabetes, especially type 1, are susceptible to diabetic ketoacidosis. Accumulation of acidic ketone bodies due to fat catabolism will cause ketoacidosis and ketonuria, which can lead to a diabetic coma and eventually death.

Some women experience a rupture of their membranes before going into true labor. A nurse recognizes that a woman who presents with premature prelabor rupture of membranes (PPROM) has completed how many weeks of gestation? less than 38 weeks less than 40 weeks less than 39 weeks less than 37 weeks

less than 37 weeks

A pregnant client diagnosed with hyperemesis gravidarum is prescribed intravenous fluids for rehydration. When preparing to administer this therapy, which solution would the nurse anticipate being prescribed initially?

normal saline For the client with hyperemesis gravidarum, parenteral fluids and drugs are prescribed to rehydrate the client and reduce the symptoms. The first choice for fluid replacement is generally isotonic, such as normal saline, which aids in preventing hyponatremia, with vitamins (pyridoxine, or vitamin B6) and electrolytes added. Dextrose 5% and water and 0.45% sodium chloride are hypotonic solutions that would cause the cells to swell and possibly burst. Albumin could lead to fluid overload.

A woman at 34 weeks' gestation presents to labor and delivery with vaginal bleeding. Which finding from the obstetric examination would lead to a diagnosis of placental abruption (abruptio placentae)?

onset of vaginal bleeding was sudden & painful Sudden onset of abdominal pain and vaginal bleeding with a rigid uterus that does not relax are signs of a placental abruption (abruptio placentae). The other findings are consistent with a diagnosis of placenta previa.

A pregnant woman with preeclampsia is to receive magnesium sulfate IV. Which assessment should the nurse prioritize before administering a new dose? blood pressure patellar reflex heart rate anxiety level

patellar reflex

The nurse understands the need to be aware of the potential of bleeding disorders in pregnant clients. Which disorder should she be aware of that occurs in the second trimester?

placenta previa Second trimester bleeding usually results from placenta previa, where the placenta lies either partially or completely over the cervical os. The pregnant client begins to experience vaginal bleeding of bright, red blood. Spontaneous abortion (miscarriage), hydatidiform mole, and ectopic pregnancy occur in the first trimester and cervical insufficiency is not a bleeding disorder.

A woman you care for has an Rh-negative blood type. Following the birth of her infant, you administer her RhIG (D immune globulin). The purpose of this is to a) prevent maternal D antibody formation. b) promote maternal D antibody formation. c) stimulate maternal D immune antigens. d) prevent fetal Rh blood formation.

prevent maternal D antibody formation. Explanation: Because RhIG contains passive antibodies, the solution will prevent the woman from forming long-lasting antibodies.

A woman in labor is at risk for abruptio placentae. Which assessment would most likely lead the nurse to suspect that this has happened? sharp fundal pain and discomfort between contractions painless vaginal bleeding and a fall in blood pressure pain in a lower quadrant and increased pulse rate an increased blood pressure and oliguria

sharp fundal pain and discomfort between contractions

A woman in labor is at risk for abruptio placentae. Which assessment would most likely lead the nurse to suspect that this has happened?

sharp fundal pain and discomfort between contractions An abruptio placentae refers to premature separation of the placenta from the uterus. As the placenta loosens, it causes sharp pain. Labor begins with a continuing nagging sensation.

When assessing a pregnant woman with vaginal bleeding, the nurse would suspect a threatened abortion based on which finding?

slight vaginal bleeding

When assessing a pregnant woman with vaginal bleeding, the nurse would suspect a threatened abortion based on which finding? cervical dilation passage of fetal tissue slight vaginal bleeding strong abdominal cramping

slight vaginal bleeding

A 44-year-old client has lost several pregnancies over the last 10 years. For the past 3 months, she has had fatigue, nausea, and vomiting. She visits the clinic and takes a pregnancy test; the results are positive. Physical examination confirms a uterus enlarged to 13 weeks' gestation; fetal heart tones are heard. Ultrasound reveals that the client is experiencing some bleeding. Considering the client's prenatal history and age, what does the nurse recognize as the greatest risk for the client at this time? spontaneous abortion (miscarriage) preterm labor premature birth hypertension

spontaneous abortion (miscarriage)

A pregnant woman has arrived to the office reporting vaginal bleeding. Which finding during the assessment would lead the nurse to suspect an inevitable spontaneous abortion (miscarriage)?

strong abdominal cramping Strong abdominal cramping is associated with an inevitable spontaneous abortion (miscarriage). Slight vaginal bleeding early in pregnancy and a closed cervical os are associated with a threatened abortion. With an inevitable abortion, passage of the products of conception may occur. No fetal tissue is passed with a threatened abortion.

The nurse recognizes which type of diabetes is characterized by the absence of insulin secretion?

type 1 diabetes Type 1 diabetes is characterized by the absence of insulin. Type 2 diabetes is characterized by a combination of insulin resistance and relative insulin deficiency.

A woman at 8 weeks' gestation is admitted for ectopic pregnancy. She is asking why this has occurred. The nurse knows that which factor is a known risk factor for ectopic pregnancy?

use of IUD for contraception

The nurse caring for a newborn has to perform assessment at various intervals. When should the nurse complete the second assessment for the newborn? -within 30 minutes after birth, in the birthing area -within the first 2 to 4 hours, when the newborn reaches the nursery -prior to the newborn being discharged -24 hours after the newborn's birth

within the first 2 to 4 hours, when the newborn reaches the nursery The nurse should complete the second assessment for the newborn within the first 2 to 4 hours, when the newborn is in the nursery. The nurse should complete the initial newborn assessment in the birthing area and the third assessment before the newborn is discharged, whenever that may be.

A pregnant client has an Rh-negative blood type. Following the birth of the client's infant, the nurse administers her Rho(D) immune globulin. The purpose of this is to:

prevent maternal D antibody formation.

The nurse is caring for a client with preeclampsia and understands the need to auscultate this client's lung sounds every 2 hours. Why would the nurse do this? Pulmonary hypertension Pulmonary edema Pulmonary emboli Pulmonary atelectasis

Pulmonary edema

The nurse is caring for a woman at 32 weeks' gestation with severe preeclampsia. Which assessment finding should the nurse prioritize after the administration of hydralazine to this client? Gastrointestinal bleeding Tachycardia Sweating Halos around lights

Tachycardia

uterus it is soft and relaxed with,Bright red vaginal bleeding

placenta previa

Eclampsia Blood Pressure? Seizures/coma? Hyperreflexia? Other signs and symptoms?

Blood pressure: >160/110 mm Hg Seizures/coma? YES Hyperreflexia? YES Other signs and symptoms? Severe headache Generalized edema right upper quadrant or epigastric pain Visual disturbances Cerebral hemorrhage Renal failure HELLP

A nurse is caring for a young woman who is in her 10th week of gestation. She comes into the clinic reporting vaginal bleeding. Which assessment finding best correlates with a diagnosis of hydatidiform mole?

Dark red, "clumpy" vaginal discharge If a complete molar pregnancy continues into the second trimester undetected, other signs and symptoms appear. The woman often presents with complaints of dark to bright red vaginal bleeding and pelvic pain. Infrequently, she will report passage of grapelike vesicles.

A 35-year-old client is seen for her 2-week postoperative appointment after a suction curettage was performed to evacuate a hydatidiform mole. The nurse explains that the human chorionic gonadotropin (hCG) levels will be reviewed every 2 weeks and teaches about the need for reliable contraception for the next 6 months to a year. The client states, "I'm 35 already. Why do I have to wait that long to get pregnant again?" What is the nurse's best response? "After a curettage procedure, it is recommended that you give your body some time to build up its stores." "A contraceptive is used so that a positive pregnancy test resulting from a new pregnancy will not be confused with the increased level of hCG that occurs with a developing malignancy." "Since you are at the end of your reproductive years, it is suggested that you don't try to have any more pregnancies." "You may need chemotherapy, so we don't want to risk pregnancy."

"A contraceptive is used so that a positive pregnancy test resulting from a new pregnancy will not be confused with the increased level of hCG that occurs with a developing malignancy."

A woman of 16 weeks' gestation telephones the nurse because she has passed some "berry-like" blood clots and now has continued dark brown vaginal bleeding. Which action would the nurse instruct the woman to do?

"Come to the health facility with any vaginal material passed."

A client at 11 weeks' gestation experiences pregnancy loss. The client asks the nurse if the bleeding and cramping that occurred during the miscarriage were caused by working long hours in a stressful environment. What is the most appropriate response from the nurse? "Your spontaneous bleeding is not work-related." "It is hard to know why a woman bleeds during early pregnancy." "I can understand your need to find an answer to what caused this. Let's talk about this further." "Something was wrong with the fetus."

"I can understand your need to find an answer to what caused this. Let's talk about this further."

A client at 11 weeks' gestation experiences pregnancy loss. The client asks the nurse if the bleeding and cramping that occurred during the miscarriage were caused by working long hours in a stressful environment. What is the most appropriate response from the nurse? "Something was wrong with the fetus." "It is hard to know why a woman bleeds during early pregnancy." "Your spontaneous bleeding is not work-related." "I can understand your need to find an answer to what caused this. Let's talk about this further."

"I can understand your need to find an answer to what caused this. Let's talk about this further."

A client at 11 weeks' gestation experiences pregnancy loss. The client asks the nurse if the bleeding and cramping that occurred during the miscarriage were caused by working long hours in a stressful environment. What is the most appropriate response from the nurse?

"I can understand your need to find an answer to what caused this. Let's talk about this further." Talking with the client may assist her to explore her feelings. She and her family may search for a cause for a spontaneous early bleeding so they can plan for future pregnancies. Even with modern technology and medical advances, however, a direct cause cannot usually be determined.

People with diabetes should avoid the use of fructose as an added sweetener for which of the following reasons?

"It causes adverse effects on serum triglycerides" Fructose causes adverse effects on serum triglycerides and LDL cholesterol.

A pregnant client at 32 weeks' gestation calls the clinic and informs the nurse that she thinks her membranes are leaking. She states that some clear fluid has run down her leg. What is the best response by the nurse?

"It is best for you to visit a hospital immediately. They can use a nitrazine strip to determine if it is amniotic fluid."

A nurse has been assigned to assess a pregnant client for placental abruption (abruptio placentae). For which classic manifestation of this condition should the nurse assess?

"Knife-like" abdominal pain with vaginal bleeding The classic manifestations of abruption placenta are painful dark red vaginal bleeding, "knife-like" abdominal pain, uterine tenderness, contractions, and decreased fetal movement. Painless bright red vaginal bleeding is the clinical manifestation of placenta previa. Generalized vasospasm is the clinical manifestation of preeclampsia and not of abruptio placentae.

A client with a history of cervical insufficiency is seen for reports of pink-tinged discharge and pelvic pressure. The primary care provider decides to perform a cervical cerclage. The nurse teaches the client about the procedure. Which client response indicates that the teaching has been effective? "Staples are put in the cervix to prevent it from dilating." "The cervix is glued shut so no amniotic fluid can escape." "Purse-string sutures are placed in the cervix to prevent it from dilating." "A cervical cap is placed so no amniotic fluid can escape."

"Purse-string sutures are placed in the cervix to prevent it from dilating." Explanation: The cerclage, or purse string suture, is inserted into the cervix to prevent preterm cervical dilation (dilatation) and pregnancy loss. Staples, glue, or a cervical cap will not prevent the cervix from dilating.

A client experiences a threatened abortion. She is concerned about losing the pregnancy and asks what activity level she should maintain. What is the most appropriate response from the nurse?

"Restrict your physical activity to moderate bedrest." With a threatened abortion, moderate bedrest and supportive care are recommended. Regular physical activity may increase the chances of miscarriage. Strict bedrest is not necessary. Activity restrictions are part of standard medical management.

A client at 11 weeks' gestation experiences pregnancy loss. The client asks the nurse if the bleeding and cramping that occurred during the miscarriage were caused by working long hours in a stressful environment. What is the most appropriate response from the nurse?

"I can understand your need to find an answer to what caused this. Let's talk about this further" Talking with the client may assist her to explore her feelings. She and her family may search for a cause for a spontaneous early bleeding so they can plan for future pregnancies. Even with modern technology and medical advances, however, a direct cause cannot usually be determined.

The nurse is preparing the plan of care for a woman hospitalized for hyperemesis gravidarum. Which interventions would the nurse most likely include? Select all that apply.

1. maintaining NPO status for the first day or two 2. administering antiemetic agents 3. obtaining baseline blood electrolyte levels 4. monitoring intake and output

A woman is receiving magnesium sulfate as part of her treatment for severe preeclampsia. The nurse is monitoring the woman's serum magnesium levels. The nurse determines that the drug is at a therapeutic level based on which result? 3.3 mEq/L 8.4 mEq/L 6.1 mEq/L 10.8 mEq/L

6.1 mEq/L

A client has been diagnosed with a partial molar pregnancy. The nurse is explaining this condition. The nurse knows the teaching was effective when the client states that as a result of an error during fertilization, there are how many chromosomes in her pregnancy?

69 A normal gamete has 23 chromosomes. At fertilization, the sperm fertilizes the ovum, resulting in 23 pairs or 46 chromosomes. In a partial molar pregnancy, two sperm penetrate the ovum, resulting in a total of 69 chromosomes. A pregnancy with 47 chromosomes results in a trisomic condition such as trisomy 21 or trisomy 18.

a. Phrenic nerve irritation

A nurse is caring for a client undergoing treatment for ectopic pregnancy. Which symptom is observed in a client if rupture or hemorrhaging occurs before the ectopic pregnancy is successfully treated? a. Phrenic nerve irritation b. Painless bright red vaginal bleeding c. Fetal distress d. Tetanic contractions

d. Administer IV NS with vitaimins and electrolytes

A nurse is caring for a client with hyperemesis gravidarum. Which nursing action is the priority for this client? a. Administer total parenteral nutrition b. Administer an antiemetic c. Set up for a percutaneous endoscopic gastrostomy d. Administer IV NS with vitamins and electrolytes

b. cervical incompetence

A pregnant client at 20 weeks' gestation arrives at the health care facility reporting excessive vaginal bleeding and no fetal movements. Which assessment finding would the nurse anticipate in this situation? a. ectopic pregnancy b. cervical incompetence c. placenta previa d. congenital malformations

c. Eat small, frequent meals throughout the day

A pregnant client with hyperemesis gravidarum needs advice on how to minimize nausea and vomiting. Which instruction should the nurse give this client? a. Lie down or recline for at least 2 hours after eating b. Avoid dry crackers, toast, and soda c. Eat small, frequent meals throughout the day d. Decrease intake of carbonated beverages

B. An inevitable abortion is characterized by vaginal bleeding that is greater than slight, rupture of membranes, cervical dilation, strong abdominal cramping, and possible passage of products of conception.

A pregnant woman, approximately 12 weeks' gestation, comes to the emergency department after calling her health care provider's office and reporting moderate vaginal bleeding. Assessment reveals cervical dilation and moderately strong abdominal cramps. She reports that she has passed some tissue with the bleeding. The nurse interprets these findings to suggest which of the following? a. Threatened abortion b. Inevitable abortion c. Incomplete abortion d. Missed abortion

d. Amniocentesis

A primigravida 28-year-old client is noted to have Rh negative blood and her husband is noted to be Rh positive. The nurse should prepare to administer RhoGAM after which diagnostic procedure? a. Contraction test b. Nonstress test c. Biophysical profile d. Amniocentesis

A nurse is caring for a pregnant client with sickle cell anemia. What should the nursing care for the client include? SATA A. Teach the client meticulous hand-washing B. Assess serum electrolyte levels of the client at each visit C. Instruct client to consume protein-rich food D. Assess hydration status of the client at each visit E. Urge the client to drink 8 to 10 glasses of fluid daily

A. Teach the client meticulous hand-washing D. Assess hydration status of the client at each visit E. Urge the client to drink 8 to 10 glasses of fluid daily

A client with preeclampsia is receiving magnesium sulfate to suppress or control seizures. Which nursing intervention should a nurse perform to determine the effectiveness of therapy? Monitor intake and output. Assess the client's skin turgor. Assess the client's mucous membrane. Assess deep tendon reflexes.

Assess deep tendon reflexes.

At 37 weeks' gestation, a woman presents to labor and delivery complaining of intense, knife-like abdominal pain that started suddenly about 1 hour ago and has not subsided. On palpation, the abdomen is rigid and board-like and no vaginal bleeding is evident. What should the nurse do next? Assess fetal heart rate Prepare the client for an epidural Insert a Foley catheter Administer oxygen by face mask

Assess fetal heart rate

A pregnant patient is diagnosed with placenta previa. Which action should the nurse implement immediately for this patient?

Assess fetal heart sounds with an external monitor.

The nurse is preparing a woman for discharge after a birth and notes the mother's record indicates Rh negative and rubella titer is positive. Which nursing intervention will the nurse prioritize?

Assess the Rh of the baby. The cord blood should be assessed to determine the infant's Rh type. If it is negative, there is no need for any further treatment or concern. However, if it is Rh positive the mother needs to be assessed for possible administration of RhoGAM. The criteria for giving RhoGAM are as follows: The woman must be Rho(D) negative; the woman must not have anti-D antibodies (must not be sensitized); the infant must be Rho(D) positive (fetus cord blood is checked after birth); and a direct Coombs test (a test for antibodies performed on cord blood at delivery) must be weakly reactive or negative. This all needs to be completed within the first 72 hours of birth, so the mother can receive RhoGAM within the proper time frame.

When caring for a client with premature rupture of membranes (PROM), the nurse observes an increase in the client's pulse. What should the nurse do next?

Assess the client's temperature. A temperature elevation or an increase in the pulse of a client with PROM would indicate infection. Increase in the pulse does not indicate preterm labor or cord compression. The nurse should monitor FHR patterns continuously, reporting any variable decelerations suggesting cord compression. Respiratory distress syndrome is one of the perinatal risks associated with PROM.

A woman has been diagnosed as having pregnancy-induced hypertension. Which of the following is the most typical symptom of this? a) Blood pressure elevation b) Weight loss c) Susceptibility to infection d) Increased perspiration

Blood pressure elevation

A woman in her 20s has experienced a miscarriage at 10 weeks' gestation and asks the nurse at the hospital what went wrong. She is concerned that she did something that caused her to lose her baby. The nurse can reassure the woman by explaining that the most common cause of spontaneous miscarriage in the first trimester is related to which factor?

Chromosomal defects in the fetus

Monozygotic Twins

Develop when a single, fertilized ovum splits during the first 2 weeks after conception

A 24-year-old woman presents with vague abdominal pains, nausea, and vomiting. An urine hCG is positive after the client mentioned that her last menstrual period was 2 months ago. The nurse should prepare the client for which intervention if the transvaginal ultrasound indicates a gestation sac is found in the right lower quadrant? -Bed rest for the next 4 weeks -Intravenous administration of a tocolytic -Immediate surgery -Internal uterine monitoring

Immediate surgery The client presents with the signs and symptoms of an ectopic pregnancy, which is confirmed by the transvaginal ultrasound. Ectopic pregnancy means an embryo has implanted outside the uterus. Surgery is necessary to remove the growing structure before damage can occur to the woman's internal organs. Bed rest, a tocolytic, and internal uterine monitoring will not correct the situation. The growing structure must be removed surgically.

A pregnant woman is being evaluated for HELLP. The nurse reviews the client's diagnostic test results. An elevation in which result would the nurse interpret as helping to confirm this diagnosis? LDH white blood cells hematocrit platelet count

LDH

A woman in labor suddenly reports sharp fundal pain accompanied by slight dark red vaginal bleeding. The nurse should prepare to assist with which situation? Placenta previa obstructing the cervix Premature separation of the placenta Preterm labor that was undiagnosed Possible fetal death or injury

Premature separation of the placenta

A pregnant patient is developing HELLP syndrome. During labor, which order should the nurse question?

Prepare for epidural anesthesia.

A client at 27 weeks' gestation is admitted to the obstetric unit after reporting headaches and edema of her hands. Review of the prenatal notes reveals blood pressure consistently above 136/90 mm Hg. The nurse anticipates the health care provider will prescribe magnesium sulfate to accomplish which primary goal? -Decrease blood pressure -Decrease protein in urine -Prevent maternal seizures -Reverse edema

Prevent maternal seizures The primary therapy goal for any client with preeclampsia is to prevent maternal seizures. Use of magnesium sulfate is the drug therapy of choice for severe preeclampsia and is only used to manage and attempt to prevent progression to eclampsia. Magnesium sulfate therapy does not have as a primary goal of decreasing blood pressure, decreasing protein in the urine, or reversing edema.

The following hourly assessments are obtained by the nurse on a client with preeclampsia receiving magnesium sulfate: 97.3oF (36.2oC), HR 88, RR 12 breaths/min, BP 148/110 mm Hg. What other priority physical assessments by the nurse should be implemented to assess for potential toxicity? Magnesium sulfate level Reflexes Oxygen saturation Lung sounds

Reflexes

The following hourly assessments are obtained by the nurse on a client with preeclampsia receiving magnesium sulfate: 97.3oF (36.2oC), HR 88, RR 12 breaths/min, BP 148/110 mm Hg. What other priority physical assessments by the nurse should be implemented to assess for potential toxicity?

Reflexes Explanation: Reflex assessment is part of the standard assessment for clients on magnesium sulfate. The first change when developing magnesium toxicity may be a decrease in reflex activity. The health care provider needs to be notified immediately. A change in lung sounds and oxygen saturation are not indicative of magnesium sulfate toxicity. Hourly blood draws to gain information on the magnesium sulfate level are not indicated.

The following hourly assessments are obtained by the nurse on a client with preeclampsia receiving magnesium sulfate: 97.3oF (36.2oC), HR 88, RR 12 breaths/min, BP 148/110 mm Hg. What other priority physical assessments by the nurse should be implemented to assess for potential toxicity? -Lung sounds -Oxygen saturation -Reflexes -Magnesium sulfate level

Reflexes Reflex assessment is part of the standard assessment for clients on magnesium sulfate. The first change when developing magnesium toxicity may be a decrease in reflex activity. The health care provider needs to be notified immediately. A change in lung sounds and oxygen saturation are not indicative of magnesium sulfate toxicity. Hourly blood draws to gain information on the magnesium sulfate level are not indicated.

A nurse is conducting a presentation for a group of pregnant women about conditions that can occur during pregnancy and that place the woman at high-risk. When discussing blood incompatibilities, which measure would the nurse explain as most effective in preventing isoimmunization during pregnancy?

Rho(D) immune globulin administration to Rh-negative women

a. Monitor the client's vital signs and bleeding.

The nurse is caring for a pregnant client with fallopian tube rupture. Which intervention is the priority for this client? a. Monitor the client's vital signs and bleeding. b. Monitor the fetal heart rate (FHR). c. Monitor the mass with transvaginal ultrasound. d. Monitor the client's beta-hCG level.

a. Keep the suction equipment readily available.

The nurse is caring for a pregnant client with severe preeclampsia. Which nursing intervention should a nurse perform to institute and maintain seizure precautions in this client? a. Keep the suction equipment readily available. b. Place the client in a supine position. c. Keep head of bed slightly elevated. d. Provide a well-lit room.

monozygotic

__________________ twins develop when a single, fertilized ovum splits during the first 2 weeks after conception.

A woman in week 35 of her pregnancy with severe hydramnios is admitted to the hospital. The nurse recognizes that which of the following is the biggest concern regarding this client? a) Preterm rupture of membranes followed by preterm birth b) Hemorrhaging c) Development of gestational trophoblastic disease d) Development of eclampsia

a) Preterm rupture of membranes followed by preterm birth Rationale: Even with precautions, in most instances of hydramnios, there will be preterm rupture of the membranes because of excessive pressure, followed by preterm birth. The other answers are not as big of concerns as preterm birth, in this situation.

Rho(D) immune globulin will be prescribed for an Rh negative mother undergoing which test?

amniocentesis

Hemolysis

associated with ABO incompatibility is limited to mothers with type O blood and their fetuses with type A or B blood.

A high-risk pregnant client is determined to have gestational hypertension. The nurse suspects that the client has developed severe preeclampsia based on which finding?

blurred vision

least 6 hours apart

bp measurment for GH

Which of the following would the nurse interpret as indicating that a pregnant client with gestational hypertension has developed severe preeclampsia? a) Proteinuria of 300 mg/24 hours b) Blood pressure of 150/100 mm Hg c) Blurred vision d) Mild facial edema

c) Blurred vision Rationale: Visual symptoms such as blurred vision and blind spots suggest severe preeclampsia. Severe preeclampsia is characterized by a blood pressure of 160/110 mm Hg. Mild facial edema or hand edema occurs with mild preeclampsia. Proteinuria in severe preeclampsia is greater than 500 mg/24 hours.

A novice nurse asks to be assigned to the least complex antepartum client. Which condition would necessitate the least complex care requirements?

gestational hypertension

A client has been admitted with abruptio placentae. She has lost 1,200 mL of blood, is normotensive, and ultrasound indicates approximately 30% separation. The nurse documents this as which classification of abruptio placentae?

grade 2

The acronym HELLP represent

hemolysis, elevated liver enzymes, and low platelets

Referred pain to the shoulder area indicates bleeding into the abdomen caused by

phrenic nerve irritation when a tubal pregnancy ruptures.

A 44-year-old client has lost several pregnancies over the last 10 years. For the past 3 months, she has had fatigue, nausea, and vomiting. She visits the clinic and takes a pregnancy test; the results are positive. Physical examination confirms a uterus enlarged to 13 weeks' gestation; fetal heart tones are heard. Ultrasound reveals that the client is experiencing some bleeding. Considering the client's prenatal history and age, what does the nurse recognize as the greatest risk for the client at this time?

pregnancy loss

A client is admitted to labor and birth for management of severe preeclampsia. An IV infusion of magnesium sulfate is started. What is the primary goal for magnesium sulfate therapy?

prevent maternal seizures

A woman in week 16 of her pregnancy calls her primary care provider's office to report that she has experienced abdominal cramping, cervical dilation, vaginal spotting, and the passing of tissue. The nurse instructs the client to bring the passed tissue to the hospital with her. What is the correct rationale for this instruction?

to determine whether gestational trophoblastic disease is present

Oxytocin

used to stimulate uterine contractions and would be inappropriate for use with an ectopic pregnancy

The nurse through assessment can best differentiate between placenta previa and abruptio placentae by which of the following signs and or symptoms. a) Bleeding amount and consistency. b) Shape of the abdomen. c) Low back pain. d) Uterine tone and contractions of the uterus.

...

The client arrives in the office and reports that her feet and legs swelling. During a client evaluation, the nurse notes that she can elicit a 4-mm skin depression that disappears in 10 to 15 seconds. The nurse correctly documents this finding as:

2+ pitting edema. Pitting edema is recorded using the following relative scale: 1+ is a 2-mm depression that disappears rapidly; 2+ is a 4-mm depression that disappears in 10 to 15 seconds; 3+ is a 6-mm depression that lasts more than one minute; and 4+ is an 8-mm depression that lasts 2 to 3 minutes.

During pregnancy a woman's blood volume increases to accommodate the growing fetus to the point that vital signs may remain within normal range without showing signs of shock until the woman has lost what percentage of her blood volume?

40%

Proteinuria in severe preeclampsia is

500 mg per 24 hours.

b. Lateral recumbent position

A client is seeking advice for his pregnant wife, who is experiencing mild elevations in blood pressure. In which position should a nurse recommend the pregnant client rest? a. Supine position b. Lateral recumbent position c. Left lateral lying position d. Head of the bed slightly elevated

d. A client who had a myomectomy to remove fibroids

A nurse is assessing pregnant clients for the risk of placenta previa. Which of the following clients faces the greatest risk for this condition? a. A 23-year-old multigravida client b. A client with a history of alcohol abuse c. A client with a structurally defective cervix d. A client who had a myomectomy to remove fibroids

B. When the placenta separates from the uterine wall, it causes irritation and bleeding into the muscle fibers, which causes pain.

A woman is suspected of having abruptio placentae. Which of the following would the nurse expect to assess as a classic symptom? a. Painless, bright-red bleeding b. "Knife-like" abdominal pain c. Excessive nausea and vomiting d. Hypertension and headache

A nurse is monitoring a client with spontaneous abortion who has been prescribed misoprostol. Which symptoms are common adverse effects associated with misoprostol? SATA A. Constipation B. Dyspepsia C. Headache D. Hypotension E. Tachycardia

B. Dyspepsia D. Hypotension E. Tachycardia

A pregnant 36-year-old woman has presented to the emergency department with vaginal bleeding. While reviewing the client's history, the nurse suspects placenta previa when which risk factors are found in her record? Select all that apply. previous induced surgical abortion advancing maternal age infertility treatment hypotension smoking

Everything but hypotension

A patient is admitted with a diagnosis of ectopic pregnancy. For what should the nurse anticipate preparing the patient?

Immediate surgery

The nurse is identifying nursing diagnoses for a patient with gestational hypertension. Which diagnosis would be the most appropriate for this patient?

Ineffective tissue perfusion related to vasoconstriction of blood vessels

A pregnant woman has been admitted to the hospital due to severe preeclampsia. Which measure will be important for the nurse to include in the care plan?

Institute and maintain seizure precautions.

A patient recovering from an uneventful vaginal delivery is prescribed Rho(D) immune globulin (RhIG). What should the nurse explain to the patient regarding the purpose of this medication?

It prevents maternal D antibody formation.

The nurse is caring for a pregnant client with fallopian tube rupture. Which intervention is the priority for this client?

Monitor the client's vital signs and bleeding.

A client at 36 weeks' gestation experiences vaginal bleeding. Which conditions might be the cause of the client's bleeding? Select all that apply. Bloody show Placental abruption (abruptio placentae) Ectopic pregnancy Placenta previa Spontaneous abortion (miscarriage)

Placental abruption (abruptio placentae) Placenta previa Bloody Show

A woman in labor has sharp fundal pain accompanied by slight vaginal bleeding. Which of the following would be the most likely cause of these symptoms? a) Placenta previa obstructing the cervix b) Possible fetal death or injury c) Preterm labor that was undiagnosed d) Premature separation of the placenta

Premature separation of the placenta Correct Explanation: Premature separation of the placenta begins with sharp fundal pain, usually followed by vaginal bleeding. Placenta previa usually produces painless bleeding; labor contractions are more often described as cramping

The following hourly assessments are obtained by the nurse on a client with preeclampsia receiving magnesium sulfate: 97.3oF (36.2oC), HR 88, RR 1, BP 148/110 mm Hg. What other priority physical assessments by the nurse should be implemented to assess for potential toxicity?

Reflexes

A nurse is describing the use of Rho(D) immune globulin as the therapy of choice for isoimmunization in Rh-negative women and for other conditions to a group of nurses working at the women's health clinic. The nurse determines that additional teaching is needed when the group identifies which situation as an indication for Rho(D) immune globulin?

STIs

A female client presents to the emergency department reporting she recently had a positive home pregnancy test but is now suspected of having an ectopic pregnancy. Which assessment should the nures prioritize?

Shoulder pain

The nurse is caring for a client who has a multifetal pregnancy. What topic should the nurse prioritize during health education?

Signs of preterm labor

latent period

The time interval from rupture of membranes to the onset of regular contractions

Latent

The time interval from rupture of membranes to the onset of regular contractions is termed the _______ period

A young woman presents at the emergency department reporting lower abdominal cramping and spotting at 12 weeks' gestation. The primary care provider performs a pelvic examination and finds that the cervix is closed. What does the care provider suspect is the cause of the cramps and spotting? Ectopic pregnancy Habitual abortion Cervical insufficiency Threatened abortion

Threatened abortion

Spontaneous miscarriage occurs in 5% to 10% of all pregnancies. a) False b) True

a) False Rationale: Spontaneous miscarriage occurs in 15% to 30% of all pregnancies and arises from natural causes.

The uterus is firm to rigid to the touch with

abrupto placenta

A client who has experienced an incomplete abortion is prescribed mifepristone to assist in removing the retained products of conception. Which medication would the nurse expect to administer if prescribed before administering mifepristone? vitamin K to reduce bleeding opioid analgesic for relief of cramping diuretic to promote fluid loss antiemetic to minimize nausea

antiemetic to minimize nausea

A woman is being admitted to the obstetric unit for severe preeclampsia. When assigning room placement, which area would be most appropriate? beside the supply room at the end of the hallway across from the nurse's station near the staff elevator

at the end of the hallway

A pregnant woman at 12 weeks' gestation calls you because she has begun minimal fresh vaginal spotting. She is distressed because her physician says she is not going to do anything for her but "wait and see." Which of the following would you suggest? a) Suggest she take an over-the-counter tocolytic just to feel secure. b) Tell her that medication to prolong a 12-week pregnancy usually is not advised. c) Advise her to ask for a second physician opinion. d) Explain that her doctor meant for her to maintain strict bed rest by "wait and see."

b) Tell her that medication to prolong a 12-week pregnancy usually is not advised. Rationale: Because many early pregnancy losses occur as the result of chromosome abnormalities, an aggressive approach to prolong these is not usually recommended.

A pregnant woman at 12 weeks' gestation calls you because she has begun minimal fresh vaginal spotting. She is distressed because her physician says she is not going to do anything for her but "wait and see." Which of the following would you suggest? a) Suggest she take an over-the-counter tocolytic just to feel secure. b) Tell her that medication to prolong a 12-week pregnancy usually is not advised. c) Explain that her doctor meant for her to maintain strict bed rest by "wait and see." d) Advise her to ask for a second physician opinion.

b) Tell her that medication to prolong a 12-week pregnancy usually is not advised. Rationale: Because many early pregnancy losses occur as the result of chromosome abnormalities, an aggressive approach to prolong these is not usually recommended.

n an effort to decrease complications for the infant right after birth, the nurse would expect to administer which medication for prophylaxis of potential eye conditions? -erythromycin ophthalmic ointment -silver nitrate solution -vitamin K -gentamicin ophthalmic ointment

erythromycin ophthalmic ointment Erythromycin ophthalmic ointment is the only drug approved by the U.S. Food and Drug Administration (FDA) for the prophylaxis of gonococcal neonatorum in the United States. Silver nitrate solution was once used for eye prophylaxis, but it is no longer used because it has little efficacy in preventing chlamydial eye disease. Vitamin K is used to promote blood clotting in the newborn. Gentamicin is not used for newborn eye prophylaxis.

A client has been admitted with abruptio placentae. She has lost 1,200 mL of blood, is normotensive, and ultrasound indicates approximately 30% separation. The nurse documents this as which classification of abruptio placentae? grade 2 grade 1 grade 3 grade 4

grade 2

A nurse is providing care to a client who has been diagnosed with a common benign form of gestational trophoblastic disease. The nurse identifies this as:

hydatidiform mole

Promethazine and ondansetron are antiemetics that may be used to trea

hyperemesis gravidarum.

Which medication would the nurse prepare to administer if prescribed as treatment for an unruptured ectopic pregnancy?

methotrexate

The nurse is caring for a client with preeclampsia and understands the need to auscultate this client's lung sounds every 2 hours to detect which condition?

pulmonary edema Explanation: In the hospital, monitor blood pressure at least every 4 hours for mild preeclampsia and more frequently for severe disease. In addition, it is important to auscultate the lungs every 2 hours. Adventitious sounds may indicate developing pulmonary edema

A woman at 28 weeks' gestation has been hospitalized with moderate bleeding that is now stabilizing. The nurse performs a routine assessment and notes the client sleeping, lying on the back, and electronic fetal heart rate (FHR) monitor showing gradually increasing baseline with late decelerations. Which action will the nurse perform first?

reposition to the side The fetus is showing signs of fetal distress. The immediate treatment is putting the client in a side-lying position to ensure adequate perfusion to the fetus. After placing the client on the side, the nurse should re-assess the FHR and determine if oxygen, IV fluids, and calling the health care provider are needed.

A client at 37 weeks' gestation presents to the emergency department with a BP 150/108 mm Hg, 1+ pedal edema, 1+ proteinuria, and normal deep tendon reflexes. Which assessment should the nurse prioritize as the client is administered magnesium sulfate IV? respiratory rate ability to sleep urine protein hemoglobin

respiratory rate

When assessing a pregnant woman with vaginal bleeding, the nurse would suspect a threatened abortion based on which finding?

slight vaginal bleeding Slight vaginal bleeding early in pregnancy, no cervical dilation, and a closed cervical os are associated with a threatened abortion. Strong abdominal cramping is associated with an inevitable abortion. With an inevitable abortion, passage of the products of conception may occur. No fetal tissue is passed with a threatened abortion.

A pregnant woman has arrived to the office reporting vaginal bleeding. Which finding during the assessment would lead the nurse to suspect an inevitable spontaneous abortion (miscarriage)? strong abdominal cramping slight vaginal bleeding no passage of fetal tissue closed cervical os

strong abdominal cramping

total placenta previa

the internal cervical os is completely covered by the placenta

partial placenta previa ?

the internal os is partially covered by the placenta

marginal placenta previa

the placenta is at the margin or edge of the internal os;

; low-lying placenta previa

the placenta is implanted in the lower uterine segment and is near the internal os but does not reach it.

A woman in week 16 of her pregnancy calls her primary care provider's office to report that she has experienced abdominal cramping, vaginal spotting, and the passing of tissue. The nurse instructs the client to bring the passed tissue to the hospital with her. What is the correct rationale for this instruction? to determine whether gestational trophoblastic disease is present to determine whether infection is present to determine whether the fetus is viable to determine the stage of development of the fetus

to determine whether gestational trophoblastic disease is present Explanation: Gestational trophoblastic disease is abnormal proliferation and then degeneration of the trophoblastic villi. The embryo fails to develop beyond a primitive start. Abnormal trophoblast cells must be identified because they are associated with choriocarcinoma, a rapidly metastasizing malignancy. This is why it is important for any woman who begins to miscarry at home to bring any clots or tissue passed to the hospital with her. The presence of clear fluid-filled cysts changes the diagnosis from a simple miscarriage to gestational trophoblastic disease. The client is not instructed to bring in passed tissue to determine whether infection is present or the fetus is viable or to determine the stage of development of the fetus.

The nurse is teaching a client about nutrition in diabetes management. What should be the target goal?

to keep blood glucose levels as near normal as possible The primary goal of diabetes management is to keep blood sugar level low and as near normal as possible. This will help to prevent the development of complications.

A young mother gives birth to twin boys who shared the same placenta. What serious complication are they at risk for?

twin-to-twin transfusion syndrome (TTTS)

A young mother gives birth to twin boys who shared the same placenta. What serious complication are they at risk for?

twin-to-twin transfusion syndrome (TTTS) When twins share a placenta, a serious condition called twin-to-twin transfusion syndrome (TTTS) can occur.

The nurse recognizes that the type of diabetes that is controlled primarily through diet, exercise, and oral antidiabetic agents is which type?

type 2 diabetes People with type 1 diabetes require exogenous insulin for survival. Recent clinical trials have demonstrated that lifestyle modification, namely, modest weight loss, moderate physical activity, and a healthy diet, is the most effective tool in preventing or delaying type 2 diabetes.

A woman at 8 weeks' gestation is admitted for ectopic pregnancy. She is asking why this has occurred. The nurse knows that which factor is a known risk factor for ectopic pregnancy? use of IUD for contraception multiple gestation pregnancy use of oral contraceptives high number of pregnancies

use of IUD for contraception

The nurse is is giving discharge instructions to a client who experienced a complete spontaneous abortion. Which question should the nurse prioritize at this time?

"Do you have someone to talk to, or may I give you the names and numbers for some possible grief counselors?" When a woman has a spontaneous abortion, or miscarriage, one important consideration is the emotional needs of the woman once she is home. She may not want to talk about the loss for a period of time, but the nurse needs to determine her support system for the future. Asking the woman if she is "going to try again" is an inappropriate question for the nurse to ask and diminishes the experience of having a spontaneous abortion. It would be inappropriate to point out the woman is not the only one to have this experience or to offer ways to prevent it from happening in the future. The woman needs to deal with this situation first before moving on to a possible "next" time.

The nurse is educating a pregnant client who has gestational diabetes. Which of the following statements should the nurse make to the client? Select all that apply.

"Gestational diabetes mellitus is hyperglycemia that develops during pregnancy, usually around the 24th week of gestation." "Gestational diabetes mellitus increases the risk of preeclampsia, caesarean delivery, and fetal macrosomia." "Gestational diabetes mellitus increases the risk of hypertension." "Screening is not needed if the client is younger than 25 years old and has no personal history of abnormal glucose metabolism." Gestational diabetes increases risks in mother and infant. It increases the risks of preeclampsia, caesarian delivery, and fetal macrosomia and the risk of hypertension and diabetes after pregnancy. As many as 70% of women who develop gestational diabetes will develop type 2 diabetes within 10 years after pregnancy. Usually, gestational diabetes disappears after the infant is born.

The nurse is teaching a client who is diagnosed with preeclampsia how to monitor her condition. The nurse determines the client needs more instruction after making which statement? -"If I have changes in my vision, I will lie down and rest." -"I will weigh myself every morning after voiding before breakfast." -"I will count my baby's movements after each meal." -"If I have a severe headache, I'll call the clinic."

"If I have changes in my vision, I will lie down and rest." Changes in the visual field may indicate the client has moved from preeclampsia to severe preeclampsia and is at risk for developing a seizure due to changes in cerebral blood flow. The client would require immediate assessment and intervention. Gaining weight is not necessarily a sign of worsening preeclampsia. The other choices are instructions which the client may be given to follow.

Select the statement by the pregnant woman that indicates the need for more teaching about preeclampsia. a) "If I have changes in my vision, I will lie down and rest." b) "I will count my baby's movements twice a day." c) "If I have a slight headache I'll take Tylenol and call if unrelieved." d) "I will weigh myself every morning after voiding before breakfast."

"If I have changes in my vision, I will lie down and rest." Explanation: Changes in the visual field may indicate the patient has moved from preeclampsia to severe preeclampisa and is at risk for developing a seizure due to changes in cerebral blood flow. The patient would require immediate assessment and intervention. Options B, C, and D would not indicate that more teaching about preeclampsia is indicated.

The nurse's discharge teaching for a client with acute diabetic complications is discussing managing hypoglycemia. The nurse should include which of the following in the client teaching?

"Keep a package of skittles in your pocket at all times" The client should be encouraged to eat 15 g of readily absorbable carbohydrate if blood glucose is ,70 mg/day (see Box 19.7). 16 Skittles candies equal 15 g of readily absorbable carbohydrate, therefore, the nurse is correct to recommend that the client carry this in his pocket at all times. Pure sugars are better than items like chocolate candy bars, which contain fat that slows the gastric emptying time and delays the rise in blood glucose. Lean protein has little or no carbohydrate content. This will not increase blood sugars quickly enough to resolve the hypoglycemia. The client should be encourage to eat a meal within an hour of the hypoglycemic episode, not 4 hours.

A nurse has been assigned to assess a pregnant client for abruptio placenta. For which classic manifestation of this condition should the nurse assess?

"knife-like" abdominal pain with vaginal bleeding

A nurse has been assigned to assess a pregnant client for placental abruption (abruptio placentae). For which classic manifestation of this condition should the nurse assess? generalized vasospasm painless bright red vaginal bleeding "knife-like" abdominal pain with vaginal bleeding increased fetal movement

"knife-like" abdominal pain with vaginal bleeding

A postpartum mother has the following lab data recorded: RH negative and rubella titer is positive. What is the appropriate nursing intervention? a) Administer RhoGam within 72 hours. b) Assess the RH of the baby. c) Administer rubella vaccine before discharge. d) Assess the rubella of the baby

...

pitting scale

1+ is a 2-mm depression that disappears rapidly; 2+ is a 4-mm depression that disappears in 10 to 15 seconds; 3+ is a 6-mm depression that lasts more than one minute; and 4+ is an 8-mm depression that lasts 2 to 3 minutes.

A student nurse asks the instructor what percentage of clinically recognized pregnancies end in miscarriages during the first trimester. Which response from the nurse is the most accurate?

15% to 20%

The client arrives in the office and reports that her feet and legs swelling. During a client evaluation, the nurse notes that she can elicit a 4-mm skin depression that disappears in 10 to 15 seconds. The nurse correctly documents this finding as:

2+ pitting edema

; grade 2 (moderate)

? moderate bleeding (1,000 to 1,500 mL), 20% to 50% separation, continuous abdominal pain, mild shock, normal maternal blood pressure, maternal tachycardia;

A primipara at 36 weeks' gestation is being monitored in the prenatal clinic for risk of preeclampsia. Which sign or symptom should the nurse prioritize?

A dipstick value of 2+ for protein.

The nurse is caring for a pregnant client who indicates that she is fond of meat, works with children, and has a pet cat. Which instructions should the nurse give this client to prevent toxoplasmosis? SATA A. Eat meat cooked at 160 F (71 C) B. Avoid cleaning the cat's litter box C. Keep the cat outdoors at all times D. Avoid contact with children when they have a cold E. Avoid outdoor activities such as gardening

A. Eat meat cooked at 160 F (71 C) B. Avoid cleaning the cat's litter box E. Avoid outdoor activities such as gardening

A client reporting she recently had a positive pregnancy test has reported to the emergency department stating one-sided lower abdominal pain. The health care provider has prescribed a series of tests. Which test will provide the most definitive confirmation of an ectopic pregnancy? Qualitative human chorionic gonadotropin (hCG) test Quantitative human chorionic gonadotropin (hCG) test Pelvic examination Abdominal ultrasound

Abdominal ultrasound

A client reporting she recently had a positive pregnancy test has reported to the emergency department stating one-sided lower abdominal pain. The health care provider has prescribed a series of tests. Which test will provide the most definitive confirmation of an ectopic pregnancy?

Abdominal ultrasound An ectopic pregnancy refers to the implantation of the fertilized egg in a location other than the uterus. Potential sites include the cervix, uterus, abdomen, and fallopian tubes. The confirmation of the ectopic pregnancy can be made by an ultrasound, which would confirm that there was no uterine pregnancy. A quantitative hCG level may be completed in the diagnostic plan. hCG levels in an ectopic pregnancy are traditionally reduced. While this would be an indication, it would not provide a positive confirmation. The qualitative hCG test would provide evidence of a pregnancy, but not the location of the pregnancy. A pelvic exam would be included in the diagnostic plan of care. It would likely show an enlarged uterus and cause potential discomfort to the client but would not be a definitive finding.

A nurse is caring for a client with hyperemesis gravidarum. Which nursing action is the priority for this client?

Administer IV NS with vitamins and electrolytes

A primigravida 28-year-old client is noted to have Rh negative blood and her husband is noted to be Rh positive. The nurse should prepare to administer RhoGAM after which diagnostic procedure? Contraction test Nonstress test Biophysical profile Amniocentesis

Amniocentesis

A primigravida 28-year-old client is noted to have Rh negative blood and her husband is noted to be Rh positive. The nurse should prepare to administer RhoGAM after which diagnostic procedure? Amniocentesis Biophysical profile Nonstress test Contraction test

Amniocentesis

A primigravida 28-year-old client is noted to have Rh negative blood and her husband is noted to be Rh positive. The nurse should prepare to administer RhoGAM after which diagnostic procedure? -Contraction test -Nonstress test -Biophysical profile -Amniocentesis

Amniocentesis Amniocentesis is a procedure requiring a needle to enter into the amniotic sac. There is a risk of mixing of the fetal and maternal blood which could result in blood incompatibility. A contraction test, a nonstress test, and biophysical profile are not invasive, so there would be no indication for Rho(D) immune globulin to be administered. Page 693

A pregnant client with hyperemesis gravidarum need advice on how to minimize nausea and vomiting. Which instruction should the nurse give this client? A. Lie down or recline for at least 2 hours after eating B. Avoid dry crackers, toast, and soda C. Eat small, frequent meals throughout the day D. Decrease intake of carbonated beverages

C. Eat small, frequent meals throughout the day

The nurse is caring for a pregnant client with fallopian tube rupture. Which intervention is the priority for this client? A. Monitor the client's beta-hCG level B. Monitor the mass with transvaginal ultrasound C. Monitor the client's vital signs and bleeding D. Monitor the fetal heart rate (FHR)

C. Monitoring the client's vital signs and bleeding

Which should the nurse identify as a risk associated with anemia during pregnancy? A. Newborn with heart problems B. Fetal asphyxia C. Preterm birth D. Newborn with an enlarged liver

C. Preterm birth

A pregnant client with hyperemesis gravidarum needs advice on how to minimize nausea and vomiting. Which instruction should the nurse give this client?

Eat small frequent meals throughout the day The nurse should instruct the client with hyperemesis gravidarum to eat small, frequent meals throughout the day to minimize nausea and vomiting. The nurse should also instruct the client to avoid lying down or reclining for at least 2 hours after eating and to increase the intake of carbonated beverages. The nurse should instruct the client to try foods that settle the stomach such as dry crackers, toast, or soda.

Current research indicates that supplementation with what before pregnancy may reduce the risk of placental abruption?

Folic Acid

The nurse is caring for a pregnant client with fallopian tube rupture. Which intervention is the priority for this client? Monitor the client's beta-hCG level. Monitor the mass with transvaginal ultrasound. Monitor the client's vital signs and bleeding. Monitor the fetal heart rate (FHR).

Monitor the client's vital signs and bleeding.

Gestational Trophoblastic Disease (GTD)

Spectrum of neoplastic disorders that originate in the human placenta

A young mother delivers twin boys who shared the same placenta. What serious complication are they at risk for? a) ABO incompatability b) Twin-to-twin transfusion syndrome (TTTS) c) TORCH syndrome d) HELLP syndrome

Twin-to-twin transfusion syndrome (TTTS) Explanation: When twins share a placenta, a serious condition called twin-to-twin transfusion syndrome (TTTS) can occur.

gestational

___________________ hypertension is characterized by hypertension without proteinuria after 20 weeks' gestation and a return of the blood pressure to normal postpartum.

A nurse is assessing pregnant clients for the risk of placenta previa. Which client faces the greatest risk for this condition?

a client who had a myomectomy to remove fibroids

A nurse is caring for a client who just experienced a miscarriage in her first trimester. When asked by the client why this happened, which is the best response from the nurse? abnormal fetal development rejection of the embryo through an immune response implantation abnormality lack of sufficient progesterone produced by the corpus luteum

abnormal fetal development

A woman with severe preeclampsia is receiving magnesium sulfate. The woman's serum magnesium level is 9.0 mEq/L. Which finding would the nurse most likely note?

diminished reflexes Diminished or absent reflexes occur when a client develops magnesium toxicity, serum levels greater than 8.0 mEq/L. Elevated liver enzymes are unrelated to magnesium toxicity and may indicate the development of HELLP syndrome. The onset of seizure activity indicates eclampsia. A serum magnesium level of 6.5 mEq/L would fall within the therapeutic range of 4 to 7 mEq/L.

A nurse is monitoring a client with PROM who is in labor and observes meconium in the amniotic fluid. What does the observation of meconium indicate?

fetal distress related to hypoxia When meconium is present in the amniotic fluid, it typically indicates fetal distress related to hypoxia. Meconium stains the fluid yellow to greenish brown, depending on the amount present. A decreased amount of amniotic fluid reduces the cushioning effect, thereby making cord compression a possibility. A foul odor of amniotic fluid indicates infection. Meconium in the amniotic fluid does not indicate CNS involvement.

A pregnant woman is diagnosed with abruptio placentae. When reviewing the woman's physical assessment in her medical record, which finding would the nurse expect?

firm, rigid uterus on palpation

A woman at 10 weeks' gestation comes to the clinic for an evaluation. Which assessment finding should the nurse prioritize? report of frequent mild nausea blood pressure of 120/84 mm Hg history of bright red spotting 6 weeks ago fundal height measurement of 18 cm

fundal height measurement of 18 cm Explanation: A fundal height of 18 cm is larger than expected and should be further investigated for gestational trophoblastic disease (hydatidiform mole). One of the presenting signs is the uterus being larger than expected for date. Mild nausea would be a normal finding at 10 weeks' gestation. Blood pressure of 120/84 mm Hg would not be associated with hydatidiform mole and depending on the woman's baseline blood pressure may be within acceptable parameters for her. Bright red spotting might suggest a spontaneous abortion (miscarriage).

A novice nurse asks to be assigned to the least complex antepartum client. Which condition would necessitate the least complex care requirements?

gestational hypertension Hypertensive disorders represent the most common complication of pregnancy. Gestational hypertension is elevated blood pressure without proteinuria, other signs of preeclampsia, or preexisting hypertension. Abruptio placenta (separation of the placenta from the uterine wall), placenta previa (placenta covering the cervical os), and preeclampsia are high-risk, potentially life-threatening conditions for the fetus and mother during labor and birth.

A novice nurse asks to be assigned to the least complex antepartum client. Which condition would necessitate the least complex care requirements? -preeclampsia -placental abruption (abruptio placentae) -placenta previa -gestational hypertension

gestational hypertension Hypertensive disorders represent the most common complication of pregnancy. Gestational hypertension is elevated blood pressure without proteinuria, other signs of preeclampsia, or preexisting hypertension. Placental abruption (abruptio placentae), a separation of the placenta from the uterine wall; placenta previa (placenta covering the cervical os); and preeclampsia are high-risk, potentially life-threatening conditions for the fetus and mother during labor and birth.

A pregnant client has been admitted with reports of brownish vaginal bleeding. On examination, there is an elevated human chorionic gonadotropin (hCG) level, absent fetal heart sounds, and a discrepancy between the uterine size and the gestational age. The nurse interprets these findings to suggest which condition?

gestational trophoblastic disease Explanation: The client is most likely experiencing gestational trophoblastic disease, or a molar pregnancy. In gestational trophoblastic disease, there is an abnormal proliferation and eventual degeneration of the trophoblastic villi. The signs and symptoms of molar pregnancy include brownish vaginal bleeding, elevated hCG levels, discrepancy between the uterine size and the gestational age, and absent fetal heart sounds. Placental abruption is characterized by premature separation of the placenta. Ectopic pregnancy is a condition where there is implantation of the blastocyst outside the uterus. In placenta previa, the placental attachment is at the lower uterine segment.

A pregnant client has been admitted with reports of brownish vaginal bleeding. On examination, there is an elevated human chorionic gonadotropin (hCG) level, absent fetal heart sounds, and a discrepancy between the uterine size and the gestational age. The nurse interprets these findings to suggest which condition?

gestational trophoblastic disease The client is most likely experiencing gestational trophoblastic disease, or a molar pregnancy. In gestational trophoblastic disease, there is an abnormal proliferation and eventual degeneration of the trophoblastic villi. The signs and symptoms of molar pregnancy include brownish vaginal bleeding, elevated hCG levels, discrepancy between the uterine size and the gestational age, and absent fetal heart sounds. Placental abruption is characterized by premature separation of the placenta. Ectopic pregnancy is a condition where there is implantation of the blastocyst outside the uterus. In placenta previa, the placental attachment is at the lower uterine segment.

A nurse is taking a history of a client at 5 weeks' gestation in the prenatal clinic; however, the client is reporting dark brown vaginal discharge, nausea, and vomiting. Which diagnosis should the nurse suspect?

gestational trophoblastic disease This client has risk factors of a "molar" pregnancy: nausea and vomiting at an early gestational week and dark brown vaginal discharge. The early nausea/vomiting can be due to a high hCG level, which is a sign of gestational trophoblastic disease. There is only one sign/symptom of hyperemesis gravidarum. Placenta previa is marked by bright red bleeding and tends to happen later in gestation. There are no data to support any psychosis at this stage.

A client has been admitted with abruptio placentae. She has lost 1,200 mL of blood, is normotensive, and ultrasound indicates approximately 30% separation. The nurse documents this as which classification of abruptio placentae?

grade 2

Risk factors for type 2 diabetes include all of the following EXCEPT:

greater than 35 years old Risk factors for type 2 diabetes are a family history of diabetes, physical inactivity or exercising less than three times per week, impaired glucose metabolism, a history of gestational diabetes, and race/ethnicity. African-Americans, Hispanics/Latinos, Native Americans, Asian Americans, and Pacific Islanders are high-risk ethnic groups. Being greater than 45 years old is considered a risk factor for type 2 diabetes.

A woman at 8 weeks' gestation is admitted for ectopic pregnancy. She is asking why this has occurred. The nurse knows that which factor is a known risk factor for ectopic pregnancy?

history of endometriosis Explanation: The nurse needs to complete a full history of the client to determine if she had any other risk factors for an ectopic pregnancy. Adhesions, scarring, and narrowing of the tubal lumen may block the zygote's progress to the uterus. Any condition or surgical procedure that can injure a fallopian tube increases the risk. Examples include salpingitis, infection of the fallopian tube, endometriosis, history of prior ectopic pregnancy, any type of tubal surgery, congenital malformation of the tube, and multiple abortions (elective terminations of pregnancy). Conditions that inhibit peristalsis of the tube can result in tubal pregnancy. A high number of pregnancies, multiple gestation pregnancy, and the use of oral contraceptives are not known risk factors for ectopic pregnancy.

A nurse is providing care to a client who has been diagnosed with a common benign form of gestational trophoblastic disease. The nurse identifies this as:

hydatidiform mole. Gestational trophoblastic disease comprises a spectrum of neoplastic disorders that originate in the placenta. The two most common types are hydatidiform mole (partial or complete) and choriocarcinoma. Ectopic pregnancy, placenta accreta, and hydramnios fall into different categories of potential pregnancy complications.

A 25-year-old client at 22 weeks' gestation is noted to have proteinuria and dependent edema on her routine prenatal visit. Which additional assessment should the nurse prioritize and convey to the RN or health care provider? initial BP 120/80mm Hg; current BP 130/88 mm Hg initial BP 140/85 mm Hg; current BP 130/80 mm Hg initial BP 110/60 mm Hg; current BP 112/86 mm Hg initial BP 100/70 mm Hg; current BP 140/90 mm Hg

initial BP 100/70 mm Hg; current BP 140/90 mm Hg

Which measure would the nurse include in the plan of care for a woman with premature rupture of membranes if her fetus's lungs are mature?

labor induction

The nurse is caring for a pregnant client with fallopian tube rupture. Which intervention is the priority for this client?

monitor client's vital signs and bleeding

A nurse is providing care to a multiparous client. The client has a history of cesarean births. The nurse anticipates the need to closely monitor the client for which condition? placenta accreta preeclampsia oligohydramnios placenta abruption

placenta accreta

A woman in labor has sharp fundal pain accompanied by slight vaginal bleeding. What would be the most likely cause of these symptoms?

premature separation of the placenta

A 24-year-old client presents in labor. The nurse notes there is an order to administer RhoGAM after the birth of her infant. When asked by the client the reason for this injection, which reason should the nurse point out?

prevent maternal D antibody formation. Because RhoGAM contains passive antibodies, the solution will prevent the woman from forming long-lasting antibodies which may harm a future fetus. The administration of RhoGAM does not promote the formation of maternal D antibodies; it does not stimulate maternal D immune antigens or prevent fetal Rh blood formation.

A 24-year-old client presents in labor. The nurse notes there is an order to administer Rho(D) immune globulin after the birth of her infant. When asked by the client the reason for this injection, which reason should the nurse point out?

prevent maternal D antibody formation. Explanation: Because Rho(D) immune globulin contains passive antibodies, the solution will prevent the woman from forming long-lasting antibodies which may harm a future fetus. The administration of Rho(D) immune globulin does not promote the formation of maternal D antibodies; it does not stimulate maternal D immune antigens or prevent fetal Rh blood formation.

A 24-year-old client presents in labor. The nurse notes there is an order to administer Rho(D) immune globulin after the birth of her infant. When asked by the client the reason for this injection, which reason should the nurse point out? promote maternal D antibody formation. prevent maternal D antibody formation. stimulate maternal D immune antigens. prevent fetal Rh blood formation.

prevent maternal D antibody formation. Explanation: Because Rho(D) immune globulin contains passive antibodies, the solution will prevent the woman from forming long-lasting antibodies which may harm a future fetus. The administration of Rho(D) immune globulin does not promote the formation of maternal D antibodies; it does not stimulate maternal D immune antigens or prevent fetal Rh blood formation.

A 24-year-old client presents in labor. The nurse notes there is an order to administer Rho(D) immune globulin after the birth of her infant. When asked by the client the reason for this injection, which reason should the nurse point out?

prevent maternal D antibody forming Because Rho(D) immune globulin contains passive antibodies, the solution will prevent the woman from forming long-lasting antibodies which may harm a future fetus. The administration of Rho(D) immune globulin does not promote the formation of maternal D antibodies; it does not stimulate maternal D immune antigens or prevent fetal Rh blood formation.

A client is admitted to labor and birth for management of severe preeclampsia. An IV infusion of magnesium sulfate is started. What is the primary goal for magnesium sulfate therapy?

prevent maternal seizures The primary therapy goal for any preeclamptic client is to prevent maternal seizures. Use of magnesium sulfate is the drug therapy of choice for severe preeclampsia and is only used to manage and attempt to prevent progression to eclampsia. Magnesium sulfate therapy does not have as its primary goal a decrease in blood pressure, a decrease in protein in the urine, nor the reversal of edema.

The nurse is caring for a client with preeclampsia and understands the need to auscultate this client's lung sounds every 2 hours. Why would the nurse do this? pulmonary atelectasis pulmonary edema pulmonary hypertension pulmonary emboli

pulmonary edema

A client with preeclampsia is receiving magnesium sulfate. Which nursing assessment should be ongoing while the medication is being administered?

respiratory rate The level of magnesium in therapeutic range is 4 to 8 mg/dL. If magnesium toxicity occurs, one sign in the client will be a decrease in the respiratory rate and a potential respiratory arrest. Respiratory rate will be monitored when on this medication. The client's hemoglobin and ability to sleep are not factors for ongoing assessments for the client on magnesium sulfate. Urinary output is measured hourly on the preeclamptic client receiving magnesium sulfate, but urine protein is not an ongoing assessment

The nurse is appraising the medical record of a pregnant client who is resting in a darkened room and receiving betamethasone and magnesium sulfate. The nurse recognizes the client is being treated for which condition? gestational diabetes gestational hypertension severe preeclampsia postterm pregnancy

severe preeclampsia

The nurse is educating a 65-year-old client before his discharge on how to prevent complications of diabetes. Which of the following factors that may accelerate the deleterious effects of diabetes should the nurse tell the client to avoid? Select all that apply.

smoking obesity high blood pressure saturated fat intake Attention should be paid to factors that may accelerate the deleterious effects of diabetes. These include smoking, high dietary saturated fats, elevated cholesterol level, obesity, high blood pressure, and lack of regular exercise. Regular exercise improves the blood glucose level and reduces insulin resistance.

A nurse is describing the use of RhoGAM as the therapy of choice for isoimmunization in Rh-negative women and for other conditions to a group of nurses working at the women's health clinic. The nurse determines that additional teaching is needed when the group identifies which situation as an indication for RhoGAM?

sti

A nurse is caring for a pregnant client admitted with mild preeclampsia. Which assessment finding should the nurse prioritize?

urine output of less than 15ml/hr Severe preeclampsia may develop suddenly and bring with it high blood pressure of more than 160/110 mm Hg, proteinuria of more than 500 mg in 24 hours, oliguria of less than 15 ml/hr, cerebral and visual symptoms, and rapid weight gain. Mild facial edema or hand edema occurs with mild preeclampsia. A urinary output of 15 ml/hr would result in an output of 360 ml/24 hours, which would be below the recommended range and should be reported. Ankle edema of 1+ could be related to regular pregnancy and not necessarily just severe preeclampsia. A finding of 3+ to 4+ pitting edema would be more alarming and require intervention.

A pregnant women calls the clinic to report a small amount of painless vaginal bleeding. What response by the nurse is best?

" Please come in now for an evaluation by your health care provider." Explanation: Bleeding during pregnancy is always a deviation from normal and should be evaluated carefully. It may be life-threatening or it may be something that is not a threat to the mother and/or fetus. Regardless, it needs to be evaluated quickly and carefully. Telling the client it may be harmless is a reassuring statement, but does not suggest the need for urgent evaluation. Having the mother lay on her left side and drink water is indicated for cramping.

A 35-year-old client is seen for her 2-week postoperative appointment after a suction curettage was performed to evacuate a hydatidiform mole. The nurse explains that the human chorionic gonadotropin (hCG) levels will be reviewed every 2 weeks and teaches about the need for reliable contraception for the next 6 months to a year. The client states, "I'm 35 already. Why do I have to wait that long to get pregnant again?" What is the nurse's best response?

"A contraceptive is used so that a positive pregnancy test resulting from a new pregnancy will not be confused with the increased level of hCG that occurs with a developing malignancy." Because of the risk of choriocarcinoma, the woman receives extensive treatment. Therapy includes baseline chest X-ray to detect lung metastasis physical exam including pelvic exam. Serum B-hCG levels weekly until negative results are obtained three consecutive times, then monthly for 6 to 12 months. The woman is cautioned to avoid pregnancy during this time because the increasing B-hCG levels associated with pregnancy would cause confusion as to whether cancer had developed. If after a year B-hCG seruim titers are within normal levels, a normal pregnancy can be achieved.

A 35-year-old client is seen for her 2-week postoperative appointment after a suction curettage was performed to evacuate a hydatidiform mole. The nurse explains that the human chorionic gonadotropin (hCG) levels will be reviewed every 2 weeks and teaches about the need for reliable contraception for the next 6 months to a year. The client states, "I'm 35 already. Why do I have to wait that long to get pregnant again?" What is the nurse's best response?

"A contraceptive is used so that a positive pregnancy test resulting from a new pregnancy will not be confused with the increased level of hCG that occurs with a developing malignancy." Because of the risk of choriocarcinoma, the woman receives extensive treatment. Therapy includes baseline chest X-ray to detect lung metastasis, plus a physical exam (including a pelvic exam). Serum B-hCG levels weekly until negative results are obtained three consecutive times, then monthly for 6 to 12 months. The woman is cautioned to avoid pregnancy during this time because the increasing B-hCG levels associated with pregnancy would cause confusion as to whether cancer had developed. If after a year B-hCG serum titers are within normal levels, a normal pregnancy can be achieved.

A 35-year-old client is seen for her 2-week postoperative appointment after a suction curettage was performed to evacuate a hydatidiform mole. The nurse explains that the human chorionic gonadotropin (hCG) levels will be reviewed every 2 weeks and teaches about the need for reliable contraception for the next 6 months to a year. The client states, "I'm 35 already. Why do I have to wait that long to get pregnant again?" What is the nurse's best response? -"After a curettage procedure, it is recommended that you give your body some time to build up its stores." -"A contraceptive is used so that a positive pregnancy test resulting from a new pregnancy will not be confused with the increased level of hCG that occurs with a developing malignancy." -"Since you are at the end of your reproductive years, it is suggested that you don't try to have any more pregnancies." -"You may need chemotherapy, so we don't want to risk pregnancy."

"A contraceptive is used so that a positive pregnancy test resulting from a new pregnancy will not be confused with the increased level of hCG that occurs with a developing malignancy." Because of the risk of choriocarcinoma, the woman receives extensive treatment. Therapy includes baseline chest X-ray to detect lung metastasis, plus a physical exam (including a pelvic exam). Serum B-hCG levels weekly until negative results are obtained three consecutive times, then monthly for 6 to 12 months. The woman is cautioned to avoid pregnancy during this time because the increasing B-hCG levels associated with pregnancy would cause confusion as to whether cancer had developed. If after a year B-hCG serum titers are within normal levels, a normal pregnancy can be achieved.

A 35-year-old client is seen for her 2-week postoperative appointment after a suction curettage was performed to evacuate a hydatidiform mole. The nurse explains that the human chorionic gonadotropin (hCG) levels will be reviewed every 2 weeks and teaches about the need for reliable contraception for the next 6 months to a year. The client states, "I'm 35 already. Why do I have to wait that long to get pregnant again?" What is the nurse's best response?

"A contraceptive is used so that a positive pregnancy test resulting from a new pregnancy will not be confused with the increased level of hCG that occurs with a developing malignancy." Explanation: Because of the risk of choriocarcinoma, the woman receives extensive treatment. Therapy includes baseline chest X-ray to detect lung metastasis, plus a physical exam (including a pelvic exam). Serum B-hCG levels weekly until negative results are obtained three consecutive times, then monthly for 6 to 12 months. The woman is cautioned to avoid pregnancy during this time because the increasing B-hCG levels associated with pregnancy would cause confusion as to whether cancer had developed. If after a year B-hCG serum titers are within normal levels, a normal pregnancy can be achieved.

Which statement made by a new nurse indicates additional teaching is needed on the topic of hyperbilirubinemia (physiologic jaundice) in newborns? -"Physiologic jaundice usually begins in the first week after birth." -"Placing the infant in direct sunlight for short periods helps in eliminating the bilirubin." -"Breastfed babies need supplements of glucose water to help lower bilirubin levels." -"The problem is a result of the shortened lifespan of the newborn's red blood cells (RBCs)."

"Breastfed babies need supplements of glucose water to help lower bilirubin levels." Physiologic jaundice (hyperbilirubinemia) is characterized by a yellowish skin, mucous membranes, and sclera that occurs within the first 3 days of life. Physiologic jaundice is caused by accelerated destruction of fetal RBCs that have a shortened life span (80 days compared with the adult 120 days). Normally the liver removes bilirubin (the by-product of RBC destruction) from the blood and changes it into a form that can be excreted. As the red blood cell breakdown continues at a fast pace, the newborn's liver cannot keep up with bilirubin removal. Thus, bilirubin accumulates in the blood, causing the characteristic signs of physiologic jaundice. Expose the newborn to natural sunlight for short periods of time throughout the day to help oxidize the bilirubin deposits on the skin. Glucose water supplementation should be avoided since it hinders elimination.

A woman of 16 weeks' gestation telephones you because she has passed some "berry-like" blood clots and now has continued dark brown vaginal bleeding. Which of the following would you instruct the woman to do? a) "Maintain bed rest and count the number of perineal pads used." b) "Come to the health care facility if uterine contractions begin." c) "Continue normal activity, but take your pulse every hour." d) "Come to the health facility with any vaginal material passed."

"Come to the health facility with any vaginal material passed." Correct Explanation: This is a typical time in pregnancy for gestational trophoblastic disease to present. Asking the woman to bring any material passed vaginally would be important so it can be assessed for this.

Which is the best question the nurse can ask a woman who is leaving the hospital after experiencing a complete spontaneous abortion?

"Do you have someone to talk to, or may I give you the names and numbers for some possible grief counselors?" When a woman has a spontaneous abortion one important consideration is the emotional needs of the woman once she is home. She may not want to talk about the loss for a period of time, but the nurse needs to determine her support system for the future. Asking the woman if she is "going to try again" is an inappropriate question for the nurse to ask and diminishes the experience of having a spontaneous abortion. Giving the woman statistical information on spontaneous abortions is not appropriate when this client needs support and caring concern. Offering to give the client resources to aid in smoking cessation is not addressed in the scenario, so this is an inappropriate response.

A client at 11 weeks' gestation experiences pregnancy loss. The client asks the nurse if the bleeding and cramping that occurred during the miscarriage were caused by working long hours in a stressful environment. What is the most appropriate response from the nurse? -"Your spontaneous bleeding is not work-related." -"It is hard to know why a woman bleeds during early pregnancy." -"I can understand your need to find an answer to what caused this. Let's talk about this further." -"Something was wrong with the fetus."

"I can understand your need to find an answer to what caused this. Let's talk about this further." Talking with the client may assist her to explore her feelings. She and her family may search for a cause for a spontaneous early bleeding so they can plan for future pregnancies. Even with modern technology and medical advances, however, a direct cause cannot usually be determined.

A client at 11 weeks' gestation experiences pregnancy loss. The client asks the nurse if the bleeding and cramping that occurred during the miscarriage were caused by working long hours in a stressful environment. What is the most appropriate response from the nurse? "Your spontaneous bleeding is not work-related." "It is hard to know why a woman bleeds during early pregnancy." "I can understand your need to find an answer to what caused this. Let's talk about this further." "Something was wrong with the fetus."

"I can understand your need to find an answer to what caused this. Let's talk about this further." Explanation: Talking with the client may assist her to explore her feelings. She and her family may search for a cause for a spontaneous early bleeding so they can plan for future pregnancies. Even with modern technology and medical advances, however, a direct cause cannot usually be determined.

A nurse is providing discharge teaching for a pregnant client with preeclampsia who will be managed at home on bedrest. The nurse determines that the teaching was successful based on which client statement?

"I need to drink at least 8 glasses of water a day" A client with mild elevation in blood pressure may be placed on bed rest at home. The client is encouraged to rest as much as possible in the lateral recumbent position to improve uteroplacental blood flow, reduce blood pressure, and promote diuresis. The client will be asked to monitor her blood pressure daily (every 4 to 6 hours while awake) and report any increased readings; she will be counseled on taking daily kick counts and report any decrease in fetal movements; and she will see her health care provider weekly for ongoing evaluations. The client should record daily fetal movement counts, and if there is any decrease in movement, she needs to be evaluated by her health care provider that day. A balanced, nutritional diet with no sodium restriction is advised. In addition, the client is encouraged to drink six to eight 8-oz glasses (1.5 to 2 liters) of water daily.

The nurse has just completed teaching a client with diabetes about dietary restrictions necessary to control diabetes complications. The nurse recognizes that the teaching has been successful when the client states which of the following?

"I will eat a diet rich in fruits and vegetables" For people at risk of cardiovascular disease, a diet rich in fruit, vegetables, whole grains, and nuts may lower the risk. People with chronic kidney disease should reduce their protein intake. Hemoglobin A1c must be maintained as close to normal as possible.

A pregnant client with preterm premature rupture of the membranes is being discharged home. A nurse is preparing the client's discharge teaching plan. Which instructions would the nurse include? Select all that apply. "Gently massage your breasts at least once each day." "Check your temperature each day, reporting any increase immediately." "Be sure to perform fetal kick counts about once every 3 days." "If you notice your belly starting to tighten, call your health care provider." "Take tub baths instead of showers." SUBMIT ANSWER

"If you notice your belly starting to tighten, call your health care provider." "Check your temperature each day, reporting any increase immediately."

A pregnant client with preterm premature rupture of the membranes is being discharged home. A nurse is preparing the client's discharge teaching plan. Which instructions would the nurse include? Select all that apply.

"If you notice your belly starting to tighten, call your health care provider." "Check your temperature each day, reporting any increase immediately." Instructions should include monitoring the baby's activity by performing fetal kick counts daily; checking temperature daily and reporting any temperature increases to the health care provider; watching for signs related to the beginning of labor and reporting any tightening of the abdomen or contractions; avoiding any touching or manipulating of the breasts, which could stimulate labor; and taking showers for daily hygiene needs and avoiding sitting in a tub bath.

After a regular prenatal visit, a pregnant client asks the nurse to describe the differences between abruptio placentae and placenta previa. Which statement should the nurse include in the teaching? "Placenta previa causes painful, dark red bleeding during pregnancy due to an abnormally implanted placentae that is too close to or covers the cervix; abruptio placenta is associated with bright red painless bleeding caused by premature separation of the placenta from the wall of the uterus before the end of labor." "Placenta previa causes painless, bright red bleeding during pregnancy due to an abnormally implanted placenta that is too close to or covers the cervix; abruptio placentae is associated with dark red painful bleeding caused by premature separation of the placenta from the wall of the uterus before the end of labor." "Placenta previa causes painless, bright red bleeding during pregnancy due to an abnormally implanted placenta that is too close to or covers the fundus; abruptio placentae is associated with dark red painful bleeding caused by premature separation of the placenta from the wall of the uterus before the end of labor." "Placenta previa causes painful, dark red bleeding during pregnancy due to an abnormally implanted placenta that is too close to or covers the fundus; abruptio placentae is associated with right red painless bleeding caused by premature separation of the placenta from the wall of the uterus before the end of labor."

"Placenta previa causes painful, dark red bleeding during pregnancy due to an abnormally implanted placentae that is too close to or covers the cervix; abruptio placenta is associated with bright red painless bleeding caused by premature separation of the placenta from the wall of the uterus before the end of labor."

After a regular prenatal visit, a pregnant client asks the nurse to describe the differences between abruptio placenta and placenta previa. Which statement should the nurse include in the teaching?

"Placenta previa causes painless, bright red bleeding during pregnancy due to an abnormally implanted placenta that is too close to or covers the cervix; abruptio placenta is associated with dark red painful bleeding caused by premature separation of the placenta from the wall of the uterus before the end of labor." Placenta previa is a condition of pregnancy in which the placenta is implanted abnormally in the lower part of the uterus and is the most common cause of painless bright red bleeding in the third trimester. Abruptio placenta is the premature separation of a normally implanted placenta that pulls away from the wall of the uterus either during pregnancy or before the end of labor.

The nurse is teaching a prenatal class on potential problems during pregnancy to a group of expectant parents. The risk factors for placental abruption (abruptio placentae) are discussed. Which comment validates accurate learning by the parents?

"Placental abruption is quite painful and I will need to let the doctor know if I begin to have abdominal pain" Placental abruption (abruptio placentae) occurs when there is a spontaneous separation of the placenta from the uterine wall. It can occur anywhere on the placenta and will cause painful, dark red vaginal bleeding. If the abruption is small, the ob/gyn will try to deliver the fetus vaginally. But if severe bleeding occurs or there is fetal distress, a cesarean birth will be performed. Women older than 35 are also at higher risk for developing placental abruption.

The nurse is teaching a prenatal class on potential problems during pregnancy to a group of expectant parents. The risk factors for placental abruption (abruptio placentae) are discussed. Which comment validates accurate learning by the parents?

"Placental abruption is quite painful and I will need to let the doctor know if I begin to have abdominal pain." Explanation: Placental abruption (abruptio placentae) occurs when there is a spontaneous separation of the placenta from the uterine wall. It can occur anywhere on the placenta and will cause painful, dark red vaginal bleeding. If the abruption is small, the ob/gyn will try to deliver the fetus vaginally. But if severe bleeding occurs or there is fetal distress, a cesarean birth will be performed. Women older than 35 are also at higher risk for developing placental abruption

The nurse is teaching a prenatal class on potential problems during pregnancy to a group of expectant parents. The risk factors for placental abruption (abruptio placentae) are discussed. Which comment validates accurate learning by the parents? "I need a cesarean section if I develop this problem." "If I develop this complication, I will have bright red vaginal bleeding," "Placental abruption is quite painful and I will need to let the doctor know if I begin to have abdominal pain." "Since I am over 30, I run a much higher risk of developing this problem."

"Placental abruption is quite painful and I will need to let the doctor know if I begin to have abdominal pain." Explanation: Placental abruption (abruptio placentae) occurs when there is a spontaneous separation of the placenta from the uterine wall. It can occur anywhere on the placenta and will cause painful, dark red vaginal bleeding. If the abruption is small, the ob/gyn will try to deliver the fetus vaginally. But if severe bleeding occurs or there is fetal distress, a cesarean birth will be performed. Women older than 35 are also at higher risk for developing placental abruption.

A pregnant women calls the clinic to report a small amount of painless vaginal bleeding. What response by the nurse is best? -"Please come in now for an evaluation by your health care provider." -"Lie on your left side and drink lots of water and monitor the bleeding." -"If the bleeding lasts more than 24 hours, call us for an appointment." -"Bleeding during pregnancy happens for many reasons, some serious and some harmless."

"Please come in now for an evaluation by your health care provider." Bleeding during pregnancy is always a deviation from normal and should be evaluated carefully. It may be life-threatening or it may be something that is not a threat to the mother and/or fetus. Regardless, it needs to be evaluated quickly and carefully. Telling the client it may be harmless is a reassuring statement, but does not suggest the need for urgent evaluation. Having the mother lay on her left side and drink water is indicated for cramping.

A client with a history of cervical insufficiency is seen for reports of pink-tinged discharge and pelvic pressure. The primary care provider decides to perform a cervical cerclage. The nurse teaches the client about the procedure. Which client response indicates that the teaching has been effective?

"Purse-string sutures are placed in the cervix to prevent it from dilating."

A client with a history of cervical insufficiency is seen for reports of pink-tinged discharge and pelvic pressure. The primary care provider decides to perform a cervical cerclage. The nurse teaches the client about the procedure. Which client response indicates that the teaching has been effective?

"Purse-string sutures are placed in the cervix to prevent it from dilating." The classic manifestations of abruption placenta are painful dark red vaginal bleeding, "knife-like" abdominal pain, uterine tenderness, contractions, and decreased fetal movement. Painless bright red vaginal bleeding is the clinical manifestation of placenta previa. Generalized vasospasm is the clinical manifestation of preeclampsia and not of abruptio placenta.

A 16-year-old client gave birth to a 12 weeks' gestation fetus last week. The client has come to the office for follow-up and while waiting in an examination room notices that on the schedule is written her name and "follow-up of spontaneous abortion." The client is upset about what is written on the schedule. How can the nurse best explain this terminology?

"Spontaneous abortion is a more specific term used to describe a spontaneous miscarriage, which is a loss of pregnancy before 20 weeks. This term does not imply that you did anything to affect the pregnancy." Abortion is a medical term for any interruption of a pregnancy before a fetus is viable, but it is better to speak of these early pregnancy losses as spontaneous abortions to avoid confusion with intentional terminations of pregnancies. The other responses are correct, but they do not provide the client with the most complete and reassuring answer.

A 16-year-old client was at 12 weeks' gestation when she gave birth to a fetus last week. The client has come to the office for follow-up and, while waiting in an examination room, notices that on the schedule is written her name and "follow-up of spontaneous abortion." The client is upset about what is written on the schedule. How can the nurse best explain this terminology? -"Spontaneous abortion is the medical name for a miscarriage." -"Abortion is a medical term for any interruption of pregnancy before a fetus is viable." -"Spontaneous abortion is a more specific term used to describe a spontaneous miscarriage, which is a loss of pregnancy before 20 weeks. This term does not imply that you did anything to affect the pregnancy." -"Oh, that just means it was a miscarriage."

"Spontaneous abortion is a more specific term used to describe a spontaneous miscarriage, which is a loss of pregnancy before 20 weeks. This term does not imply that you did anything to affect the pregnancy." Abortion is a medical term for any interruption of a pregnancy before a fetus is viable, but it is better to speak of these early pregnancy losses as spontaneous abortions to avoid confusion with intentional terminations of pregnancies. The other responses are correct, but they do not provide the client with the most complete nor reassuring answer.

A client in her first trimester arrives at the emergency room with reports of severe cramping and vaginal spotting. On examination, the health care provider informs her that no fetal heart sounds are evident and orders a dilatation and curettage. The client looks frightened and confused and states that she does not believe in abortion. Which statement by the nurse is best?

"Unfortunately the pregnancy is already lost. The procedure is to clear the uterus to prevent further complications" The nurse should not inform the client what she must do but supply information about what has happened and teach the client about the treatments that are used to correct the situation. A threatened spontaneous abortion (miscarriage) becomes an imminent (inevitable) miscarriage if uterine contractions and cervical dilation (dilatation) occur. A woman who reports cramping or uterine contractions is asked to seek medical attention. If no fetal heart sounds are detected and an ultrasound reveals an empty uterus or nonviable fetus, her health care provider may perform a dilatation and curettage (D&C) or a dilation and evacuation (D&E) to ensure all products of conception are removed. Be certain the woman has been told the pregnancy was already lost and all procedures, such as suction curettage, are to clear the uterus and prevent further complications such as infection, not to end the pregnancy. This scenario does not involve an abortion (elective termination of pregnancy) or an incomplete miscarriage. A 24-year-old woman presents with vague abdominal pains, nausea, and vomiting. An urine hCG is positive after the client mentioned that her last menstrual period was 2 months ago. The nurse should prepare the client for which intervention if the transvaginal ultrasound indicates a gestation sac is found in the right lower quadrant?

A client in her first trimester arrives at the emergency room with reports of severe cramping and vaginal spotting. On examination, the health care provider informs her that no fetal heart sounds are evident and orders a dilatation and curettage. The client looks frightened and confused and states that she does not believe in abortion. Which statement by the nurse is best? -"Unfortunately, the pregnancy is already lost. The procedure is to clear the uterus to prevent further complications." -"I know that it is sad but the pregnancy must be terminated to save your life." -"The choice is up to you but the health care provider is recommending an abortion." -"You have experienced an incomplete miscarriage and must have the placenta and any other tissues cleaned out."

"Unfortunately, the pregnancy is already lost. The procedure is to clear the uterus to prevent further complications." The nurse should not inform the client what she must do but supply information about what has happened and teach the client about the treatments that are used to correct the situation. A threatened spontaneous abortion (miscarriage) becomes an imminent (inevitable) miscarriage if uterine contractions and cervical dilation (dilatation) occur. A woman who reports cramping or uterine contractions is asked to seek medical attention. If no fetal heart sounds are detected and an ultrasound reveals an empty uterus or nonviable fetus, her health care provider may perform a dilatation and curettage (D&C) or a dilation and evacuation (D&E) to ensure all products of conception are removed. Be certain the woman has been told the pregnancy was already lost and all procedures, such as suction curettage, are to clear the uterus and prevent further complications such as infection, not to end the pregnancy. This scenario does not involve an abortion (elective termination of pregnancy) or an incomplete miscarriage.

A client is 11 weeks' pregnant after many years trying to conceive. After arriving home from a normal prenatal visit, she experiences mild cramping and has a gush of bright red vaginal bleeding. She calls the nurse and reports having soaked a pad with fresh blood in fewer than 30 minutes. The uterine cramping is worsening. What is the most appropriate response from the nurse? "This is nothing to worry about. Many women bleed during pregnancy." "Lie down and call your health care provider tomorrow if symptoms continue." "I am sorry. There is nothing you can do because you are likely miscarrying." "You need to seek immediate attention from the primary care provider."

"You need to seek immediate attention from the primary care provider." Explanation: Pregnancy loss during the early weeks of pregnancy may seem like a heavy menstrual period. A primary care provider should assess blood loss of this amount with or without uterine cramping as soon as possible.

A client experiences a threatened abortion. She is concerned about losing the pregnancy and asks what activity level she should maintain. What is the most appropriate response from the nurse? a) "There is no research evidence that I can recommend to you." b) "Carry on with the activity you engaged in before this happened." c) "Strict bedrest is necessary so as not to jeopardize this pregnancy." d) "Restrict your physical activity to moderate bedrest.

...

A pregnant client in her 35th week of gestation arrives at the clinic with complaints of abdominal pain and spotting. The ultrasound indicates that the placenta is partially covering the dilated internal os. The nurse interprets this as which type of placenta previa? a) Type III b) Type IV c) Type II d) Type I

...

A woman in week 35 of her pregnancy with severe hydramnios is admitted to the hospital. The nurse recognizes that which of the following is the biggest concern regarding this client? a) Development of eclampsia b) Development of gestational trophoblastic disease c) Hemorrhaging d) Preterm rupture of membranes followed by preterm birth

...

b. Initial BP 100/70 mm Hg; current BP 140/90 mm Hg

A 25-year-old client at 22 weeks' gestation is noted to have proteinuria and dependent edema on her routine prenatal visit. Which additional assessment should the nurse prioritize and alert the RN or health care provider? a. Initial BP 110/60 mm Hg; current BP 112/86 mm Hg b. Initial BP 100/70 mm Hg; current BP 140/90 mm Hg c. Initial BP 120/80mm Hg; current BP 130/88 mm Hg d. Initial BP 140/85 mm Hg; current BP 130/80 mm Hg

b. Assess client for anti-D antibodies

A 28-year-old client and her current partner present for the first antenatal OB appointment. The client has no children but does question a possible miscarriage 2 years ago; however, she never sought medical attention because she felt fine. Labs reveal both client and partner are Rh negative. Which action should the nurse prioritize? a. Continue with routine procedures and tasks b. Assess client for anti-D antibodies c. Arrange for an amniocentesis d. Perform direct Coombs test

c. Ectopic pregnancy

A 28-year-old client with a history of endometriosis presents to the emergency department with severe abdominal pain and nausea and vomiting. The client also reports her periods are irregular with the last one being 2 months ago. The nurse prepares to assess for which possible cause for this client's complaints? a. Healthy pregnancy b. Molar pregnancy c. Ectopic pregnancy d. Placenta previa

a. Palpate the fundus, and check fetal heart rate.

A 32-year-old gravida 3 para 2 at 36 weeks' gestation comes to the obstetric department reporting abdominal pain. Her blood pressure is 164/90 mm/Hg, her pulse is 100 beats per minute, and her respirations are 24 per minute. She is restless and slightly diaphoretic with a small amount of dark red vaginal bleeding. What assessment should the nurse make next? a. Palpate the fundus, and check fetal heart rate. b. Measure fundal height. c. Obtain a voided urine specimen, and determine blood type. d. Check deep tendon reflexes.

A woman is to undergo labor induction. The nurse determines that the woman requires cervical ripening if her Bishop score is: 5. 6. 7. 9.

A Bishop score less than 6 usually indicates that a cervical ripening method should be used before labor induction.

c. gestational hypertension

A client has come to the office for a prenatal visit during her 22nd week of gestation. On examination, it is noted that her blood pressure has increased to 138/90 mm Hg. Her urine is negative for proteinuria. The nurse recognizes which factor as the potential cause? a. chronic hypertension b. HELLP c. gestational hypertension d. preeclampsia

c. "Unfortunately, the pregnancy is already lost. The procedure is to clear the uterus to prevent further complications."

A client in her first trimester arrives at the emergency room with reports of severe cramping and vaginal spotting. On examination, the health care provider informs her that no fetal heart sounds are evident and orders a dilatation and curettage. The client looks frightened and confused and states that she does not believe in abortion. Which statement by the nurse is best? a. "I know that it is sad but the pregnancy must be terminated to save your life." b. "The choice is up to you but the healthcare provider is recommending an abortion." c. "Unfortunately, the pregnancy is already lost. The procedure is to clear the uterus to prevent further complications." d. "You have experienced an incomplete miscarriage and must have the placenta and any other tissues cleaned out."

a. There is no treatment for newborns with zika, but they will have supportive care based on the defects. b. Zika can be transmitted by mosquitoes, sexual activity, and blood exposure. d. It is best for men who have been exposed to zika to wait six months before attempting conception. e. A pregnant woman with zika may have a baby with microcephaly and other congenital anomalies.

A community health nurse is teaching a group of clients about the zika virus. Which statements by the clients indicate to the nurse that the teaching was effective? (Select all that apply.) a. There is no treatment for newborns with zika, but they will have supportive care based on the defects. b. Zika can be transmitted by mosquitoes, sexual activity, and blood exposure. c. Women who have been exposed to zika should wait six months before attempting conception. d. It is best for men who have been exposed to zika to wait six months before attempting conception. e. A pregnant woman with zika may have a baby with microcephaly and other congenital anomalies.

A primipara at 36 weeks' gestation is being monitored in the prenatal clinic for risk of preeclampsia. Which sign or symptom should the nurse prioritize? A systolic blood pressure increase of 10 mm Hg Weight gain of 1.2 lb (0.54 kg) during the past 1 week A dipstick value of 2+ for protein Pedal edema

A dipstick value of 2+ for protein

A primipara at 36 weeks' gestation is being monitored in the prenatal clinic for risk of preeclampsia. Which sign or symptom should the nurse prioritize? -A systolic blood pressure increase of 10 mm Hg -Weight gain of 1.2 lb (0.54 kg) during the past 1 week -A dipstick value of 2+ for protein -Pedal edema

A dipstick value of 2+ for protein The increasing amount of protein in the urine is a concern the preeclampsia may be progressing to severe preeclampsia. The woman needs further assessment by the health care provider. Dependent edema may be seen in a majority of pregnant women and is not an indicator of progression from preeclampsia to eclampsia. Weight gain is no longer considered an indicator for the progression of preeclampsia. A systolic blood pressure increase is not the highest priority concern for the nurse, since there is no indication what the baseline blood pressure was.

infection

A foul odor of amniotic fluid indicates _________________.

A woman at 10 weeks' gestation comes to the clinic for an evaluation. Which assessment finding should the nurse prioritize?

A fundal height of 18 cm is larger than expected and should be further investigated for gestational trophoblastic disease (hydatidiform mole). One of the presenting signs is the uterus being larger than expected for date. Mild nausea would be a normal finding at 10 weeks' gestation. Blood pressure of 120/84 mm Hg would not be associated with hydatidiform mole and depending on the woman's baseline blood pressure may be within acceptable parameters for her. Bright red spotting might suggest a spontaneous abortion (miscarriage).

d. Dependent edema may be seen in the sacral area if the client is on bed rest

A nurse is assessing a pregnant client with preeclampsia for suspected dependent edema. Which description of dependent edema is most accurate? a. Dependent edema leaves a small depression or pit after finger pressure is applied to a swollen area. b. Dependent edema occurs only in clients on bed rest. c. Dependent edema can be measured when pressure is applied. d. Dependent edema may be seen in the sacral area if the client is on bed rest

c. Coma occurs after seizure

A nurse is caring for a pregnant client with eclamptic seizure. Which is a characteristic of eclampsia? a. Muscle rigidity is followed by facial twitching b. Respirations are rapid during the seizure c. Coma occurs after seizure d. Respiration fails after the seizure

d. fetal distress related to hypoxia

A nurse is monitoring a client with PROM who is in labor and observes meconium in the amniotic fluid. What does the observation of meconium indicate? a. infection b. central nervous system (CNS) involvement c. cord compression d. fetal distress related to hypoxia

d. Institute and maintain seizure precautions.

A pregnant woman has been admitted to the hospital due to severe preeclampsia. Which measure will be important for the nurse to include in the care plan? a. Plan for immediate induction of labor. b. Admit the client to the middle of ICU where she can be constantly monitored. c. Institute NPO status. d. Institute and maintain seizure precautions.

b. assessing fetal heart tones by use of an external monitor

A pregnant woman is admitted to the hospital with a diagnosis of placenta previa. Which action would be the priority for this woman on admission? a. performing a vaginal examination to assess the extent of bleeding b. assessing fetal heart tones by use of an external monitor c. assessing uterine contractions by an internal pressure gauge d. helping the woman remain ambulatory to reduce bleeding

A woman at 31 weeks' gestation presents to the emergency department with bright red vaginal bleeding, reporting that the onset of the bleeding was sudden and without pain. Which diagnostic test should the nurse prioritize?

A transvaginal ultrasound

b. Premature separation of the placenta

A woman in labor suddenly reports sharp fundal pain accompanied by slight dark red vaginal bleeding. The nurse should prepare to assist with which situation? a. Possible fetal death or injury b. Premature separation of the placenta c. Preterm labor that was undiagnosed d. Placenta previa obstructing the cervix

b. diminished reflexes

A woman is being closely monitored and treated for severe preeclampsia with magnesium sulfate. Which finding would alert the nurse to the development of magnesium toxicity in this client? a. serum magnesium level of 6.5 mEq/L b. diminished reflexes c. elevated liver enzymes d. seizures

A. because assessment of serum chorionic gonadotropin (hCG) is considered a specific tumor marker for gestational trophoblastic disease that isn't resolved. hCG levels are assayed at frequent intervals for up to a year. Pregnancy would obscure the evidence of choriocarcinoma by the normal secretion of hCG.

A woman is being discharged after receiving treatment for a hydatidiform molar pregnancy. The nurse should include which of the following in her discharge teaching? a. Do not become pregnant for at least a year; use contraceptives to prevent it b. Have the client's blood pressure checked weekly in the clinic c. RhoGAM must be given within the next month to her at the clinic d. An amniocentesis can detect a recurrence of this disorder in the future

The nurse is caring for an Rh-negative nonimmunized clients at 14 weeks' gestation. What information would the nurse provide to the client? A. Obtain RhoGAM at 28 weeks' gestation B. Consume a well-balanced, nutritional diet C. Avoid sexual activity until after 28 weeks D. Undergo periodic transvaginal ultrasounds

A. Obtain RhoGAM at 28 weeks' gestation

The nurse is caring for a 2-day-old newborn whose mother was diagnosed with cytomegalovirus during the first trimester. On which health care provider order should the nurse place the priority? A. Perform a hearing screen test B. Obtain a urine specimen C. Monitor growth and development D. Assess pulse rate

A. Perform a hearing screen test

A nursing is caring for a client undergoing treatment for ectopic pregnancy. Which symptom is observed in a client if rupture or hemorrhaging occurs before the ectopic pregnancy is successfully treated? A. Phrenic nerve irritation B. Painless bright red vaginal bleeding C. Fetal distress D. Tetanic contractions

A. Phrenic nerve irritation

A pregnant client is brought to the health care facility with signs of premature rupture of membranes (PROM). Which conditions and complications are associated with PROM? SATA A. Prolapsed cord B. Abruptio placenta C. Spontaneous abortion D. Placenta previa E. Preterm labor

A. Prolapsed cord B. Abruptio placenta E. Preterm labor

A client reporting she recently had a positive pregnancy test has reported to the emergency department stating one-sided lower abdominal pain. The health care provider has prescribed a series of tests. Which test will provide the most definitive confirmation of an ectopic pregnancy? -Quantitative human chorionic gonadotropin (hCG) test -Qualitative human chorionic gonadotropin (hCG) test -Pelvic examination -Abdominal ultrasound

Abdominal ultrasound An ectopic pregnancy refers to the implantation of the fertilized egg in a location other than the uterus. Potential sites include the cervix, uterus, abdomen, and fallopian tubes. The confirmation of the ectopic pregnancy can be made by an ultrasound, which would confirm that there was no uterine pregnancy. A quantitative hCG level may be completed in the diagnostic plan. hCG levels in an ectopic pregnancy are traditionally reduced. While this would be an indication, it would not provide a positive confirmation. The qualitative hCG test would provide evidence of a pregnancy, but not the location of the pregnancy. A pelvic exam would be included in the diagnostic plan of care. It would likely show an enlarged uterus and cause potential discomfort to the client but would not be a definitive finding.

A client reporting she recently had a positive pregnancy test has reported to the emergency department stating one-sided lower abdominal pain. The health care provider has prescribed a series of tests. Which test will provide the most definitive confirmation of an ectopic pregnancy? Quantitative human chorionic gonadotropin (hCG) test Qualitative human chorionic gonadotropin (hCG) test Pelvic examination Abdominal ultrasound

Abdominal ultrasound Explanation: An ectopic pregnancy refers to the implantation of the fertilized egg in a location other than the uterus. Potential sites include the cervix, uterus, abdomen, and fallopian tubes. The confirmation of the ectopic pregnancy can be made by an ultrasound, which would confirm that there was no uterine pregnancy. A quantitative hCG level may be completed in the diagnostic plan. hCG levels in an ectopic pregnancy are traditionally reduced. While this would be an indication, it would not provide a positive confirmation. The qualitative hCG test would provide evidence of a pregnancy, but not the location of the pregnancy. A pelvic exam would be included in the diagnostic plan of care. It would likely show an enlarged uterus and cause potential discomfort to the client but would not be a definitive finding.

A nurse is caring for a client with hyperemesis gravidarum. Which nursing action is the priority for this client? Administer total parenteral nutrition. Administer an antiemetic. Set up for a percutaneous endoscopic gastrostomy. Administer IV NS with vitamins and electrolytes.

Administer IV NS with vitamins and electrolytes.

A nurse is caring for a client with hyperemesis gravidarum. Which nursing action is the priority for this client?

Administer IV NS with vitamins and electrolytes. The first choice for fluid replacement is generally NS with vitamins and electrolytes added. If the client does not improve after several days of bed rest, "gut rest," IV fluids, and antiemetics, then total parenteral nutrition or percutaneous endoscopic gastrostomy tube feeding is instituted to prevent malnutrition.

A pregnant client is admitted to a health care unit with disseminated intravascular coagulation (DIC). Which prescription is the nurse most likely to receive regarding the therapy for such a client? Administer a ratio of 1 unit of blood to 4 units of frozen plasma. Administer cryoprecipitate and platelets. Aim at keeping the client's hematocrit above 20%. Give each unit of blood to raise the hematocrit by 3 g/dl (30 g/L).

Administer cryoprecipitate and platelets. Explanation: In a pregnant client with DIC, the nurse may be told to administer cryoprecipitate and platelets. Whole blood does not contain clotting factors. Therefore, a ratio of 4 units of blood to 1 unit of fresh frozen plasma, and not 1 unit of blood to 4 units of frozen plasma, should be considered. The nurse should aim at maintaining the client's hematocrit above 30% and not just 20%. The nurse should expect one unit of blood to increase the hematocrit by 1.5 g/dl (15 g/L) not 3g/dl (30 g/L).

A primigravida 28-year-old client is noted to have Rh negative blood and her husband is noted to be Rh positive. The nurse should prepare to administer RhoGAM after which diagnostic procedure? Contraction test Nonstress test Biophysical profile Amniocentesis

Amniocentesis Explanation: Amniocentesis is a procedure requiring a needle to enter into the amniotic sac. There is a risk of mixing of the fetal and maternal blood which could result in blood incompatibility. A contraction test, a nonstress test, and biophysical profile are not invasive, so there would be no indication for Rho(D) immune globulin to be administered.

A 28-year-old client and her current partner present for the first antenatal OB appointment. The client has no children but does question a possible miscarriage 2 years ago; however, she never sought medical attention because she felt fine. Labs reveal both client and partner are Rh negative. Which action should the nurse prioritize?

Assess client for anti-D antibodies The client should be checked for sensitization to Rh-positive blood. It is unknown if the client did have a miscarriage earlier, and if so, what the blood type was of that fetus. The risk is high for the current fetus to be affected with hemolytic disease and this can be easily ruled out by assessing the mother for sensitization. If this screening is negative, then no further testing is required. If the father were Rh positive, then the mother be given RhoGAM to prevent the woman from developing antibodies to the Rho(D) factor. However, if it is positive, the health care provider may order an amniocentesis to evaluate the fetus for hemolytic disease so proper treatment and monitoring may be given. It is too early to perform a direct Coombs test. It would be improper to ignore the potential of serious complications and simply continue with routine tasks and procedures at this time.

A pregnant patient is diagnosed with placenta previa. Which action should the nurse implement immediately for this patient?

Assess fetal heart sounds with an external monitor. For placenta previa, the nurse should attach external monitoring equipment to record fetal heart sounds and uterine contractions. Internal pressure gauges to measure uterine contractions are contraindicated. A pelvic or rectal examination should never be done with painless bleeding late in pregnancy because any agitation of the cervix when there is a placenta previa might tear the placenta further and initiate massive hemorrhage, which could be fatal to both mother and child. To ensure an adequate blood supply to the patient and fetus, the patient should be placed immediately on bed rest in a side-lying position.

A nurse in the maternity triage unit is caring for a client with a suspected ectopic pregnancy. Which nursing intervention should the nurse perform first? -Assess the client's vital signs. -Administer oxygen to the client. -Obtain a surgical consent from the client. -Provide emotional support to the client and significant other.

Assess the client's vital signs. A suspected ectopic pregnancy can put the client at risk for hypovolemic shock. The assessment of vital signs should be performed first, followed by any procedures to maintain the ABCs. Providing emotional support would also occur, as would obtaining a surgical consent, if needed, but these are not first steps.

A nurse in the maternity triage unit is caring for a client with a suspected ectopic pregnancy. Which nursing intervention should the nurse perform first? Assess the client's vital signs. Administer oxygen to the client. Obtain a surgical consent from the client. Provide emotional support to the client and significant other.

Assess the client's vital signs. Explanation: A suspected ectopic pregnancy can put the client at risk for hypovolemic shock. The assessment of vital signs should be performed first, followed by any procedures to maintain the ABCs. Providing emotional support would also occur, as would obtaining a surgical consent, if needed, but these are not first steps.

At 37 weeks' gestation, a woman presents to labor and delivery complaining of intense, knife-like abdominal pain that started suddenly about 1 hour ago and has not subsided. On palpation, the abdomen is rigid and board-like and no vaginal bleeding is evident. What should the nurse do next?

Assess the fetal heart rate The presence of intense, knife-like abdominal pain with a sudden onset, a rigid and board-like abdomen, and no vaginal bleeding is evidence of a placental abruption (abruptio placentae). The next action by the nurse is to assess the fetal heart rate to determine the fetus's status. The priority is saving the life of the fetus and the mother. Inserting a urinary catheter and administering oxygen can be done once the status of the fetus is known. This client is not an appropriate candidate for an epidural at this time.

A pregnant woman is admitted to the hospital with a diagnosis of placenta previa. Which of the following would be the priority for this woman on admission? a) Helping the woman remain ambulatory to reduce bleeding b) Assessing uterine contractions by an internal pressure gauge c) Performing a vaginal examination to assess the extent of bleeding d) Assessing fetal heart tones by use of an external monitor

Assessing fetal heart tones by use of an external monitor Correct Explanation: Not disrupting the placenta is a prime responsibility. An internal monitor, a vaginal examination, and remaining ambulatory could all do this and thus are contraindicated

The nurse is required to assess a client for HELLP syndrome. Which are the signs and symptoms of this condition? SATA A. Blood pressure higher than 160/110 B. Epigastric pain C. Oliguria D. Upper right quadrant pain E. Hyperbilirubinemia

B. Epigastric pain D. Upper right quadrant pain E. Hyperbilirubinemia

A nurse is monitoring a client with PROM who is in labor and observes meconium in the amniotic fluid. What does this indicate? A. Cord Compression B. Fetal distress related to hypoxia C. Infection D. Central nervous system "CNS" involvement

B. Fetal distress related to hypoxia

A nurse is documenting a dietary plan for a pregnant client with progestational diabetes. What instructions should the nurse include in the dietary plan for this client? A. Include more dairy products in the diet B. Include complex carbohydrates in the diet C. Eat only two meals per day D. Eat at least one egg per day

B. Include complex carbohydrates in the diet

A nurse caring for a pregnant client suspected substance use during pregnancy. What is the priority nursing intervention for this client? A. Determine how long the client has been using drugs B. Obtain a urine specimen for a drug screening C. Determine if the client has emotional support D. Provide education material on cessation of substance use

B. Obtain a urine specimen for a drug screening

A patient in labor and delivery has just been diagnosed with pre-eclampsia. Which signs and symptom should the nurse prioritize when assessing this client? Select all that apply.

BP 140/90 mm Hg edema of face headache Clinical manifestations of preeclampsia include hypertension of greater than 140 mm Hg systolic or 90 mm Hg diastolic, protein in the urine, edema, severe headache, hyperactive deep tendon reflexes (not slow) and clonus, blurred or double vision, and nausea and pain int he epigastric region. Glucose in the urine would possibly indicate diabetes. TAKE A PRACTICE QUIZ

The nurse is assisting a client who has just undergone an amniocentesis. Blood results indicate the mother has type O blood and the fetus has type AB blood. The nurse should point out the mother and fetus are at an increased risk for which situation related to this procedure?

Baby developing postbirth jaundice

The nurse is assisting a client who has just undergone an amniocentesis. Blood results indicate the mother has type O blood and the fetus has type AB blood. The nurse should point out the mother and fetus are at an increased risk for which situation related to this procedure? Placental abruption (abruptio placentae) Preterm birth Baby developing hemolytic anemia Baby developing postbirth jaundice

Baby developing postbirth jaundice Explanation: The infant and mother have ABO incompatibility. The result is a development of antibodies and breaking down of the blood, resulting in jaundice in the infant after delivery. The mixing of some fetal blood with maternal blood during the amniocentesis would not cause placental abruption or preterm birth. Hemolytic anemia is caused by Rh incompatibility. ABO incompatibility will cause hemolytic disease.

The nurse is caring for a newborn of a mother with human immunodeficiency virus (HIV). What is the priority for the nurse to complete following delivery? -Test the newborn for HIV -Bathe the newborn thoroughly -Administer zidovudine -Assist the mother to breastfeed

Bathe the newborn thoroughly The newborn should have a thorough bath immediately after birth to decrease the possibility of HIV transmission. It is recommended the newborn be tested for HIV at 14 to 21 days after birth, at 1-2 months and again at 4-6 months. Zidovudine should be administered within 6-12 hours post-delivery to help prevent transmission of HIV from the mother to the newborn.

A nurse is preparing a nursing care plan for a client who is admitted at 22 weeks' gestation with advanced cervical dilatation to 5 cm, cervical insufficiency, and a visible amniotic sac at the cervical opening. Which primary goal should the nurse prioritize at this point?

Bed rest to maintain pregnancy as long as possible

A nurse is preparing a nursing care plan for a client who is admitted at 22 weeks' gestation with advanced cervical dilation (dilatation) to 5 cm, cervical insufficiency, and a visible amniotic sac at the cervical opening. Which primary goal should the nurse prioritize at this point? Notification of social support for loss of pregnancy Education on causes of cervical insufficiency for the future Bed rest to maintain pregnancy as long as possible Give birth vaginally

Bed rest to maintain pregnancy as long as possible

A nurse is preparing a nursing care plan for a client who is admitted at 22 weeks' gestation with advanced cervical dilation (dilatation) to 5 cm, cervical insufficiency, and a visible amniotic sac at the cervical opening. Which primary goal should the nurse prioritize at this point?

Bed rest to maintain pregnancy as long as possible Explanation: At 22 weeks' gestation, the fetus is not viable. The woman would be placed on total bed rest with every attempt made to halt any further progression of dilation (dilatation) of the cervix. The nurse would not want this fetus to be born vaginally at this stage of gestation. It is not the nurse's responsibility to notify the client's social support of a possible loss of the pregnancy. It is not appropriate at this time to educate the mother on causes of cervical insufficiency for future pregnancies.

The nurse is orientating in the Labor and Delivery unit and asks her preceptor how to differentiate a client with preeclampsia from one with eclampsia. Which symptoms would the preceptor describe to the new nurse as indicative of severe preeclampsia? Select all that apply.

Blood pressure above 160/110 mm Hg Nondependent edema Hyperactive deep tendon reflexes Preeclampsia occurs when a pregnant woman develops hypertension occurring after 20 weeks gestation and only resolves after the fetus is delivered. Preeclampsia is exhibited by 2+ or more proteinuria, nondependent edema, blood pressure greater than 140 mm Hg systolic and above 90 mm Hg diastolic, and CNS irritability demonstrated by hyperactive deep tendon reflexes. If the client has a seizure, he has moved to eclampsia. Glycosuria is not associated with preeclampsia.

Preeclampsia with Severe Features Blood Pressure? Seizures/coma? Hyperreflexia? Other signs and symptoms?

Blood pressure: ≥160/110 mm Hg on two occasions at least 6 hours apart while on bed rest Seizures/coma? NO Hyperreflexia? YES Other signs and symptoms? Headache Oliguria Blurred vision, scotomata (blind spots) Pulmonary edema Thrombocytopenia (platelet count <100,000 platelets/mm3) Cerebral disturbances Persistent epigastric or right upper quadrant pain HELLP Progressive renal insufficiency

When performing an assessment on a neonate, which assessment finding is suggestive of hypothermia? -Bradycardia -Hyperglycemia -Metabolic alkalosis -Shivering

Bradycardia Bradycardia is an indicator that the neonate is hypothermic. A cold infant may develop acidosis as a result of metabolism of brown fat. Newborns do not shiver when cold. Hyperglycemia and metabolic alkalosis are not signs or consequences of hypothermia.

Hyperreflexia

Brisk reflexes; a common presenting symptom of preeclampsia and is the result of an irritable cortex.

The nurse is comforting and listening to a young couple who just suffered a miscarriage. When asked why this happened, which reason should the nurse share as a common cause?

Chromosomal abnormality The most common cause for the loss of a fetus in the first trimester is associated with a genetic defect or chromosomal abnormality. There is nothing that can be done, and the mother should feel no fault. The nurse needs to encourage the parents to speak with a health care provider for further information and questions related to genetic testing. Early pregnancy loss is not associated with maternal smoking, lack of prenatal care, or the age of the mother.

The nurse is comforting and listening to a young couple who just suffered a spontaneous abortion (miscarriage). When asked why this happened, which reason should the nurse share as a common cause? Maternal smoking Lack of prenatal care Chromosomal abnormality The age of the mother

Chromosomal abnormality Explanation: The most common cause for the loss of a fetus in the first trimester is associated with a genetic defect or chromosomal abnormality. There is nothing that can be done and the mother should feel no fault. The nurse needs to encourage the parents to speak with a health care provider for further information and questions related to genetic testing. Early pregnancy loss is not associated with maternal smoking, lack of prenatal care, or the age of the mother.

A woman in her 20s has experienced a spontaneous abortion (miscarriage) at 10 weeks' gestation and asks the nurse at the hospital what went wrong. She is concerned that she did something that caused her to lose her baby. The nurse can reassure the woman by explaining that the most common cause of miscarriage in the first trimester is related to which factor? Advanced maternal age Faulty implantation Chromosomal defects in the fetus Exposure to chemicals or radiation

Chromosomal defects in the fetus

A woman in her 20s has experienced a spontaneous abortion (miscarriage) at 10 weeks' gestation and asks the nurse at the hospital what went wrong. She is concerned that she did something that caused her to lose her baby. The nurse can reassure the woman by explaining that the most common cause of miscarriage in the first trimester is related to which factor? -Exposure to chemicals or radiation -Advanced maternal age -Chromosomal defects in the fetus -Faulty implantation

Chromosomal defects in the fetus Fetal factors are the most common cause of early miscarriages, with chromosomal abnormalities in the fetus being the most common reason. This client fits the criteria for early spontaneous abortion since she was only 10 weeks' pregnant and early miscarriage occurs before 12 weeks.

A woman in her 20s has experienced a spontaneous abortion (miscarriage) at 10 weeks' gestation and asks the nurse at the hospital what went wrong. She is concerned that she did something that caused her to lose her baby. The nurse can reassure the woman by explaining that the most common cause of miscarriage in the first trimester is related to which factor? Exposure to chemicals or radiation Advanced maternal age Chromosomal defects in the fetus Faulty implantation

Chromosomal defects in the fetus Explanation: Fetal factors are the most common cause of early miscarriages, with chromosomal abnormalities in the fetus being the most common reason. This client fits the criteria for early spontaneous abortion (miscarriage) since she was only 10 weeks' pregnant and early miscarriage occurs before 12 weeks.

A pregnant client with multiple gestation arrives at the maternity clinic for a regular antenatal check up. The nurse would be aware that client is at risk for which perinatal complication?

Congenital anomalies

An 18-year-old pregnant client is hospitalized as she recovers from hyperemesis gravidarum. The client reveals she wanted to have an abortion but her cultural background forbids it. She is very unhappy about being pregnant and even expresses a wish for a miscarriage. Which action by the nurse is most appropriate? -Encourage the client to be positive about the situation. -Continue to monitor the client's hyperemesis gravidarum. -Contact the health care provider to report the client's feelings. -Share the information with the client's family.

Contact the health care provider to report the client's feelings. The client may be experiencing a psychological situation that needs intervention by a trained professional in the area of mental health. The hyperemesis gravidarum may worsen her feelings toward the pregnancy, so reporting her feelings to the health care provider is the best action at this time. Although the nurse will continue to monitor the client's hyperemesis gravidarum, this is not the only action needed at this time and there is a better action. Encouraging the client to be positive about her situation may obstruct therapeutic communication. Sharing the information with the client's family is not appropriate, because the scenario described does not indicate that the nurse has the client's permission to share this information with the family.

The nurse is conducting a preadmission class for a group of parents on the safety features that are utilized to help prevent infant abduction. The nurse should prioritize which factor as most essential to ensure the program's success? -Use of pass codes onto the unit -Use of monitor attached to babies -Use of cameras at all doors -Cooperation by the parents with the hospital policies

Cooperation by the parents with the hospital policies The most essential piece to an effective infant abduction prevention plan is the cooperation of the parents. If the parents are not willing to participate in the unit policy, the unit is at risk. Using pass codes, placing cameras at each door, and using monitors on the infants will all help, but only if the parents are cooperative.

A nurse is care for a 45-year-old pregnant client with a cardiac disorder, who has been instructed by her physician to follow class I functional activity recommendations. The nurse correctly instructs the client to follow which limitations? A. "You will need to be on bedrest for the remainder of your pregnancy." B. "It is important for you to rest after my physical activity in order to prevent any cardiac complications." C. "It will be beneficial if you plan rest periods throughout the day." D. "You do not need to limit your physical activity unless you experience any problems such as fatigue, chest pain, or shortness of breath."

D. "You do not need to limit your physical activity unless you experience any problems such as fatigue, chest pain, or shortness of breath."

A nurse is assessing a pregnant client with preeclampsia for suspected dependent edema. Which description of dependent edema is most accurate? A. Dependent edema leaves a small depression or pit after finger pressure is applied to a swollen area B. Dependent edema occurs only in client's on bed rest C. Dependent edema can be measured when pressure is applied D. Dependent edema may be seen in the sacral area if the client is on bed rest

D. Dependent edema may be seen in the sacral area if the client is on bed rest

The nurse is caring for a pregnant client with severe preeclampsia. Which nursing interventions should a nurse perform to institute and maintain seizure precautions in this client? A. Provide a well-lit room B. Keep head of bed slightly elevated C. Place the client in supine position D. Keep the suction equipment readily available

D. Keep the suction equipment readily available

A nurse is caring for a pregnant client who is human immunodeficiency virus (HIV) positive. What is a priority issue that the nurse should discuss with the client? A. The client's relationship with the spouse B. The amount of physical contact that should occur with the infant C. The client's plan for future pregnancies D. The need for the client to avoid breast-feeding

D. The need for the client to avoid breast-feeding

A nurse is caring for a young woman who is in her 10th week of gestation. She comes into the clinic reporting vaginal bleeding. Which assessment finding best correlates with a diagnosis of hydatidiform mole? Bright red, painless vaginal bleeding Painful uterine contractions and nausea Dark red, "clumpy" vaginal discharge Brisk deep tendon reflexes and shoulder pain

Dark red, "clumpy" vaginal discharge

A nurse is caring for a young woman who is in her 10th week of gestation. She comes into the clinic reporting vaginal bleeding. Which assessment finding best correlates with a diagnosis of hydatidiform mole?

Dark red, "clumpy" vaginal discharge Explanation: Women with hydatidiform mole ("molar pregnancy") often pass blood clots or watery brown/dark red discharge from the vagina in the first trimester. If a complete molar pregnancy continues into the second trimester undetected, other signs and symptoms appear. The woman often presents with complaints of dark to bright red vaginal bleeding and pelvic pain. Infrequently, she will report passage of grapelike vesicles.

The nurse is assessing a new client who is being admitted with gestational hypertension. Which nursing diagnosis should the nurse prioritize for this client? Deficient fluid volume related to vasospasm of arteries Decreased reflexes due to medication administration Risk for injury related to fetal distress Imbalanced nutrition related to decreased sodium levels

Deficient fluid volume related to vasospasm of arteries

The nurse is assessing a new client who is being admitted with gestational hypertension. Which nursing diagnosis should the nurse prioritize for this client? Decreased reflexes due to medication administration Imbalanced nutrition related to decreased sodium levels Deficient fluid volume related to vasospasm of arteries Risk for injury related to fetal distress

Deficient fluid volume related to vasospasm of arteries

The nurse is assessing a new client who is being admitted with gestational hypertension. Which nursing diagnosis should the nurse prioritize for this client? Deficient fluid volume related to vasospasm of arteries Decreased reflexes due to medication administration Risk for injury related to fetal distress Imbalanced nutrition related to decreased sodium levels

Deficient fluid volume related to vasospasm of arteries Explanation: Gestational hypertension is caused by vasospasms of the arteries. This leads to increased blood pressure and edema. Extensive edema leads to a deficiency of fluid volume. Decreased reflexes is related to the use of magnesium, which is given if the client has developed severe preeclampsia. It would not be appropriate for a client with gestational hypertension. The primary care provider may prescribe a antihypertensive if the benefits outweigh the risks for the mother and fetus.

What special interventions would the nurse implement in a client who is carrying twin fetuses? -Schedule non-stress tests (NST) starting at 16 weeks. -Demonstrate to the client how to perform fetal movement counts after 32 weeks. -Assist the physician on doing uterine ultrasounds every 2 weeks to monitor fetal size and placental information. -Remind the client to monitor her intake since she does not need any more food for a multiple pregnancy than she would ingest for a singleton pregnancy.

Demonstrate to the client how to perform fetal movement counts after 32 weeks. A woman carrying a multiple gestation needs to keep up with how her fetuses are doing, and an excellent way to do that is by doing fetal movement counts, or "kick counts" as they are sometimes called. This starts at around 32 weeks' gestation for an uncomplicated pregnancy and continues until birth. Weekly or bi-weekly NSTs begin after 32 weeks. Obstetrical ultrasounds are done every 4 to 6 weeks after confirmation of a multiple fetal pregnancy. The client needs to increase her intake, along with her iron and folic acid intake, to provide adequate nutrition for both fetuses.

What special interventions would the nurse implement in a client who is carrying twin fetuses?

Demonstrate to the client how to perform fetal movement counts after 32 weeks. A woman carrying a multiple gestation needs to keep up with how her fetuses are doing, and an excellent way to do that is by doing fetal movement counts, or "kick counts" as they are sometimes called. This starts at around 32 weeks' gestation for an uncomplicated pregnancy and continues until delivery. Weekly or bi-weekly NSTs begin after 32 weeks. Obstetrical ultrasounds are done every 4 to 6 weeks after confirmation of a multiple fetal pregnancy. The client needs to increase her intake, along with her iron and folic acid intake, to provide adequate nutrition for both fetuses.

A nurse is assessing a pregnant client with preeclampsia for suspected dependent edema. Which description of dependent edema is most accurate?

Dependent edema may be seen in the sacral area if the client is on bed rest.

A nurse is assessing a pregnant client with preeclampsia for suspected dependent edema. Which description of dependent edema is most accurate?

Dependent edema may be seen in the sacral area if the client is on bed rest. Explanation: The nurse should know that dependent edema may be seen in the sacral area if the client is on bed rest. Pitting edema leaves a small depression or pit after finger pressure is applied to a swollen area and can be measured. Dependent edema may occur in clients who are both ambulatory and on bed rest.

A client reports bright red, painless vaginal bleeding during her 32nd week of pregnancy. A sonogram reveals that the placenta has implanted low in the uterus and is partially covering the cervical os. Which immediate care measures are initiated? Select all that apply. Place the woman on bed rest maintaining the supine position. Determine the time the bleeding began and about how much blood has been lost. Obtain baseline vital signs and compare to those vital signs previously obtained. Assist the client in stirrups and perform a pelvic examination. Attach external monitoring equipment to record fetal heart sounds and kick counts.

Determine the time the bleeding began and about how much blood has been lost. Obtain baseline vital signs and compare to those vital signs previously obtained. Attach external monitoring equipment to record fetal heart sounds and kick counts.

When administering magnesium sulfate to a woman with severe preeclampsia, which finding would alert the nurse to the development of magnesium toxicity? a) Seizures b) Serum magnesium level of 6.5 mEq/L c) Diminished reflexes d) Elevated liver enzymes

Diminished reflexes Correct Explanation: Diminished or absent reflexes occur when a client develops magnesium toxicity. Elevated liver enzymes are unrelated to magnesium toxicity and may indicate the development of HELLP syndrome. The onset of seizure activity indicates eclampsia. A serum magnesium level of 6.5 mEq/L would fall within the therapeutic range of 4 to 7 mEq/L.

A pregnant client with hyperemesis gravidarum needs advice on how to minimize nausea and vomiting. Which instruction should the nurse give this client?

Eat small, frequent meals throughout the day. The nurse should instruct the client with hyperemesis gravidarum to eat small, frequent meals throughout the day to minimize nausea and vomiting. The nurse should also instruct the client to avoid lying down or reclining for at least 2 hours after eating and to increase the intake of carbonated beverages. The nurse should instruct the client to try foods that settle the stomach such as dry crackers, toast, or soda.

A 28-year-old client with a history of endometriosis presents to the emergency department with severe abdominal pain and nausea and vomiting. The client also reports her periods are irregular with the last one being 2 months ago. The nurse prepares to assess for which possible cause for this client's complaints?

Ectopic pregnancy Explanation: The most commonly reported symptoms of ectopic pregnancy are pelvic pain and/or vaginal spotting. Other symptoms of early pregnancy, such as breast tenderness, nausea, and vomiting, may also be present. The diagnosis is not always immediately apparent because many women present with complaints of diffuse abdominal pain and minimal to no vaginal bleeding. Steps are taken to diagnose the disorder and rule out other causes of abdominal pain. Given the history of the client and the amount of pain, the possibility of ectopic pregnancy needs to be considered. A healthy pregnancy would not present with severe abdominal pain unless the client were term and she was in labor. With a molar pregnancy the woman typically presents between 8 to 16 weeks' gestation reporting painless (usually) brown to bright red vaginal bleeding. Placenta previa typically presents with painless, bright red bleeding that begins with no warning.

A 28-year-old client with a history of endometriosis presents to the emergency department with severe abdominal pain and nausea and vomiting. The client also reports her periods are irregular with the last one being 2 months ago. The nurse prepares to assess for which possible cause for this client's complaints?

Ectopic pregnancy The most commonly reported symptoms of ectopic pregnancy are pelvic pain and/or vaginal spotting. Other symptoms of early pregnancy, such as breast tenderness, nausea, and vomiting, may also be present. The diagnosis is not always immediately apparent because many women present with complaints of diffuse abdominal pain and minimal to no vaginal bleeding. Steps are taken to diagnose the disorder and rule out other causes of abdominal pain. Given the history of the client and the amount of pain, the possibility of ectopic pregnancy needs to be considered. A healthy pregnancy would not present with severe abdominal pain unless the client were term and she was in labor. With a molar pregnancy the woman typically presents between 8 to 16 weeks' gestation reporting painless (usually) brown to bright red vaginal bleeding. Placenta previa typically presents with painless, bright red bleeding that begins with no warning.

A 28-year-old client with a history of endometriosis presents to the emergency department with severe abdominal pain and nausea and vomiting. The client also reports her periods are irregular with the last one being 2 months ago. The nurse prepares to assess for which possible cause for this client's complaints? -Healthy pregnancy -Ectopic pregnancy -Molar pregnancy -Placenta previa

Ectopic pregnancy The most commonly reported symptoms of ectopic pregnancy are pelvic pain and/or vaginal spotting. Other symptoms of early pregnancy, such as breast tenderness, nausea, and vomiting, may also be present. The diagnosis is not always immediately apparent because many women present with complaints of diffuse abdominal pain and minimal to no vaginal bleeding. Steps are taken to diagnose the disorder and rule out other causes of abdominal pain. Given the history of the client and the amount of pain, the possibility of ectopic pregnancy needs to be considered. A healthy pregnancy would not present with severe abdominal pain unless the client were term and she was in labor. With a molar pregnancy the woman typically presents between 8 to 16 weeks' gestation reporting painless (usually) brown to bright red vaginal bleeding. Placenta previa typically presents with painless, bright red bleeding that begins with no warning.

A 28-year-old woman presents in the emergency room with severe abdominal pain. She has not had a normal period for 2 months but she reports that that is not abnormal for her. She has a history of endometriosis. What might the nurse suggest to the physician as a possible cause of the patient's abdominal pain? a) Healthy pregnancy b) Ectopic pregnancy c) Molar pregnancy d) Placenta previa

Ectopic pregnancy Correct Explanation: Ectopic pregnancy can present with severe unilateral abdominal pain. Given the history of the client , the amount of pain, the possibility of ectopic pregnancy needs to be considered. A healthy pregnancy would not present with severe abdominal pain unless the patient were term and she was in labor. With a molar pregnancy the woman typically presents between 8 to 16 weeks' gestation with complaints of painless (usually) brown to bright red vaginal bleeding. Placenta previa typically presents with painless, bright red bleeding that begins with no warning.

A woman with an incomplete abortion is to receive misoprostol. The nurse understands that the rationale for administering this drug is to: a) Halt the progression of the abortion b) Suppress the immune response to prevent isoimmunization c) Ensure passage of all the products of conception d) Alleviate strong uterine cramping

Ensure passage of all the products of conception Correct Explanation: Misoprostol is used to stimulate uterine contractions and evacuate the uterus after an abortion to ensure passage of all the products of conception. Rh (D) immunoglobulin is used to suppress the immune response and prevent isoimmunization.

In returning to the hospital floor after a weekend off, the nurse takes over care of a pregnant patient who is resting in a darkened room. The patient is receiving betamethasone and magnesium sulfate. What could the nurse deduce from those findings? a) The patient is suffering from hypertension and the care team is trying to lower her blood pressure so that she may return home until the baby is full term. b) The patient is suffering from severe preeclampsia and the care team is attempting to prevent advancement of the disorder to eclampsia; they are attempting to help the baby's lungs mature quickly so that they can deliver as soon as possible. c) The patient is suffering from eclampsia and the care team is attempting to prevent stroke and induce labor. d) The patient is suffering from mild preeclampsia and the care team is attempting to stabilize her and the baby before discharging her to home.

Explanation: The administration of magnesium sulfate is to relax the skeletal muscles and raise the threshold for a seizure. The administration of the betamethasone is to try and hasten the maturity of the fetus' lungs for delivery. This woman is in advanced preeclampsia and must be monitored for progression to eclampsia. The scenario described does not indicate a patient with hypertension who may be discharged home once the condition is under control. A woman in eclampsia would either be seizing or comatose, not resting in a quiet room. Symptoms of mild preeclampsia are limited to slightly elevated blood pressure and small amounts of protein in the urine. Betamethasone may be indicated at this time. (less) Reference: Ricci, S. S. Essentials of Maternity, Newborn, and Women's Health Nursing, 3rd ed., Philadelphia: Wolters Kluwer Health/Lippincott Williams & Wilkins, 2013, Chapter 19: Nursing Management of Pregnancy at Risk: Pregnancy-Related Complications,

A new mother calls her pediatrician's office concerned about her 2-week-old infant "crying all the time." When the nurse explores further, the mother reports that the infant cries at least 2 hours each day, usually in the afternoons. What recommendation would the nurse not make to this mother? -Rocking and talking to the infant -Swaddling the infant before returning to the crib -Feeding the infant more formula whenever she begins to fuss -Gently patting or stroking the infant's back

Feeding the infant more formula whenever she begins to fuss Crying by a young infant is frustrating for parents, so it is suggested that the parents first be sure that the infant's physical needs are met, then soothing measures are implemented. Feeding the infant every time he cries is not needed nor suggested. Swaddling, a soothing touch, and gentle pats on the back all help calm a fussy infant.

A novice nurse asks to be assigned to the least complex antepartum patient. Which of the following conditions would necessitate the least complex care requirements? a) Abruptio placenta. b) Placenta previa. c) Gestational hypertension. d) Pre-ecalmpsia.

Gestational hypertension. Correct Explanation: Hypertensive disorders represent the most common complication of pregnancy. Gestational hypertension is elevated blood pressure without proteinuria, other signs of pre-eclampsia, or pre-existing hypertension. Abruptio placenta (separation of the placenta from the uterine wall), placenta previa (placenta covering the cervical os), and pre-eclampsia are high-risk, potentially life-threatening conditions for the fetus and mother during labor and birth.

A client tells that nurse in the doctor's office that her friend developed high blood pressure on her last pregnancy. She is concerned that she will have the same problem. What is the standard of care for preeclampsia?

Have her blood pressure checked at every prenatal visit. Explanation: Preeclampsia and eclampsia are common problems for pregnant clients and require regular blood pressure monitoring at all prenatal visits. Antihypertensives are not prescribed unless the client is already hypertensive. Monitoring for headaches and swelling is a good predictor of a problem but doesn't address prevention—nor does it predict who will have hypertension. Taking aspirin has shown to reduce the risk in women who have moderate to high risk factors, but has shown no effect on those women with low risk factors.

A client tells that nurse in the doctor's office that her friend developed high blood pressure on her last pregnancy. She is concerned that she will have the same problem. What is the standard of care for preeclampsia? Take a low-dose antihypertensive prophylactically. Have her blood pressure checked at every prenatal visit. Monitor the client for headaches or swelling on the body. Take one aspirin every day.

Have her blood pressure checked at every prenatal visit. Explanation: Preeclampsia and eclampsia are common problems for pregnant clients and require regular blood pressure monitoring at all prenatal visits. Antihypertensives are not prescribed unless the client is already hypertensive. Monitoring for headaches and swelling is a good predictor of a problem but doesn't address prevention—nor does it predict who will have hypertension. Taking aspirin has shown to reduce the risk in women who have moderate to high risk factors, but has shown no effect on those women with low risk factors.

A client at 11 weeks' gestation experiences pregnancy loss. The client asks the nurse if the bleeding and cramping that occurred during the miscarriage were caused by working long hours in a stressful environment. What is the most appropriate response from the nurse?

I can understand your need to find an answer to what caused this. Let's talk about this further. Talking with the client may assist her to explore her feelings. She and her family may search for a cause for a spontaneous early bleeding so they can plan for future pregnancies. Even with modern technology and medical advances, however, a direct cause cannot usually be determined.

A client at 11 weeks' gestation experiences pregnancy loss. The client asks the nurse if the bleeding and cramping that occurred during the miscarriage were caused by working long hours in a stressful environment. What is the most appropriate response from the nurse? a) "It is hard to know why a woman bleeds during early pregnancy." b) "I can understand your need to find an answer to what caused this. Let's talk about this further." c) "Your spontaneous bleeding is not work-related." d) "Something was wrong with the fetus."

I can understand your need to find an answer to what caused this. Let's talk about this further." Correct Explanation: Talking with the client may assist her to explore her feelings. She and her family may search for a cause for a spontaneous early bleeding so they can plan for future pregnancies. Even with modern technology and medical advances, however, a direct cause cannot usually be determined.

A woman is admitted with a diagnosis of ectopic pregnancy. For which of the following would you anticipate beginning preparation? a) Bed rest for the next 4 weeks. b) Intravenous administration of a tocolytic. c) Immediate surgery. d) Internal uterine monitoring.

Immediate surgery. Correct Explanation: Ectopic pregnancy means an embryo has implanted outside the uterus, usually in the fallopian tube. Surgery is usually necessary to remove the growing structure before the tube ruptures or repair the tube if rupture has already occurred. Bed rest will not correct the problem of an ectopic pregnancy. Administering a tocolytic is not indicated, nor is internal uterine monitoring. This makes options A, B, and D incorrect. (less)

Medications Related to Abortions PGE2, dinoprostone (Cervidil, Prepidil Gel, Prostin E2)

Indication: Stimulates uterine contractions, causing expulsion of uterine contents; expels uterine contents in fetal death or missed abortion during second trimester; effaces and dilates the cervix in pregnancy at term Nursing Actions: -Bring gel to room temperature before administering. -Avoid contact with skin. -Use sterile technique to administer. -Keep client supine for 30 minutes after administering. -Document time of insertion and dosing intervals. -Remove insert with retrieval system after 12 hours or at the onset of labor. -Explain purpose and expected response to client.

Medications Related to Abortions Rh(D) immunoglobulin (Gamulin, HydroRho-D, RhoGAM, MICRhoGAM)

Indication: Suppresses immune response of nonsensitized Rh-negative clients who are exposed to Rh-positive blood to prevent isoimmunization in Rh-negative women exposed to Rh-positive blood after abortions, miscarriages, and pregnancies. Nursing Actions: -Administer intramuscularly in deltoid area. -Give only MICRhoGAM for abortions and miscarriages <12 weeks unless fetus or father is Rh-negative (unless client is Rh-positive, Rh antibodies are present). -Educate woman that she will need this after subsequent deliveries if fetuses are Rh-positive; also check lab study results prior to administering the drug.

Indications for Rho(D) immune globulin include isoimmunization, ectopic pregnancy, chorionic villus sampling, amniocentesis, prenatal hemorrhage, molar pregnancy, maternal trauma, percutaneous umbilical sampling, therapeutic or spontaneous abortion, fetal death, or fetal surgery.

Indications for Rho(D) immune globulin include isoimmunization, ectopic pregnancy, chorionic villus sampling, amniocentesis, prenatal hemorrhage, molar pregnancy, maternal trauma, percutaneous umbilical sampling, therapeutic or spontaneous abortion, fetal death, or fetal surgery.

The nurse is identifying nursing diagnoses for a patient with gestational hypertension. Which diagnosis would be the most appropriate for this patient?

Ineffective tissue perfusion related to vasoconstriction of blood vessels In gestational hypertension, vasospasm occurs in both small and large arteries during pregnancy. This can lead to ineffective tissue perfusion. There is no evidence to suggest that the fetus is in distress. There is no enough information to support imbalanced nutrition. Gestational hypertension does not affect heart contractions.

A 25-year-old client at 22 weeks' gestation is noted to have proteinuria and dependent edema on her routine prenatal visit. Which additional assessment should the nurse prioritize and alert the RN or health care provider? Initial BP 120/80mm Hg; current BP 130/88 mm Hg Initial BP 100/70 mm Hg; current BP 140/90 mm Hg Initial BP 140/85 mm Hg; current BP 130/80 mm Hg Initial BP 110/60 mm Hg; current BP 112/86 mm Hg

Initial BP 100/70 mm Hg; current BP 140/90 mm Hg

A pregnant woman has been admitted to the hospital due to severe preeclampsia. Which measure will be important for the nurse to include in the care plan?

Institute and maintain seizure precautions.

A pregnant woman has been admitted to the hospital due to severe preeclampsia. Which measure will be important for the nurse to include in the care plan? Institute NPO status. Admit the client to the middle of ICU where she can be constantly monitored. Plan for immediate induction of labor. Institute and maintain seizure precautions.

Institute and maintain seizure precautions.

A pregnant woman has been admitted to the hospital due to severe preeclampsia. Which measure will be important for the nurse to include in the care plan? Institute and maintain seizure precautions. Institute NPO status. Admit the client to the middle of ICU where she can be constantly monitored. Plan for immediate induction of labor.

Institute and maintain seizure precautions.

A pregnant woman has been admitted to the hospital due to preeclampsia with severe features. Which measure will be important for the nurse to include in the care plan? -Institute and maintain seizure precautions. -Institute NPO status. -Admit the client to the middle of ICU where she can be constantly monitored. -Plan for immediate induction of labor.

Institute and maintain seizure precautions. The woman with preeclampsia with severe features should be maintained on complete bed rest in a dark and quiet room to avoid stimulation. The client is at risk for seizures; therefore, institution and maintenance of seizure precautions should be in place.

A pregnant woman has been admitted to the hospital due to preeclampsia with severe features. Which measure will be important for the nurse to include in the care plan? Institute and maintain seizure precautions. Institute NPO status. Admit the client to the middle of ICU where she can be constantly monitored. Plan for immediate induction of labor.

Institute and maintain seizure precautions. Explanation: The woman with preeclampsia with severe features should be maintained on complete bed rest in a dark and quiet room to avoid stimulation. The client is at risk for seizures; therefore, institution and maintenance of seizure precautions should be in place.

A patient recovering from an uneventful vaginal delivery is prescribed Rho(D) immune globulin (RhIG). What should the nurse explain to the patient regarding the purpose of this medication?

It prevents maternal D antibody formation. Rho(D) immune globulin (RhIG) is given to Rh-negative pregnant patients to prevent the formation of maternal antibodies to the Rh-positive blood type of the developing fetus. This medication does not prevent fetal Rh blood formation, stimulate maternal immune antigens, or promote maternal antibody formation.

The nurse is caring for a pregnant client with severe preeclampsia. Which nursing intervention should a nurse perform to institute and maintain seizure precautions in this client?

Keep the suction equipment readily available. The nurse should institute and maintain seizure precautions such as padding the side rails and having oxygen, suction equipment, and call light readily available to protect the client from injury. The nurse should provide a quiet, darkened room to stabilize the client. The nurse should maintain the client on complete bed rest in the left lateral lying position and not in a supine position. Keeping the head of the bed slightly elevated will not help maintain seizure precautions.

A woman who is 10 weeks' pregnant calls the physician's office reporting "morning sickness" but, when asked about it, tells the nurse that she is nauseated and vomiting all the time and has lost 5 pounds. What interventions would the nurse anticipate for this client? Lab work will be drawn to rule out acid-base imbalances. An ultrasound will be done to reassess the correctness of gestational dates. Since morning sickness is a common problem for pregnant women, the nurse will suggest the woman drink more fluids and eat crackers. The nurse will encourage the woman to lie down and rest whenever she feels ill.

Lab work will be drawn to rule out acid-base imbalances. Explanation: Morning sickness that lasts all day and is severe is called hyperemesis gravidarum. It is much more serious than "morning sickness" and can lead to significant weight loss and electrolyte imbalance. Lab work needs to be drawn to determine the extent of electrolyte loss and acid-base balance. An ultrasound is performed but it is done to determine if the mother is experiencing a molar pregnancy. Treatment for hyperemesis gravidarum requires much more care than just rest, drinking fluids and eating crackers.

Which of the following would the nurse prepare to administer if ordered as treatment for an unruptured ectopic pregnancy? a) Promethazine b) Ondansetron c) Oxytocin d) Methotrexate

Methotrexate Correct Explanation: Methotrexate, a folic acid antagonist that inhibits cell division in the developing embryo, is most commonly used to treat ectopic pregnancy. Oxytocin is used to stimulate uterine contractions and would be inappropriate for use with an ectopic pregnancy. Promethazine and ondansetron are antiemetics that may be used to treat hyperemesis gravidarum

A pregnant client has been admitted with complaints of brownish vaginal bleeding. On examination there is an elevated hCG level, absent fetal heart sounds and a discrepancy between the uterine size and the gestational age. The nurse interprets these findings to suggest which of the following? a) Ectopic pregnancy b) Placenta previa c) Molar pregnancy d) Abruption of placenta

Molar pregnancy Correct Explanation: The client is most likely experiencing molar pregnancy. In molar pregnancy, there is an abnormal proliferation and eventual degeneration of the trophoblastic villi. The signs and symptoms of molar pregnancy include brownish vaginal bleeding, elevated hCG levels, discrepancy between the uterine size and the gestational age, and absent fetal heart sounds. Abruption of placenta is characterized by premature separation of the placenta. Ectopic pregnancy is a condition where there is implantation of the blastocyst outside the uterus. In placenta previa the placental attachment is at the lower uterine segment.

The nurse is caring for a pregnant client with fallopian tube rupture. Which intervention is the priority for this client?

Monitor the client vital signs & bleeding A nurse should closely monitor the client's vital signs and bleeding (peritoneal or vaginal) to identify hypovolemic shock that may occur with tubal rupture. Beta-hCG level is monitored to diagnose an ectopic pregnancy or impending abortion. Monitoring the mass with transvaginal ultrasound and determining the size of the mass are done for diagnosing an ectopic pregnancy. Monitoring the FHR does not help to identify hypovolemic shock.

The nurse is caring for a pregnant client with fallopian tube rupture. Which intervention is the priority for this client?

Monitor the client's vital signs and bleeding. A nurse should closely monitor the client's vital signs and bleeding (peritoneal or vaginal) to identify hypovolemic shock that may occur with tubal rupture. Beta-hCG level is monitored to diagnose an ectopic pregnancy or impending abortion. Monitoring the mass with transvaginal ultrasound and determining the size of the mass are done for diagnosing an ectopic pregnancy. Monitoring the FHR does not help to identify hypovolemic shock.

The nurse is caring for a pregnant client with fallopian tube rupture. Which intervention is the priority for this client? -Monitor the client's beta-hCG level. -Monitor the mass with transvaginal ultrasound. -Monitor the client's vital signs and bleeding. -Monitor the fetal heart rate (FHR).

Monitor the client's vital signs and bleeding. A nurse should closely monitor the client's vital signs and bleeding (peritoneal or vaginal) to identify hypovolemic shock that may occur with tubal rupture. Beta-hCG level is monitored to diagnose an ectopic pregnancy or impending abortion. Monitoring the mass with transvaginal ultrasound and determining the size of the mass are done for diagnosing an ectopic pregnancy. Monitoring the FHR does not help to identify hypovolemic shock.

The nurse is caring for a pregnant client with fallopian tube rupture. Which intervention is the priority for this client?

Monitor the client's vital signs and bleeding. Explanation: A nurse should closely monitor the client's vital signs and bleeding (peritoneal or vaginal) to identify hypovolemic shock that may occur with tubal rupture. Beta-hCG level is monitored to diagnose an ectopic pregnancy or impending spontaneous abortion (miscarriage). Monitoring the mass with transvaginal ultrasound and determining the size of the mass are done for diagnosing an ectopic pregnancy. Monitoring the FHR does not help to identify hypovolemic shock.

A 25-week-gestation client presents with a blood pressure of 142/90, pulse 78, no edema, and urine negative for protein. What would the nurse do next?

Notify the healthcare provider The client is exhibiting a sign of gestational hypertension, elevated blood pressure greater than or equal to 140/90 mm Hg that develops for the first time during pregnancy. The healthcare provider should be notified to assess the client. Without the presence of edema or protein in the urine, the client does not have preeclampsia.

A 32-year-old gravida 3 para 2 at 36 weeks' gestation comes to the obstetric department reporting abdominal pain. Her blood pressure is 164/90 mm Hg, her pulse is 100 beats per minute, and her respirations are 24 per minute. She is restless and slightly diaphoretic with a small amount of dark red vaginal bleeding. What assessment should the nurse make next? Measure fundal height. Obtain a voided urine specimen and determine blood type. Palpate the fundus and check fetal heart rate. Check deep tendon reflexes.

Palpate the fundus and check fetal heart rate.

A 32-year-old gravida 3 para 2 at 36 weeks' gestation comes to the obstetric department reporting abdominal pain. Her blood pressure is 164/90 mm Hg, her pulse is 100 beats per minute, and her respirations are 24 per minute. She is restless and slightly diaphoretic with a small amount of dark red vaginal bleeding. What assessment should the nurse make next? -Check deep tendon reflexes. -Measure fundal height. -Palpate the fundus and check fetal heart rate. -Obtain a voided urine specimen and determine blood type.

Palpate the fundus and check fetal heart rate. The classic signs of placental abruption (abruptio placentae) are pain, dark red vaginal bleeding, a rigid, board-like abdomen, hypertonic labor, and fetal distress.

A 32-year-old gravida 3 para 2 at 36 weeks' gestation comes to the obstetric department reporting abdominal pain. Her blood pressure is 164/90 mm Hg, her pulse is 100 beats per minute, and her respirations are 24 per minute. She is restless and slightly diaphoretic with a small amount of dark red vaginal bleeding. What assessment should the nurse make next? Check deep tendon reflexes. Measure fundal height. Palpate the fundus and check fetal heart rate. Obtain a voided urine specimen and determine blood type.

Palpate the fundus and check fetal heart rate. Explanation: The classic signs of placental abruption (abruptio placentae) are pain, dark red vaginal bleeding, a rigid, board-like abdomen, hypertonic labor, and fetal distress.

A 32-year-old gravida 3 para 2 at 36 weeks' gestation comes to the obstetric department reporting abdominal pain. Her blood pressure is 164/90 mm/Hg, her pulse is 100 beats per minute, and her respirations are 24 per minute. She is restless and slightly diaphoretic with a small amount of dark red vaginal bleeding. What assessment should the nurse make next? a) Palpate the fundus and check fetal heart rate. b) Obtain a voided urine specimen and determine blood type. c) Measure fundal height. d) Check deep tendon reflexes.

Palpate the fundus and check fetal heart rate. Explanation: The classic signs of abruption placentea are pain, dark red vaginal bleeding, a rigid, board-like abdomen, hypertonic labor, and fetal distres

A 32-year-old gravida 3 para 2 at 36 weeks' gestation comes to the obstetric department reporting abdominal pain. Her blood pressure is 164/90 mm/Hg, her pulse is 100 beats per minute, and her respirations are 24 per minute. She is restless and slightly diaphoretic with a small amount of dark red vaginal bleeding. What assessment should the nurse make next? a) Measure fundal height. b) Check deep tendon reflexes. c) Palpate the fundus and check fetal heart rate. d) Obtain a voided urine specimen and determine blood type.

Palpate the fundus and check fetal heart rate. Explanation: The classic signs of abruption placentea are pain, dark red vaginal bleeding, a rigid, board-like abdomen, hypertonic labor, and fetal distress.

A 32-year-old gravida 3 para 2 at 36 weeks' gestation comes to the obstetric department reporting abdominal pain. Her blood pressure is 164/90 mm/Hg, her pulse is 100 beats per minute, and her respirations are 24 per minute. She is restless and slightly diaphoretic with a small amount of dark red vaginal bleeding. What assessment should the nurse make next?

Palpate the fundus, and check fetal heart rate. The classic signs of abruptio placentae are pain, dark red vaginal bleeding, a rigid, board-like abdomen, hypertonic labor, and fetal distress.

A pregnant woman with preeclampsia is to receive magnesium sulfate IV. Which of the following assessments would be most important prior to administering a new dose? a) Blood pressure b) Anxiety level c) Pulse rate d) Patellar reflex

Patellar reflex Correct Explanation: A symptom of magnesium sulfate toxicity is loss of deep tendon reflexes. Assessing for one of these before administration is assurance the drug administration will be safe

After a regular prenatal visit, a pregnant client asks the nurse to describe the differences between placental abruption (abruptio placentae) and placenta previa. Which statement will the nurse include in the teaching? Placenta previa causes painful, dark red vaginal bleeding during pregnancy. Placenta previa is an abnormally implanted placenta that is too close to the cervix. Placental abruption requires "watchful waiting" during labor and birth. Placental abruption results in painless, bright red vaginal bleeding during labor

Placenta previa is an abnormally implanted placenta that is too close to the cervix.

A pregnant women calls the clinic to report a small amount of painless vaginal bleeding. What response by the nurse is best? "Please come in now for an evaluation by your health care provider." "Lie on your left side and drink lots of water and monitor the bleeding." "If the bleeding lasts more than 24 hours, call us for an appointment." "Bleeding during pregnancy happens for many reasons, some serious and some harmless."

Please come in now for an evaluation by your health care provider." Explanation: Bleeding during pregnancy is always a deviation from normal and should be evaluated carefully. It may be life-threatening or it may be something that is not a threat to the mother and/or fetus. Regardless, it needs to be evaluated quickly and carefully. Telling the client it may be harmless is a reassuring statement, but does not suggest the need for urgent evaluation. Having the mother lay on her left side and drink water is indicated for cramping.

A 44-year-old client has lost several pregnancies over the last 10 years. For the past 3 months, she has had fatigue, nausea, and vomiting. She visits the clinic and takes a pregnancy test; the results are positive. Physical examination confirms a uterus enlarged to 13 weeks' gestation; fetal heart tones are heard. Ultrasound reveals that the client is experiencing some bleeding. Considering the client's prenatal history and age, what does the nurse recognize as the greatest risk for the client at this time? a) Hypertension. b) Pregnancy loss. c) Premature birth. d) Preterm labor.

Pregnancy loss. Correct Explanation: The client's advanced maternal age (pregnancy in a woman 35 years or older) increases her risk for pregnancy loss. Hypertension, preterm labor, and prematurity are risks as this pregnancy continues. Her greatest risk at 13 weeks' gestation is losing this pregnancy

A woman in labor has sharp fundal pain accompanied by slight vaginal bleeding. Which of the following would be the most likely cause of these symptoms? a) Preterm labor that was undiagnosed. b) Premature separation of the placenta. c) Placenta previa obstructing the cervix. d) Possible fetal death or injury.

Premature separation of the placenta. Explanation: Premature separation of the placenta begins with sharp fundal pain, usually followed by vaginal bleeding. Placenta previa usually produces painless bleeding; Preterm labor contractions are more often described as cramping. Possible fetal death or injury does not present with sharp fundal pain. It is usually painless.

A client at 27 weeks' gestation is admitted to the obstetric unit after reporting headaches and edema of her hands. Review of the prenatal notes reveals blood pressure consistently above 136/90 mm Hg. The nurse anticipates the health care provider will prescribe magnesium sulfate to accomplish which primary goal? Reverse edema Prevent maternal seizures Decrease protein in urine Decrease blood pressure

Prevent maternal seizures

A client at 27 weeks' gestation is admitted to the obstetric unit after reporting headaches and edema of her hands. Review of the prenatal notes reveals blood pressure consistently above 136/90 mm Hg. The nurse anticipates the health care provider will prescribe magnesium sulfate to accomplish which primary goal? Decrease protein in urine Prevent maternal seizures Reverse edema Decrease blood pressure

Prevent maternal seizures

A client at 27 weeks' gestation is admitted to the obstetric unit after reporting headaches and edema of her hands. Review of the prenatal notes reveals blood pressure consistently above 136/90 mm Hg. The nurse anticipates the health care provider will prescribe magnesium sulfate to accomplish which primary goal? Decrease blood pressure Decrease protein in urine Prevent maternal seizures Reverse edema

Prevent maternal seizures Explanation: The primary therapy goal for any client with preeclampsia is to prevent maternal seizures. Use of magnesium sulfate is the drug therapy of choice for severe preeclampsia and is only used to manage and attempt to prevent progression to eclampsia. Magnesium sulfate therapy does not have as a primary goal of decreasing blood pressure, decreasing protein in the urine, or reversing edema.

The nurse is caring for a client with preeclampsia and understands the need to auscultate this client's lung sounds every 2 hours. Why would the nurse do this?

Pulmonary edema

You are caring for a patient with preeclampsia. You know that you need to auscultate this patient's lung sounds every two hours. Why would you do this? a) Pulmonary atelectasis b) Pulmonary emboli c) Pulmonary hypertension d) Pulmonary edema

Pulmonary edema Correct Explanation: In the hospital, monitor blood pressure at least every four hours for mild preeclampsia and more frequently for severe disease. In addition, it is important to auscultate the lungs every two hours. Adventitious sounds may indicate, developing pulmonary edema

A client with a history of cervical insufficiency is seen for reports of pink-tinged discharge and pelvic pressure. The primary care provider decides to perform a cervical cerclage. The nurse teaches the client about the procedure. Which client response indicates that the teaching has been effective?

Purse-string sutures are placed in the cervix to prevent it from dilating The cerclage, or purse string suture, is inserted into the cervix to prevent preterm cervical dilation (dilatation) and pregnancy loss. Staples, glue, or a cervical cap will not prevent the cervix from dilating.

A client with a history of cervical insufficiency is seen for reports of pink-tinged discharge and pelvic pressure. The primary care provider decides to perform a cervical cerclage. The nurse teaches the client about the procedure. Which client response indicates that the teaching has been effective?

Purse-string sutures are placed in the cervix to prevent it from dilating." Explanation: The cerclage, or purse string suture, is inserted into the cervix to prevent preterm cervical dilation (dilatation) and pregnancy loss. Staples, glue, or a cervical cap will not prevent the cervix from dilating.

When teaching a client newly diagnosed with type 1 diabetes about exercise, it is important to include which instructions? Select all that apply.

Reduce the insulin dose before planned exercise. Eat a carbohydrate snack before beginning to exercise. Exercise should occur within 2 hours of eating. Stop activity if signs and symptoms of hypoglycemia develop. Reducing the insulin dose before exercising may be the best way to prevent hypoglycemia. Eating a carbohydrate snack before beginning to exercise is advised, especially if the blood glucose level is less than 100 mg/dL. If possible, exercise should occur within 2 hours of eating to prevent hypoglycemia.

A woman at 28 weeks' gestation has been hospitalized with moderate bleeding that is now stabilizing. The nurse performs a routine assessment and notes the client sleeping, lying on the back, and electronic fetal heart rate (FHR) monitor showing gradually increasing baseline with late decelerations. Which action will the nurse perform first? -Administer oxygen to the client. -Notify the health care provider. -Reposition the client to left side. -Increase the rate of IV fluids.

Reposition the client to left side. The fetus is showing signs of fetal distress. The immediate treatment is putting the client in a side-lying position to ensure adequate perfusion to the fetus. After placing the client on the side, the nurse should re-assess the FHR and determine if oxygen, IV fluids, and calling the health care provider are needed.

A woman at 28 weeks' gestation has been hospitalized with moderate bleeding that is now stabilizing. The nurse performs a routine assessment and notes the client sleeping, lying on the back, and electronic fetal heart rate (FHR) monitor showing gradually increasing baseline with late decelerations. Which action will the nurse perform first?

Reposition the client to left side. Explanation: The fetus is showing signs of fetal distress. The immediate treatment is putting the client in a side-lying position to ensure adequate perfusion to the fetus. After placing the client on the side, the nurse should re-assess the FHR and determine if oxygen, IV fluids, and calling the health care provider are needed.

A woman at 28 weeks' gestation has been hospitalized with moderate bleeding that is now stabilizing. The nurse performs a routine assessment and notes the client sleeping, lying on the back, and electronic fetal heart rate (FHR) monitor showing gradually increasing baseline with late decelerations. Which action will the nurse perform first? Administer oxygen to the client. Notify the health care provider. Reposition the client to left side. Increase the rate of IV fluids.

Reposition the client to left side. Explanation: The fetus is showing signs of fetal distress. The immediate treatment is putting the client in a side-lying position to ensure adequate perfusion to the fetus. After placing the client on the side, the nurse should re-assess the FHR and determine if oxygen, IV fluids, and calling the health care provider are needed.

A client at 37 weeks' gestation presents to the emergency department with a BP 150/108 mm Hg, 1+ pedal edema, 1+ proteinuria, and normal deep tendon reflexes. Which assessment should the nurse prioritize as the client is administered magnesium sulfate IV? Urine protein Ability to sleep Hemoglobin Respiratory rate

Respiratory rate

A client at 37 weeks' gestation presents to the emergency department with a BP 150/108 mm Hg, 1+ pedal edema, 1+ proteinuria, and normal deep tendon reflexes. Which assessment should the nurse prioritize as the client is administered magnesium sulfate IV? Hemoglobin Ability to sleep Urine protein Respiratory rate

Respiratory rate

A client at 37 weeks' gestation presents to the emergency department with a BP 150/108 mm Hg, 1+ pedal edema, 1+ proteinuria, and normal deep tendon reflexes. Which assessment should the nurse prioritize as the client is administered magnesium sulfate IV?

Respiratory rate The level of magnesium in therapeutic range is 4 to 8 mg/dL. If magnesium toxicity occurs, one sign in the client will be a decrease in the respiratory rate and a potential respiratory arrest. Respiratory rate will be monitored when on this medication. The client's hemoglobin and ability to sleep are not factors for ongoing assessments for the client on magnesium sulfate. Urinary output is measured hourly on the preeclamptic client receiving magnesium sulfate, but urine protein is not an ongoing assessment.

A patient with preeclampsia is receiving magnesium sulfate. Which of the following nursing assessments should be ongoing while the medication is being administered? a) Respiratory rate. b) Ability to sleep. c) Hemoglobin. d) Urine protein.

Respiratory rate. Explanation: The level of magnesium in therapeutic range is 4 to 8 mg/dL. If magnesium toxicity occurs, one sign in the patient will be a decrease in the respiratory rate and a potential respiratory arrest. Respiratory rate will be monitored when on this medication. The patient's hemoglobin and ability to sleep are not factors for on-going assessments for the patient on magnesium sulfate. Urinary output is measured hourly on the preeclamptic patient receiving magnesium sulfate, but urine protein is not an ongoing assessment. (less)

Which action will the nurse avoid when performing basic care for a newborn male? -Inspecting the genital area for irritated skin -Palpating if testes are descended into the scrotal sac -Determining the location of the urethral opening -Retracting the foreskin over the glans to assess for secretions

Retracting the foreskin over the glans to assess for secretions The foreskin in male newborns does not normally retract and should not be forced. The nurse will inspect the genital area for irritated skin to prevent and/or treat possible skin irritations. The nurse will palpate the testes to determine if the newborn has cryptorchidism. It is important to verify that the urethral opening is at the tip of the glans and not on the dorsal or ventral sides as these would need intervention. This can be accomplished without overmanipulating the foreskin.

incompatibility

Rh ____________ is a condition that develops when a woman with Rh-negative blood type is exposed to Rh-positive blood cells and subsequently develops circulating titers of Rh antibodies.

Which measure would be most effective in preventing isoimmunization during pregnancy? a) RhoGAM administration to Rh-negative women b) Amniocentesis c) Cerclage d) Blood typing of mothers with type A or B blood

RhoGAM administration to Rh-negative women Correct Explanation: Rh incompatibility can be prevented with the use of RhoGAM. Hemolysis associated with ABO incompatibility is limited to mothers with type O blood and their fetuses with type A or B blood. Amniocentesis would be appropriate for treatment of polyhydramnios, not isoimmunization. Cerclage is a treatment for cervical insufficiency

A. Any time there is a pregnancy with the chance of maternal and fetal blood mixing, RhoGAM is needed to prevent sensitization or antibody production.

RhoGAM is given to Rh-negative women to prevent maternal sensitization. In addition to pregnancy, Rh-negative women would also receive this medication after which of the following? a. Therapeutic or spontaneous abortion b. Head injury from a car accident c. Blood transfusion after a hemorrhage d. Unsuccessful artificial insemination procedure

A woman in labor is at risk for abruptio placentae. Which of the following assessments would most likely lead you to suspect that this has happened? a) Pain in a lower quadrant and increased pulse rate. b) An increased blood pressure and oliguria. c) Sharp fundal pain and discomfort between contractions. d) Painless vaginal bleeding and a fall in blood pressure.

Sharp fundal pain and discomfort between contractions. Explanation: An abruptio placentae refers to premature separation of the placenta from the uterus. As the placenta loosens, it causes sharp pain. Labor begins with a continuing nagging sensation. Painless vaginal bleeding and a fall in blood pressure are indicative of placenta previa. Pain in a lower quadrant and increased pulse rate are indicative of an ectopic pregnancy. Hypertension and oliguria are indicative of preeclampsia.

The nurse is monitoring a pregnant patient who is receiving intravenous magnesium sulfate for eclampsia. During the last assessment, the nurse was unable to elicit a patellar reflex. What should the nurse do?

Stop the current infusion. When infusing magnesium sulfate, the nurse should stop the infusion if deep tendon reflexes are absent. Checking the fetal heart rate and measuring blood pressure could waste time and provide the patient with more magnesium sulfate. The infusion rate should not be increased because this could lead to cardiac dysrhythmias and respiratory depression.

When assessing a pregnant woman with vaginal bleeding, which finding would lead the nurse to suspect an inevitable abortion? a) Closed cervical os b) No passage of fetal tissue c) Strong abdominal cramping d) Slight vaginal bleeding

Strong abdominal cramping Correct Explanation: Strong abdominal cramping is associated with an inevitable abortion. Slight vaginal bleeding early in pregnancy and a closed cervical os are associated with a threatened abortion. With an inevitable abortion, passage of the products of conception may occur. No fetal tissue is passed with a threatened abortion

A pregnant woman at 12 weeks' gestation comes to the office reporting she has begun minimal fresh vaginal spotting. She is distressed because her primary care provider indicates after examining her that they will "wait and see." Which response would be most appropriate from the nurse in answering this client's concerns? -Advise her to ask for a second care provider opinion. -Tell her that medication to prolong a 12-week pregnancy usually is not advised. -Explain that "wait and see" means that her care provider wants her to maintain strict bed rest. -Suggest she take an over-the-counter tocolytic just to feel secure.

Tell her that medication to prolong a 12-week pregnancy usually is not advised. Because many early pregnancy losses occur as the result of chromosome abnormalities, an aggressive approach to prolong these is not usually recommended. It would not be appropriate for the nurse to suggest an over-the-counter tocolytic, nor to tell the client that the care provider meant something else such as maintaining strict bed rest. Advising the client to seek a second opinion would not change the end results.

A pregnant woman at 12 weeks' gestation calls you because she has begun minimal fresh vaginal spotting. She is distressed because her physician says she is not going to do anything for her but "wait and see." Which of the following would you suggest? a) Suggest she take an over-the-counter tocolytic just to feel secure. b) Explain that her doctor meant for her to maintain strict bed rest by "wait and see." c) Tell her that medication to prolong a 12-week pregnancy usually is not advised. d) Advise her to ask for a second physician opinion.

Tell her that medication to prolong a 12-week pregnancy usually is not advised. Explanation: Because many early pregnancy losses occur as the result of chromosome abnormalities, an aggressive approach to prolong these is not usually recommended

A client is 20 weeks pregnant. At a prenatal visit, the nurse begins the prenatal assessment. Which finding would necessitate calling the primary care provider to assess the client?

The client has pink vaginal discharge and pelvic pressure.

A client is 20 weeks pregnant. At a prenatal visit, the nurse begins the prenatal assessment. Which finding would necessitate calling the primary care provider to assess the client?

The client has pink vaginal discharge and pelvic pressure. Cervical dilatation usually occurs painlessly, and often the first symptom is pink vaginal discharge or increased pelvic pressure, which then is followed by rupture of membranes and discharge of the amniotic fluid. The other answers are nonthreatening signs and symptoms.

In returning to the hospital floor after a weekend off, the nurse takes over care of a pregnant client who is resting in a darkened room. The client is receiving betamethasone and magnesium sulfate. What could the nurse deduce from those findings?

The client is suffering from severe preeclampsia, and the care team is attempting to prevent advancement of the disorder to eclampsia; they are attempting to help the baby's lungs mature quickly so that they can have the baby be born as soon as possible. The administration of magnesium sulfate is to relax the skeletal muscles and raise the threshold for a seizure. The administration of the betamethasone is to try and hasten the maturity of the fetus' lungs for birth. This woman is in advanced preeclampsia and must be monitored for progression to eclampsia. The scenario described does not indicate a client with hypertension who may be discharged home once the condition is under control. A woman in eclampsia would either be seizing or comatose, not resting in a quiet room. Symptoms of mild preeclampsia are limited to slightly elevated blood pressure and small amounts of protein in the urine. Betamethasone may be indicated at this time.

first

The most common cause for __________________ trimester abortions is fetal genetic abnormalities, usually unrelated to the mother.

a. Obtain RhoGAM at 28 weeks' gestation

The nurse is caring for an Rh-negative nonimmunized client at 14 weeks' gestation. What information would the nurse provide to the client? a. Obtain RhoGAM at 28 weeks' gestation b. Consume a well-balanced, nutritional diet c. Avoid sexual activity until after 28 weeks d. Undergo periodic transvaginal ultrasounds

b. Epigastric pain d. Upper right quadrant pain e. Hyperbilirubinemia

The nurse is required to assess a client for HELLP syndrome. Which are the signs and symptoms of this condition? (Select all that apply.) a. Blood pressure higher than 160/110 b. Epigastric pain c. Oliguria d. Upper right quadrant pain e. Hyperbilirubinemia

d. assessing the amount and color of the bleeding

The nurse is required to assess a pregnant client who is reporting vaginal bleeding. Which nursing action is the priority? a. determining the amount of funneling b. monitoring uterine contractility c. assessing signs of shock d. assessing the amount and color of the bleeding

A young woman presents at the emergency department reporting lower abdominal cramping and spotting at 12 weeks' gestation. The primary care provider performs a pelvic examination and finds that the cervix is closed. What does the care provider suspect is the cause of the cramps and spotting? Threatened abortion Cervical insufficiency Ectopic pregnancy Habitual abortion

Threatened abortion

Which of the following statements is CORRECT about type 1 and type 2 diabetes?

Type 2 can occur at any age and accounts for 90% to 95% of diagnosed cases of diabetes. Type 2 diabetes, previously referred to as non-insulin-dependent diabetes mellitus or adult-onset diabetes, can occur at any age and accounts for 90% to 95% of diagnosed cases of diabetes. Type 2 diabetes is a slowly progressive disease characterized by a combination of insulin resistance and relative insulin deficiency. Type 1 diabetes, formerly known as insulin-dependent diabetes mellitus or juvenile diabetes, is characterized by the absence of insulin. In type 1 diabetes, there is a relatively abrupt and absolute end to insulin production.

A woman at 8 weeks' gestation is admitted for ectopic pregnancy. She is asking why this has occurred. The nurse knows that which of the following is a known risk factor for ectopic pregnancy? a) Use of IUD for contraception b) High number of pregnancies c) Use of oral contraceptives d) Multiple gestation pregnancy

Use of IUD for contraception Correct Explanation: Use of an IUD with progesterone has a known increased risk for development of ectopic pregnancies. The nurse needs to complete a full history of the patient to determine if she had any other risk factors for an ectopic pregnancy. Adhesions, scarring, and narrowing of the tubal lumen may block the zygote's progress to the uterus. Any condition or surgical procedure that can injure a fallopian tube increases the risk. Examples include salpingitis, infection of the fallopian tube, endometriosis, history of prior ectopic pregnancy, any type of tubal surgery, congenital malformation of the tube, and multiple elective abortions. Conditions that inhibit peristalsis of the tube can result in tubal pregnancy. Hormonal factors may play a role because tubal pregnancy occurs more frequently in women who take fertility drugs or who use progesterone intrauterine contraceptive devices (IUDs). A high number of pregnancies, multiple gestation pregnancy, and the use of oral contraceptives are not known risk factors for ectopic pregnancy

The nurse is caring for a multigravid who experienced a placental abruption 4 hours ago. For which potential situation will the nurse prioritize assessment? Uterine atony Blood incompatibilities Hypertensive crisis Maternal blood loss

Uterine atony

You are the post-partum nurse caring for a woman who delivered four hours ago. During delivery this patient had placental abruption. She is now at risk for postpartum hemorrhage because of what? a) Blood incompatabilities b) Retained products of conception c) Uterine atony d) Renal failure

Uterine atony Correct Explanation: After delivery, the woman who has had abruption placentae requires close monitoring for postpartum hemorrhage because she is at risk for uterine atony. (

A. Nonsensitized Rh-negative mother with a Rh-negative newborn

Which of the following women should receive RhoGAM postpartum? a. Nonsensitized Rh-negative mother with a Rh-negative newborn b. Nonsensitized Rh-negative mother with a Rh-positive newborn c. Sensitized Rh-negative mother with a Rh-positive newborn d. Sensitized Rh-negative mother with a Rh-negative newborn

The nurse is transcribing messages from the answering service. Which phone message should the nurse return first?

a 35-year-old, 21-week G3P2 client with blood pressure of 160/110 mm Hg, blurred vision, and whose last blood pressure was 143/99 mm Hg and urine dipstick showed a +2 proteinuria The nurse should call the at-risk 35-year-old client first. She is 21 weeks and has symptoms (blurred vision) of preeclampsia. She also had an increase of protein in her urine (2+) and a 15% increase in her BP. The nurse will need more information to determine if the 38-week client may be in the early stages of labor, and if the 31-week client with flank pain has a kidney infection. The client with malaise and rhinitis will need to talk to the nurse last to find out what over-the-counter medication she is able to take.

A nurse is assessing pregnant clients for the risk of placenta previa. Which client faces the greatest risk for this condition?

a client who had a myomectomy to remove fibroids A previous myomectomy to remove fibroids can be associated with the cause of placenta previa. Risk factors also include advanced maternal age (greater than 30 years old). A structurally defective cervix cannot be associated with the cause of placenta previa. However, it can be associated with the cause of cervical insufficiency. Alcohol ingestion is not a risk factor for developing placenta previa but is associated with abruption placenta.

placenta accreta

a condition where the placenta attaches itself too deeply into the wall of the uterus. According to the literature, a cesarean birth increases the possibility of a future placenta accreta; the more cesarean births that are done, the greater the incidence.

The nurse must stress to which client being discharged for home the critical need to return for monthly follow-up visits?

a woman who has experienced a molar pregnancy. Molar pregnancies can indicate the possibility of developing malignancy. The woman will need close observation and follow-up for the year following the diagnosis. Follow-up visits after an ectopic pregnancy or a complete spontaneous abortion are typically scheduled at six weeks, not monthly. A woman who is Rh negative does not need a follow-up visit because of her Rh status.

A client at 11 weeks' gestation experiences pregnancy loss. The client asks the nurse if the bleeding and cramping that occurred during the miscarriage were caused by working long hours in a stressful environment. What is the most appropriate response from the nurse? a) "I can understand your need to find an answer to what caused this. Let's talk about this further." b) "It is hard to know why a woman bleeds during early pregnancy." c) "Your spontaneous bleeding is not work-related." d) "Something was wrong with the fetus."

a) "I can understand your need to find an answer to what caused this. Let's talk about this further." Rationale: Talking with the client may assist her to explore her feelings. She and her family may search for a cause for a spontaneous early bleeding so they can plan for future pregnancies. Even with modern technology and medical advances, however, a direct cause cannot usually be determined.

A client experiences a threatened abortion. She is concerned about losing the pregnancy and asks what activity level she should maintain. What is the most appropriate response from the nurse? a) "Restrict your physical activity to moderate bedrest." b) "There is no research evidence that I can recommend to you." c) "Carry on with the activity you engaged in before this happened." d) "Strict bedrest is necessary so as not to jeopardize this pregnancy."

a) "Restrict your physical activity to moderate bedrest." Rationale: With a threatened abortion, moderate bedrest and supportive care are recommended. Regular physical activity may increase the chances of miscarriage. Strict bedrest is not necessary. Activity restrictions are part of standard medical management.

A primipara at 36 weeks gestation is being monitored in the prenatal clinic for risk of preeclampsia. Which of the following signs or symptoms is the priority concern for the nurse? a) A dipstick value of 2+ for protein. b) Weight gain of 1.2 lb during the past 1 week. c) A systolic blood pressure increase of 10 mm hg. d) Pedal edema.

a) A dipstick value of 2+ for protein. Rationale: The increasing amount of protein in the urine is a concern the preeclampsia may be progressing to severe preeclampsia. The woman needs further assessment by the health care provider. Dependent edema may be seen in the majority of pregnant women and is not an indicator of progression from preeclampsia to eclampsia. Weight gain is no longer considered an indicator for the progression of preeclampsia. A systolic blood pressure increase is not the highest priority concern for the nurse, since we have no idea what the baseline blood pressure was.

A pregnant client is admitted to a health care unit with disseminated intravascular coagulation (DIC). Which of the following orders is the nurse most likely to receive regarding the therapy for such a client? a) Administer cryoprecipitate and platelets b) Give each unit of blood to raise the hematocrit by 3 g/dL c) Administer a ratio of 1 unit of blood to 4 units of frozen plasma d) Aim at keeping the client's hematocrit above 20%

a) Administer cryoprecipitate and platelets. Rationale: In a pregnant client with DIC, the nurse may be ordered to administer cryoprecipitate and platelets. Whole blood does not contain clotting factors. Therefore a ratio of 4 units of blood to 1 unit of fresh frozen plasma, and not 1 unit of blood to 4 units of frozen plasma, should be considered. The nurse should aim at maintaining the client's hematocrit above 30% and not just 20%. The nurse should expect one unit of blood to increase the hematocrit by 1.5 g/dL, and not 3g/dL.

A patient is admitted at 22 weeks gestation with advanced cervical dilatation to 5 centimeters, cervical insufficiency, and a visible amniotic sac at the cervical opening. What is the primary goal for this patient at this point? a) Bed rest to maintain pregnancy as long as possible b) Deliver vaginally c) Notification of social support for loss of pregnancy d) Education on causes of cervical insufficiency for the future

a) Bed rest to maintain pregnancy as long as possible Rationale: At 22 weeks gestation, the fetus is not viable. The woman would be placed on bed rest, total, with every attempt made to halt any further progression of dilatation as long as possible. You would not want to deliver this fetus vaginally at this stage of gestation. It is not your responsibility to notify the patient's social support of a possible loss of the pregnancy. It is not appropriate at this time to educate the mother on causes of cervical insufficiency for future pregnancies.

You are caring for a young woman who is in her 10th week of gestation. She comes into the clinic complaining of vaginal bleeding. Which assessment finding best correlates with a diagnosis of hydatidiform mole? a) Dark red, "clumpy" vaginal discharge b) Painful uterine contractions and nausea c) Bright red painless vaginal bleeding d) Brisk deep tendon reflexes and shoulder pain

a) Dark red, "clumpy" vaginal discharge Rationale: If a complete molar pregnancy continues into the second trimester undetected, other signs and symptoms appear. The woman often presents with complaints of dark to bright red vaginal bleeding and pelvic pain. Infrequently, she will report passage of grapelike vesicles.

A client reports to her obstetrician complaining of a significant amount of bright red, painless vaginal bleeding. A sonogram reveals that her placenta has implanted low in the uterus and is partially covering the cervical os. Which of the following immediate care measures should the nurse initiate? (Select all that apply.) a) Determine from the client the time the bleeding began and about how much blood has been lost b) Attach external monitoring equipment to record fetal heart sounds c) Continue to assess blood pressure every 5 to 15 minutes d) Perform a pelvic examination e) Place the woman on bed rest in a side-lying position f) Obtain baseline vital signs

a) Determine from the client the time the bleeding began and about how much blood has been lost b) Attach external monitoring equipment to record fetal heart sounds c) Continue to assess blood pressure every 5 to 15 minutes e) Place the woman on bed rest in a side-lying position f) Obtain baseline vital signs Rationale: With the exception of performing a pelvic examination, all of the answers are appropriate immediate care measures for the client with placenta previa. Never attempt a pelvic or rectal examination with painless bleeding late in pregnancy because any agitation of the cervix when there is a placenta previa might tear the placenta further and initiate massive hemorrhage, possibly fatal to both mother and child.

A woman with an incomplete abortion is to receive misoprostol. The nurse understands that the rationale for administering this drug is to: a) Ensure passage of all the products of conception b) Suppress the immune response to prevent isoimmunization c) Halt the progression of the abortion d) Alleviate strong uterine cramping

a) Ensure passage of all the products of conception Rationale: Misoprostol is used to stimulate uterine contractions and evacuate the uterus after an abortion to ensure passage of all the products of conception. Rh (D) immunoglobulin is used to suppress the immune response and prevent isoimmunization.

Which of the following would the nurse most likely include when planning the care for a woman requiring hospitalization for hyperemesis gravidarum? Select all that apply. a) Maintaining NPO status for the first day or two b) Administering antiemetic agents c) Preparing the woman for insertion of a feeding tube d) Obtaining baseline blood electrolyte levels e) Monitoring intake and output

a) Maintaining NPO status for the first day or two b) Administering antiemetic agents d) Obtaining baseline blood electrolyte levels e) Monitoring intake and output Rationale: When hospitalization is necessary, oral food and fluids are withheld to allow the gut to rest. Antiemetic agents are ordered to help control nausea and vomiting. The woman is likely to be dehydrated, so the nurse would obtain baseline blood electrolyte levels and administer intravenous fluid and electrolyte replacement therapy as indicated. Once the nausea and vomiting subside, oral food and fluids are gradually reintroduced. Total parenteral nutrition or a feeding tube is used to prevent malnutrition only if the client does not improve with these interventions.

A pregnant client has been admitted with complaints of brownish vaginal bleeding. On examination there is an elevated hCG level, absent fetal heart sounds and a discrepancy between the uterine size and the gestational age. The nurse interprets these findings to suggest which of the following? a) Molar pregnancy b) Abruption of placenta c) Placenta previa d) Ectopic pregnancy

a) Molar pregnancy Rationale: The client is most likely experiencing molar pregnancy. In molar pregnancy, there is an abnormal proliferation and eventual degeneration of the trophoblastic villi. The signs and symptoms of molar pregnancy include brownish vaginal bleeding, elevated hCG levels, discrepancy between the uterine size and the gestational age, and absent fetal heart sounds. Abruption of placenta is characterized by premature separation of the placenta. Ectopic pregnancy is a condition where there is implantation of the blastocyst outside the uterus. In placenta previa the placental attachment is at the lower uterine segment.

A 24-year-old woman presents at the emergency room with abdominal pain, vaginal bleeding, and fatigue. What should the nurse's first action be? a) Monitor the patient for signs of shock. b) Ascertain from the patient if she has been sexually active to rule out pregnancy. c) Prepare for IV blood transfusion. d) Assume a pregnancy and prepare to determine the viability of the fetus.

a) Monitor the patient for signs of shock. Rationale: Anytime a patient presents with hemorrhage, the initial nursing consideration is assessment and evaluation of shock. This is also the case in the pregnant patient. If the mother is in shock the fetus will have lack of oxygen. Monitor the maternal status first. Ascertaining sexual activity to rule out pregnancy or assuming a pregnancy and preparing to determine the viability of the fetus is not the initial nursing consideration for this patient. The scenario does not indicate the need for a blood transfusion.

The following hourly assessments are obtained by the nurse on a patient with preeclampsia receiving Magnesium Sulfate: 97.3, P88, R10, blood pressure 148/110. What other priority physical assessment by the nurse should be implemented to assess for potential toxicity? a) Reflexes b) Magnesium sulfate level c) Lung sounds d) Oxygen saturation

a) Reflexes Rationale: Reflex assessment is part of the standard assessment for patients on magnesium sulfate. The first change when developing magnesium toxicity may be a decrease in reflex activity. The health care provider needs to be notified immediately. A change in lung sounds and oxygen saturation are not indicative of magnesium sulfate toxicity. Hourly blood draws to gain information on the magnesium sulfate level are not indicated.

Sometimes an ectopic pregnancy occurs outside the woman's uterus. This usually occurs in one of the fallopian tubes. If the embryo continues to grow, it may rupture the tube. What are the signs and symptoms of a ruptured fallopian tube? a) Shoulder pain b) Unilateral abdominal pain c) Rectal pain d) Bilateral abdominal pain

a) Shoulder pain Rationale: Rarely, a woman may present with late signs, such as shoulder pain or hypovolemic shock. These signs are associated with tubal rupture, which occurs when the pregnancy expands beyond the tube's ability to stretch.

A client reporting she recently had a positive pregnancy test has reported to the emergency department stating one-sided lower abdominal pain. The health care provider has prescribed a series of tests. Which test will provide the most definitive confirmation of an ectopic pregnancy?

abdominal ultrasound An ectopic pregnancy refers to the implantation of the fertilized egg in a location other than the uterus. Potential sites include the cervix, uterus, abdomen, and fallopian tubes. The confirmation of the ectopic pregnancy can be made by an ultrasound, which would confirm that there was no uterine pregnancy. A quantitative hCG level may be completed in the diagnostic plan. hCG levels in an ectopic pregnancy are traditionally reduced. While this would be an indication, it would not provide a positive confirmation. The qualitative hCG test would provide evidence of a pregnancy, but not the location of the pregnancy. A pelvic exam would be included in the diagnostic plan of care. It would likely show an enlarged uterus and cause potential discomfort to the client but would not be a definitive finding.

A pregnant client is brought to the health care facility with signs of premature rupture of the membranes (PROM). Which conditions and complications are associated with PROM? Select all that apply. placenta previa abruptio placenta preterm labor prolapsed cord spontaneous abortion

abruptio placenta preterm labor prolapsed cord

risk factors for placenta previa

advancing maternal age (more than 35 years), previous cesarean birth, multiparity, uterine insult or injury, cocaine use, prior placenta previa, infertility treatment, Asian ethnic background, multiple gestations, previous induced surgical abortion, smoking, previous myomectomy to remove fibroids, short interval between pregnancies, and hypertension or diabetes.

A nurse in the maternity triage unit is caring for a client with a suspected ectopic pregnancy. Which nursing intervention should the nurse perform first?

assess the client vital signs A suspected ectopic pregnancy can put the client at risk for hypovolemic shock. The assessment of vital signs should be performed first, followed by any procedures to maintain the ABCs. Providing emotional support would also occur, as would obtaining a surgical consent, if needed, but these are not first steps.

A nurse in the maternity triage unit is caring for a client with a suspected ectopic pregnancy. Which nursing intervention should the nurse perform first?

assess the clients vital signs A suspected ectopic pregnancy can put the client at risk for hypovolemic shock. The assessment of vital signs should be performed first, followed by any procedures to maintain the ABCs. Providing emotional support would also occur, as would obtaining a surgical consent, if needed, but these are not first steps.

A pregnant woman is admitted to the hospital with a diagnosis of placenta previa. Which action would be the priority for this woman on admission? performing a vaginal examination to assess the extent of bleeding helping the woman remain ambulatory to reduce bleeding assessing fetal heart tones by use of an external monitor assessing uterine contractions by an internal pressure gauge

assessing fetal heart tones by use of an external monitor

A pregnant woman is admitted to the hospital with a diagnosis of placenta previa. Which action would be the priority for this woman on admission?

assessing fetal heart tones by use of an external monitor Not disrupting the placenta is a prime responsibility. An internal monitor, a vaginal examination, and remaining ambulatory could all do this and thus are contraindicated.

The nurse is required to assess a pregnant client who is reporting vaginal bleeding. Which nursing action is the priority?

assessing the amount and color of the bleeding

The nurse is required to assess a pregnant client who is reporting vaginal bleeding. Which nursing action is the priority? monitoring uterine contractility assessing the amount and color of the bleeding determining the amount of funneling assessing signs of shock

assessing the amount and color of the bleeding

The nurse is required to assess a pregnant client who is reporting vaginal bleeding. Which nursing action is the priority? assessing signs of shock assessing the amount and color of the bleeding determining the amount of funneling monitoring uterine contractility

assessing the amount and color of the bleeding

The nurse is required to assess a pregnant client who is reporting vaginal bleeding. Which nursing action is the priority?

assessing the amount and color of the bleeding When the woman arrives and is admitted, assessing her vital signs, the amount and color of the bleeding, and current pain rating on a scale of 1 to 10 are the priorities. Assessing the signs of shock, monitoring uterine contractility, and determining the amount of funneling are not priority assessments when a pregnant woman complaining of vaginal bleeding is admitted to the hospital.

The nurse is developing a nutrition plan for a client with nephropathy as a complication of type 2 diabetes. Which intervention does the nurse include in the plan?

avoid high protein intake Key components of the diet include managing blood sugar through carbohydrate control, managing blood pressure by decreasing sodium intake, and reducing protein intake.

A client is 11 weeks pregnant after many years trying to conceive. After arriving home from a normal prenatal visit, she experiences mild cramping and has a gush of bright red vaginal bleeding. She calls the nurse and reports having soaked a pad with fresh blood in fewer than 30 minutes. The uterine cramping is worsening. What is the most appropriate response from the nurse? a) "This is nothing to worry about. Many women bleed during pregnancy." b) "You need to seek immediate attention from the primary care provider." c) "I am sorry. There is nothing you can do, because you are likely miscarrying." d) "Lie down and call your health care provider tomorrow if symptoms continue."

b) "You need to seek immediate attention from the primary care provider." Rationale: Pregnancy loss during the early weeks of pregnancy may seem like a heavy menstrual period. A primary care provider should assess blood loss of this amount with or without uterine cramping as soon as possible.

A postpartum mother has the following lab data recorded: RH negative and rubella titer is positive. What is the appropriate nursing intervention? a) Assess the rubella of the baby b) Administer RhoGam within 72 hours. c) Administer rubella vaccine before discharge. d) Assess the RH of the baby.

b) Administer RhoGam within 72 hours. Rationale: The rubella is a virus and the mother has a positive titer indicating she is immune, this is important data for prenatal care and only has a bearing if the patient were negative. The RH is negative for the mother and the infant status is unknown, to protect future pregnancies the mother should be given RhoGam. It would not be appropriate to administer the rubella vaccine, assess the rubella titer of the baby, or assess the Rh status of the baby.

A woman has been diagnosed as having pregnancy-induced hypertension. Which of the following is the most typical symptom of this? a) Increased perspiration b) Blood pressure elevation c) Weight loss d) Susceptibility to infection

b) Blood pressure elevation Rationale: The symptom of hypertension of pregnancy is blood pressure elevation (140/90 mm Hg).

When providing counseling on early pregnancy loss, the nurse should include what as the most common cause for spontaneous abortion? a) Maternal smoking b) Chromosomal defect c) Lack of prenatal care d) The age of the mother

b) Chromosomal defect Rationale: The most common cause for the loss of a fetus in the first trimester is associated with a genetic defect or chromosomal defect. There is nothing that can be done and the mother should feel no fault. The nurse needs to educate the parents to speak with a health care provider for further information and questions related to genetic testing. Early pregnancy loss is not associated with maternal smoking, lack of prenatal care, or the age of the mother.

Which of the following nursing diagnoses would be most appropriate for a woman diagnosed with pregnancy-induced hypertension? a) Risk for injury related to fetal distress b) Deficient fluid volume related to vasospasm of arteries c) Imbalanced nutrition related to decreased sodium levels d) Ineffective tissue perfusion related to poor heart contraction

b) Deficient fluid volume related to vasospasm of arteries Rationale: Hypertension of pregnancy is caused by vascular spasm. This leads to increased blood pressure and edema. Extensive edema leads to a deficiency of fluid volume.

A client visits a health care facility with complaints of amenorrhea for 10 weeks, fatigue, and breast tenderness. Which of the following additional signs and symptoms suggest the presence of molar pregnancy? Select all that apply. a) Whitish discharge from the vagina b) Elevated hCG levels c) Hyperemesis gravidarum d) Dyspareunia e) Absence of fetal heart sound

b) Elevated hCG levels c) Hyperemesis gravidarum e) Absence of fetal heart sound Rationale: The signs and symptoms of molar pregnancy include an elevated hCG level, absence of fetal heart sounds, and hyperemesis gravidarum. Whitish discharge from the vagina and dyspareunia (painful sexual intercourse) are seen in cases of infection, and not in molar pregnancy. In molar pregnancy a brownish vaginal bleeding is seen.

A novice nurse asks to be assigned to the least complex antepartum patient. Which of the following conditions would necessitate the least complex care requirements? a) Pre-ecalmpsia. b) Gestational hypertension. c) Placenta previa. d) Abruptio placenta.

b) Gestational hypertension. Rationale: Hypertensive disorders represent the most common complication of pregnancy. Gestational hypertension is elevated blood pressure without proteinuria, other signs of pre-eclampsia, or pre-existing hypertension. Abruptio placenta (separation of the placenta from the uterine wall), placenta previa (placenta covering the cervical os), and pre-eclampsia are high-risk, potentially life-threatening conditions for the fetus and mother during labor and birth.

To which of the following patients being discharged for home must the nurse stress that it is absolutely critical the patient return for monthly follow-up visits? A woman who: a) Has experienced a complete spontaneous abortion b) Has experienced a molar pregnancy c) Has experienced an ectopic pregnancy d) Is Rh negative

b) Has experienced a molar pregnancy Rationale: Molar pregnancies can indicate the possibility of developing malignancy. The woman will need close observation and follow-up for the year following the diagnosis. Follow-up visits after an ectopic pregnancy or a complete spontaneous abortion are typically scheduled at six weeks, not monthly. A woman who is Rh negative does not need a follow-up visit because of her Rh status.

A woman in week 35 of her pregnancy with severe hydramnios is admitted to the hospital. The nurse recognizes that which of the following is the biggest concern regarding this client? a) Development of gestational trophoblastic disease b) Preterm rupture of membranes followed by preterm birth c) Development of eclampsia d) Hemorrhaging

b) Preterm rupture of membranes followed by preterm birth Rationale: Even with precautions, in most instances of hydramnios, there will be preterm rupture of the membranes because of excessive pressure, followed by preterm birth. The other answers are not as big of concerns as preterm birth, in this situation.

A nurse is explaining to a group of nursing students that eclampsia or seizures in pregnant women are preceded by an acute increase in maternal blood pressure. Which of the following are features of an acute increase in blood pressure? Select all that apply. a) Auditory hallucinations b) Proteinuria c) Hypereflexia d) Blurring of vision e) Hyperglycemia

b) Proteinuria c) Hypereflexia d) Blurring of vision Rationale: Eclampsia is usually preceded by an acute increase in blood pressure as well as worsening signs of multi-organ system failure seen as increasing liver enzymes, proteinuria, and symptoms such as blurred vision and hyperreflexia. Hyperglycemia and auditory hallucinations are not seen with an acute increase in maternal BP.

A patient with preeclampsia is receiving magnesium sulfate. Which of the following nursing assessments should be ongoing while the medication is being administered? a) Hemoglobin. b) Respiratory rate. c) Urine protein. d) Ability to sleep.

b) Respiratory rate. Rationale: The level of magnesium in therapeutic range is 4 to 8 mg/dL. If magnesium toxicity occurs, one sign in the patient will be a decrease in the respiratory rate and a potential respiratory arrest. Respiratory rate will be monitored when on this medication. The patient's hemoglobin and ability to sleep are not factors for on-going assessments for the patient on magnesium sulfate. Urinary output is measured hourly on the preeclamptic patient receiving magnesium sulfate, but urine protein is not an ongoing assessment.

A woman and her partner present at her first antenatal obstetrics appointment. She is 6 weeks pregnant. Her blood tests show that she is Rh negative although she has not previously known this. She has no other children but suspects that she miscarried early in a pregnancy 2 years previously. She felt fine afterward so she never received medical attention for that suspected episode. Her partner reports that he is also Rh negative. What action will the nurse be likely to take? a) If an antibody screen is positive, the nurse will administer RhoGam to the patient. b) The nurse will perform all normal procedures and follow-up tasks because the Rh status of the patient and the child are not a concern at this time. c) The nurse will arrange for an amniocentesis to assess hemolytic disease in the fetus. d) If an antibody screen is negative, the nurse will administer RhoGam to the patient.

b) The nurse will perform all normal procedures and follow-up tasks because the Rh status of the patient and the child are not a concern at this time. Rationale: If the woman is Rh negative and her partner is Rh negative the fetus will also be Rh negative and the woman will not require treatment with RhoGam. Therefore options A, B, and C are incorrect.

A young woman presents at the emergency department with complaints of lower abdominal cramping and spotting at 12 weeks' gestation. The physician performs a pelvic examination and finds that the cervix is closed. What does the physician suspect is the cause of the cramps and spotting? a) Cervical insufficiency b) Threatened abortion c) Habitual abortion d) Ectopic pregnancy

b) Threatened abortion Rationale: Spontaneous abortion occurs along a continuum: threatened, inevitable, incomplete, complete, missed. The definition of each category is related to whether or not the uterus is emptied, or for how long the products of conception are retained.

A woman you care for has an Rh-negative blood type. Following the birth of her infant, you administer her RhIG (D immune globulin). The purpose of this is to a) promote maternal D antibody formation. b) prevent maternal D antibody formation. c) stimulate maternal D immune antigens. d) prevent fetal Rh blood formation.

b) prevent maternal D antibody formation. Rationale: Because RhIG contains passive antibodies, the solution will prevent the woman from forming long-lasting antibodies.

A client is admitted at 22 weeks' gestation with advanced cervical dilatation to 5 centimeters, cervical insufficiency, and a visible amniotic sac at the cervical opening. What is the primary goal for this client at this point?

bed rest to maintain pregnancy as long as possible At 22 weeks' gestation, the fetus is not viable. The woman would be placed on bed rest, total, with every attempt made to halt any further progression of dilatation as long as possible. The nurse would not want this fetus to be born vaginally at this stage of gestation. It is not the nurse's responsibility to notify the client's social support of a possible loss of the pregnancy. It is not appropriate at this time to educate the mother on causes of cervical insufficiency for future pregnancies.

Select the statement by the pregnant woman that indicates the need for more teaching about preeclampsia. a) "I will count my baby's movements twice a day." b) "If I have a slight headache I'll take Tylenol and call if unrelieved." c) "If I have changes in my vision, I will lie down and rest." d) "I will weigh myself every morning after voiding before breakfast."

c) "If I have changes in my vision, I will lie down and rest." Rationale: Changes in the visual field may indicate the patient has moved from preeclampsia to severe preeclampisa and is at risk for developing a seizure due to changes in cerebral blood flow. The patient would require immediate assessment and intervention. Options B, C, and D would not indicate that more teaching about preeclampsia is indicated.

The nurse is caring for a woman who is 28 weeks pregnant and has been hospitalized with moderate bleeding which seems to be stabilizing; she is asleep on her side. While checking the fetal monitor tape the nurse notices that the baseline has gradually increased with late decelerations. What is the first action the nurse will take? a) Reposition the mother in a semirecumbent position on her back. b) Notify the physician. c) Administer oxygen to the mother. d) Decrease the rate of IV fluids.

c) Administer oxygen to the mother. Rationale: The fetus is showing signs of fetal distress. The immediate treatment is using oxygen, putting the patient in a side lying position, and increasing IV fluids. The interventions are to improve fetal circulation. Therefore options B, C, and D are incorrect answers.

Which of the following changes in B/P assessment findings during the second trimester indicate the highest risk for preeclampsia. a) Initial BP 120/80, current BP 130/88. b) Initial BP 110/60, current BP 112/86. c) Initial BP 100/70, current BP 140/90. d) Initial BP 140/85, current BP 130/80.

c) Initial BP 100/70, current BP 140/90. Rationale: A rise in blood pressure to above 140/90 is a concern the patient may be developing preeclampsia. The B/Ps noted in options A, C, and D are not indicative of developing preeclampsia, so these are incorrect responses.

Which of the following would be the physiologic basis for a placenta previa? a) A loose placental implantation b) A placenta with multiple lobes c) Low placental implantation d) A uterus with a midseptum

c) Low placental implantation Rationale: The cause of placenta previa is usually unknown, but for some reason the placenta is implanted low instead of high on the uterus.

Which of the following would the nurse prepare to administer if ordered as treatment for an unruptured ectopic pregnancy? a) Promethazine b) Oxytocin c) Methotrexate d) Ondansetron

c) Methotrexate Rationale: Methotrexate, a folic acid antagonist that inhibits cell division in the developing embryo, is most commonly used to treat ectopic pregnancy. Oxytocin is used to stimulate uterine contractions and would be inappropriate for use with an ectopic pregnancy. Promethazine and ondansetron are antiemetics that may be used to treat hyperemesis gravidarum.

A woman is 9 weeks gestation and admitted to the obstetrical unit for hyperemesis gravidarum. The highest priority intervention the nurse should anticipate is which of the following? a) Administration of antiemetics b) IV rehydration c) NPO for 24 hours d) Bed rest with bathroom privileges

c) NPO for 24 hours Rationale: The initial intervention to have priority is to stop all food and fluids by mouth until the vomiting has stopped. The next steps will depend on the severity of the hyperemesis gravidarum. Use of medications should be based on the severity of the hyperemesis gravidarum and only under orders as most medications are pregnancy category C. IV rehydration may be ordered by the physician.

When assessing a woman with an ectopic pregnancy, which of the following would lead the nurse to suspect that the tube has ruptured? a) Breast tenderness b) Vaginal spotting c) Referred shoulder pain d) Nausea

c) Referred shoulder pain Rationale: Referred pain to the shoulder area indicates bleeding into the abdomen caused by phrenic nerve irritation when a tubal pregnancy ruptures. Vaginal spotting, nausea, and breast tenderness are typical findings of early pregnancy and an unruptured ectopic pregnancy.

Which measure would be most effective in preventing isoimmunization during pregnancy? a) Cerclage b) Blood typing of mothers with type A or B blood c) RhoGAM administration to Rh-negative women d) Amniocentesis

c) RhoGAM administration to Rh-negative women Rationale: Rh incompatibility can be prevented with the use of RhoGAM. Hemolysis associated with ABO incompatibility is limited to mothers with type O blood and their fetuses with type A or B blood. Amniocentesis would be appropriate for treatment of polyhydramnios, not isoimmunization. Cerclage is a treatment for cervical insufficiency.

A 45-year-old pregnant woman with type O blood has had an amniocentesis to rule out Down syndrome. The fetus has type AB blood. What can the nurse warn the patient is a likely outcome if some fetal blood mixed with maternal blood during the procedure? a) The baby will develop hemolytic anemia. b) Preterm delivery c) The baby will have postdelivery jaundice. d) Placental abruption

c) The baby will have postdelivery jaundice. Rationale: The infant and mother have ABO incompatibility. The result is a development of antibodies and breaking down of the blood resulting in jaundice in the infant after delivery. The mixing of some fetal blood with maternal blood during the amniocentesis would not cause placental abruption or preterm delivery. Hemolytic anemia is caused by Rh incompatibility, not ABO incompatibility.

A woman in week 16 of her pregnancy calls her physician's office to report that she has experienced abdominal cramping, cervical dilation, vaginal spotting, and the passing of tissue. The nurse instructs the client to bring the passed tissue to the hospital with her. What is the correct rationale for this instruction? a) To determine the stage of development of the fetus b) To determine whether infection is present c) To determine whether gestational trophoblastic disease is present d) To determine whether the fetus is viable

c) To determine whether gestational trophoblastic disease is present Rationale: Gestational trophoblastic disease is abnormal proliferation and then degeneration of the trophoblastic villi. The embryo fails to develop beyond a primitive start. Abnormal trophoblast cells must be identified because they are associated with choriocarcinoma, a rapidly metastasizing malignancy. This is why it is important for any woman who begins to miscarry at home to bring any clots or tissue passed to the hospital with her. The presence of clear fluid-filled cysts changes the diagnosis from a simple miscarriage to gestational trophoblastic disease. The client is not instructed to bring in passed tissue to determine whether infection is present or the fetus is viable or to determine the stage of development of the fetus.

A woman at 8 weeks' gestation is admitted for ectopic pregnancy. She is asking why this has occurred. The nurse knows that which of the following is a known risk factor for ectopic pregnancy? a) High number of pregnancies b) Multiple gestation pregnancy c) Use of IUD for contraception d) Use of oral contraceptives

c) Use of IUD for contraception Rationale: Use of an IUD with progesterone has a known increased risk for development of ectopic pregnancies. The nurse needs to complete a full history of the patient to determine if she had any other risk factors for an ectopic pregnancy. Adhesions, scarring, and narrowing of the tubal lumen may block the zygote's progress to the uterus. Any condition or surgical procedure that can injure a fallopian tube increases the risk. Examples include salpingitis, infection of the fallopian tube, endometriosis, history of prior ectopic pregnancy, any type of tubal surgery, congenital malformation of the tube, and multiple elective abortions. Conditions that inhibit peristalsis of the tube can result in tubal pregnancy. Hormonal factors may play a role because tubal pregnancy occurs more frequently in women who take fertility drugs or who use progesterone intrauterine contraceptive devices (IUDs). A high number of pregnancies, multiple gestation pregnancy, and the use of oral contraceptives are not known risk factors for ectopic pregnancy.

The guidelines for carbohydrate counting for diabetes mellitus include all of the following EXCEPT:

carbohydrate counting includes unlimited intake of saturated fat and cholesterol Limiting saturated fat and using polyunsaturated fats is encouraged.

A pregnant client at 20 weeks' gestation arrives at the health care facility reporting excessive vaginal bleeding and no fetal movements. Which assessment finding would the nurse anticipate in this situation?

cervical incompetence This client has reported symptoms of a spontaneous abortion or miscarriage of the second trimester. Miscarriages in the second trimester are most often related to maternal factors such as cervical incompetence, congenital or acquired anomaly of the uterine cavity, hypothyroidism, diabetes mellitus, chronic nephritis, use of crack cocaine, inherited and acquired thrombophilias, lupus, polycystic ovary syndrome, severe hypertension, and acute infection such as rubella virus, cytomegalovirus, herpes simplex virus, bacterial vaginosis, and toxoplasmosis. Cervical incompetence is a condition where there is painless cervical dilatation and results in second trimester fetal loss or can progress to preterm premature rupture of membranes. Ectopic pregnancy, congenital malformations, and placenta previa are not involved in causing second trimester fetal loss. Ectopic pregnancy usually leads to first trimester fetal loss. Placenta previa is a condition in which there is implantation of the placenta to the lower uterine segment. Congenital malformations result in first trimester fetal loss.

A pregnant client with multiple gestation arrives at the maternity clinic for a regular antenatal check up. The nurse would be aware that client is at risk for which perinatal complication? postterm birth maternal hypotension congenital anomalies fetal nonimmune hydrops

congenital anomalies

A pregnant client with multiple gestation arrives at the maternity clinic for a regular antenatal check up. The nurse would be aware that client is at risk for which perinatal complication?

congenital anomalies Multiple gestation involves two or more fetuses. The perinatal complications associated with multiple pregnancy include preterm birth, maternal hypertension and congenital anomalies. Fetal nonimmune hydrops occurs in the infection of pregnant clients with parvovirus. Postterm birth, maternal hypotension, and fetal nonimmune hydrops are not seen as complications of multiple pregnancy.

Type 2 diabetes may go undiagnosed for years. This statement is:

correct Because hyperglycemia develops gradually in type 2 diabetes and is often not severe enough for patients to recognize any of the classic diabetes symptoms, type 2 diabetes may go undiagnosed for years. Many patients will have already developed complications by the time of diagnosis (Ahmad & Crandall, 2010). Many patients will have already developed complications by the time of diagnosis.

People with diabetes should limit their intake of saturated fat and cholesterol because their risk of coronary heart disease is very high. This statement is:

correct People with diabetes are advised to limit their intake of saturated fat to less than 7% of total calories, minimize their intake of trans fat, and consume less than 200 mg of cholesterol daily.

Alcohol is more likely to cause hypoglycemia when consumed without food rather than with food. This statement is:

correct People with diabetes should not drink alcohol because it raises blood glucose levels very quickly. Any alcohol that is consumed should be consumed with food to reduce the risk of nighttime hypoglycemia.

Which of the following is the best question the nurse can ask a woman who is leaving the hospital after experiencing a complete spontaneous abortion? a) "Did you know that 75 percent of women who are trying to get pregnant experience spontaneous abortions like you have?" b) "Are you going to try again?" c) "May I give you some resources that you can use to try to stop smoking?" d) "Do you have someone to talk to or may I give you the names and numbers for some possible grief counselors?"

d) "Do you have someone to talk to or may I give you the names and numbers for some possible grief counselors?" Rationale: When a woman has a spontaneous abortion one important consideration is the emotional needs of the woman once she is home. She may not want to talk about the loss for a period of time, but the nurse needs to determine her support system for the future. Asking the woman if she is "going to try again" is an inappropriate question for the nurse to ask, and diminishes the experience of having a spontaneous abortion. Giving the woman statistical information on spontaneous abortions is not appropriate when this patient needs support and caring concern. Offering to give the patient resources to aid in smoking cessation is not addressed in the scenario so this is an inappropriate response.

During a routine prenatal visit, a client is found to have proteinuria and a blood pressure rise to 140/90 mm Hg. The nurse recognizes that the client has which of the following conditions? a) Eclampsia b) Severe pre-eclampsia c) Gestational hypertension d) Mild pre-eclampsia

d) Mild pre-eclampsia Rationale: A woman is said to have gestational hypertension when she develops an elevated blood pressure (140/90 mm Hg) but has no proteinuria or edema. If a seizure from gestational hypertension occurs, a woman has eclampsia, but any status above gestational hypertension and below a point of seizures is pre-eclampsia. A woman is said to be mildly pre-eclamptic when she has proteinuria and a blood pressure rise to 140/90 mm Hg, taken on two occasions at least 6 hours apart. A woman has passed from mild to severe pre-eclampsia when her blood pressure rises to 160 mm Hg systolic and 110 mm Hg diastolic or above on at least two occasions 6 hours apart at bed rest (the position in which blood pressure is lowest) or her diastolic pressure is 30 mm Hg above her prepregnancy level. Marked proteinuria, 3+ or 4+ on a random urine sample or more than 5 g in a 24-hour sample, and extensive edema are also present. A woman has passed into eclampsia when cerebral edema is so acute a grand-mal seizure (tonic-clonic) or coma has occurred.

A 32-year-old gravida 3 para 2 at 36 weeks' gestation comes to the obstetric department reporting abdominal pain. Her blood pressure is 164/90 mm/Hg, her pulse is 100 beats per minute, and her respirations are 24 per minute. She is restless and slightly diaphoretic with a small amount of dark red vaginal bleeding. What assessment should the nurse make next? a) Obtain a voided urine specimen and determine blood type. b) Measure fundal height. c) Check deep tendon reflexes. d) Palpate the fundus and check fetal heart rate.

d) Palpate the fundus and check fetal heart rate. Rationale: The classic signs of abruption placentea are pain, dark red vaginal bleeding, a rigid, board-like abdomen, hypertonic labor, and fetal distress.

A woman develops HELLP syndrome. During labor, which of the following orders would you question? a) Assess her blood pressure every 15 minutes. b) Assess the urine output every hour. c) Urge her to lie on her left side during labor. d) Prepare her for epidural anesthesia.

d) Prepare her for epidural anesthesia. Rationale: A consequence of the HELLP syndrome is poor blood coagulation. Epidural anesthesia is not recommended when blood coagulation is in doubt.

When assessing a pregnant woman with vaginal bleeding, which finding would lead the nurse to suspect an inevitable abortion? a) No passage of fetal tissue b) Slight vaginal bleeding c) Closed cervical os d) Strong abdominal cramping

d) Strong abdominal cramping Rationale: Strong abdominal cramping is associated with an inevitable abortion. Slight vaginal bleeding early in pregnancy and a closed cervical os are associated with a threatened abortion. With an inevitable abortion, passage of the products of conception may occur. No fetal tissue is passed with a threatened abortion.

In returning to the hospital floor after a weekend off, the nurse takes over care of a pregnant patient who is resting in a darkened room. The patient is receiving betamethasone and magnesium sulfate. What could the nurse deduce from those findings? a) The patient is suffering from eclampsia and the care team is attempting to prevent stroke and induce labor. b) The patient is suffering from mild preeclampsia and the care team is attempting to stabilize her and the baby before discharging her to home. c) The patient is suffering from hypertension and the care team is trying to lower her blood pressure so that she may return home until the baby is full term. d) The patient is suffering from severe preeclampsia and the care team is attempting to prevent advancement of the disorder to eclampsia; they are attempting to help the baby's lungs mature quickly so that they can deliver as soon as possible.

d) The patient is suffering from severe preeclampsia and the care team is attempting to prevent advancement of the disorder to eclampsia; they are attempting to help the baby's lungs mature quickly so that they can deliver as soon as possible. Rationale: The administration of magnesium sulfate is to relax the skeletal muscles and raise the threshold for a seizure. The administration of the betamethasone is to try and hasten the maturity of the fetus' lungs for delivery. This woman is in advanced preeclampsia and must be monitored for progression to eclampsia. The scenario described does not indicate a patient with hypertension who may be discharged home once the condition is under control. A woman in eclampsia would either be seizing or comatose, not resting in a quiet room. Symptoms of mild preeclampsia are limited to slightly elevated blood pressure and small amounts of protein in the urine. Betamethasone may be indicated at this time.

The nurse through assessment can best differentiate between placenta previa and abruptio placentae by which of the following signs and or symptoms. a) Bleeding amount and consistency. b) Low back pain. c) Shape of the abdomen. d) Uterine tone and contractions of the uterus.

d) Uterine tone and contractions of the uterus. Rationale: With placenta previa the bleeding is often bright red and painless, with abruptio placentae the bleeding is usually dark and painful. The uterus is firm and hard and painful with the abruption; the uterus is often soft and lacks tone with the previa. The contractions of the uterus, low back pain, or the shape of the abdomen do not help to distinguish between placenta previa and abruption placentae.

A woman who is Rh negative asks you how many children she will be able to have before Rh incompatibility causes them to die in utero. Your best response would be that a) she will have to ask her physician. b) no more than three children is recommended. c) only her next child will be affected. d) as long as she receives RhIG, there is no limit.

d) as long as she receives RhIG, there is no limit. Rationale: Because RhIG supplies passive antibodies, it prevents the woman from forming antibodies. Without antibodies that could affect the fetus, the woman could have as many children as she wants.

A nurse is assessing a pregnant client for the possibility of preexisting conditions that could lead to complications during pregnancy. The nurse suspects that the woman is at risk for hydramnios based on which preexisting condition?

diabetes

A nurse is assessing a pregnant client for the possibility of preexisting conditions that could lead to complications during pregnancy. The nurse suspects that the woman is at risk for hydramnios based on which preexisting condition? diabetes hypertension late maternal age isoimmunization

diabetes

A nurse is assessing a pregnant client for the possibility of preexisting conditions that could lead to complications during pregnancy. The nurse suspects that the woman is at risk for hydramnios based on which preexisting condition? -diabetes -hypertension -late maternal age -isoimmunization

diabetes Approximately 18% of all women with diabetes will develop hydramnios during their pregnancy. Hydramnios occurs in approximately 2% of all pregnancies and is associated with fetal anomalies of development.

A woman is being closely monitored and treated for severe preeclampsia with magnesium sulfate. Which finding would alert the nurse to the development of magnesium toxicity in this client?

diminished reflexes

A woman with severe preeclampsia is receiving magnesium sulfate. The woman's serum magnesium level is 9.0 mEq/L. Which finding would the nurse most likely note? diminished reflexes elevated liver enzymes seizures serum magnesium level of 6.5 mEq/L

diminished reflexes

A woman is being closely monitored and treated for severe preeclampsia with magnesium sulfate. Which finding would alert the nurse to the development of magnesium toxicity in this client?

diminished reflexes Diminished or absent reflexes occur when a client develops magnesium toxicity. Elevated liver enzymes are unrelated to magnesium toxicity and may indicate the development of HELLP syndrome. The onset of seizure activity indicates eclampsia. A serum magnesium level of 6.5 mEq/L would fall within the therapeutic range of 4 to 7 mEq/L.

A woman is being closely monitored and treated for severe preeclampsia with magnesium sulfate. Which finding would alert the nurse to the development of magnesium toxicity in this client?

diminshed reflexes

A pregnant woman is being evaluated for HELLP. The nursereviews the client's diagnostic test results. Which result would the nurse interpret as helping to confirm this diagnosis?

elevated LDH

A nurse is conducting a refresher program for a group of perinatal nurses. Part of the program involves a discussion of HELLP. The nurse determines that the group needs additional teaching when they identify which aspect as a part of HELLP? -elevated lipoproteins -hemolysis -liver enzyme elevation -low platelet count

elevated lipoproteins The acronym HELLP represents hemolysis, elevated liver enzymes, and low platelets. This syndrome is a variant of preeclampsia/eclampsia syndrome that occurs in 10% to 20% of clients whose diseases are labeled as severe.

A nurse is conducting a refresher program for a group of perinatal nurses. Part of the program involves a discussion of HELLP. The nurse determines that the group needs additional teaching when they identify which aspect as a part of HELLP?

elevated lipoproteins The acronym HELLP represents hemolysis, elevated liver enzymes, and low platelets. This syndrome is a variant of preeclampsia/eclampsia syndrome that occurs in 10% to 20% of clients whose diseases are labeled as severe.

The nurse is assisting in planning a community diabetes prevention program. Which of the following information should the nurse recommend?

engage in heavy physical activity for at least 30 minutes The goal for weight loss is from 5% to 15% of initial weight, with a target of 1 to 2 pounds per week. Moderate physical activity is advised along with an increased intake of fruit, vegetables, and high-fiber foods.

A woman with a recent incomplete abortion is to receive therapeutic misoprostol. The nurse understands that the rationale for administering this drug is to:

ensure passage of all the products of conception.

A woman with a recent incomplete abortion is to receive therapeutic misoprostol. The nurse understands that the rationale for administering this drug is to: ensure passage of all the products of conception. suppress the immune response to prevent isoimmunization. alleviate strong uterine cramping. halt the progression of the abortion.

ensure passage of all the products of conception.

A woman with an incomplete abortion is to receive misoprostol. The woman asks the nurse, "Why am I getting this drug?" The nurse responds to the client, integrating understanding that this drug achieves which effect

ensures passage of all the products of conception

A woman with an incomplete abortion is to receive misoprostol. The woman asks the nurse, "Why am I getting this drug?" The nurse responds to the client, integrating understanding that this drug achieves which effect? ensures passage of all the products of conception alleviates strong uterine cramping suppresses the immune response to prevent isoimmunization halts the progression of the abortion

ensures passage of all the products of conception

A woman with an incomplete abortion is to receive misoprostol. The woman asks the nurse, "Why am I getting this drug?" The nurse responds to the client, integrating understanding that this drug achieves which effect? ensures passage of all the products of conception alleviates strong uterine cramping suppresses the immune response to prevent isoimmunization halts the progression of the abortion

ensures passage of all the products of conception Explanation: Misoprostol is used to stimulate uterine contractions and evacuate the uterus after an abortion to ensure passage of all the products of conception. Rho(D) immune globulin is used to suppress the immune response and prevent isoimmunization.

The nurse is required to assess a client for HELLP syndrome. Which are the signs and symptoms of this condition? Select all that apply. epigastric pain blood pressure higher than 160/110 mm Hg hyperbilirubinemia oliguria upper right quadrant pain

epigastric pain hyperbilirubinemia upper right quadrant pain

A woman at 9 weeks' gestation was unable to control the nausea and vomiting of hyperemesis gravidarum through conservative measures at home. With nausea and vomiting becoming severe, the woman was omitted to the obstetrical unit. Which action should the nurse prioritize? instruct on NPO status administration of antiemetics bed rest with bathroom privileges establish IV for rehydration

establish IV for rehydration

A woman at 9 weeks' gestation was unable to control the nausea and vomiting of hyperemesis gravidarum through conservative measures at home. With nausea and vomiting becoming severe, the woman was omitted to the obstetrical unit. Which action should the nurse prioritize? -bed rest with bathroom privileges -instruct on NPO status -establish IV for rehydration -administration of antiemetics

establish IV for rehydration With severe nausea and vomiting the client may be dehydrated upon coming to hospital for assistance, so establishing an IV line is the priority intervention. This will also allow for hydration, and if needed, the administration of an antiemetic to bypass the gastrointestinal tract. Although the nurse will explain the NPO status to the client (so that vomiting may be brought under control) and the likelihood of being placed on bed rest with bathroom privileges, these teaching are not the priority.

A woman at 9 weeks' gestation was unable to control the nausea and vomiting of hyperemesis gravidarum through conservative measures at home. With nausea and vomiting becoming severe, the woman was omitted to the obstetrical unit. Which action should the nurse prioritize?

establish IV for rehydration Explanation: With severe nausea and vomiting the client may be dehydrated upon coming to hospital for assistance, so establishing an IV line is the priority intervention. This will also allow for hydration, and if needed, the administration of an antiemetic to bypass the gastrointestinal tract. Although the nurse will explain the NPO status to the client (so that vomiting may be brought under control) and the likelihood of being placed on bed rest with bathroom privileges, these teaching are not the priority.

A woman at 9 weeks' gestation was unable to control the nausea and vomiting of hyperemesis gravidarum through conservative measures at home. With nausea and vomiting becoming severe, the woman was omitted to the obstetrical unit. Which action should the nurse prioritize? bed rest with bathroom privileges instruct on NPO status establish IV for rehydration administration of antiemetics

establish IV for rehydration Explanation: With severe nausea and vomiting the client may be dehydrated upon coming to hospital for assistance, so establishing an IV line is the priority intervention. This will also allow for hydration, and if needed, the administration of an antiemetic to bypass the gastrointestinal tract. Although the nurse will explain the NPO status to the client (so that vomiting may be brought under control) and the likelihood of being placed on bed rest with bathroom privileges, these teachings are not the priority.

A nurse is monitoring a client with PROM who is in labor and observes meconium in the amniotic fluid. What does the observation of meconium indicate?

fetal distress related to hypoxia Explanation: When meconium is present in the amniotic fluid, it typically indicates fetal distress related to hypoxia. Meconium stains the fluid yellow to greenish brown, depending on the amount present. A decreased amount of amniotic fluid reduces the cushioning effect, thereby making cord compression a possibility. A foul odor of amniotic fluid indicates infection. Meconium in the amniotic fluid does not indicate CNS involvement.

A nurse is monitoring a client with PROM who is in labor and observes meconium in the amniotic fluid. What does the observation of meconium indicate? -cord compression -fetal distress related to hypoxia -infection -central nervous system (CNS) involvement

fetal distress related to hypoxia When meconium is present in the amniotic fluid, it typically indicates fetal distress related to hypoxia. Meconium stains the fluid yellow to greenish brown, depending on the amount present. A decreased amount of amniotic fluid reduces the cushioning effect, thereby making cord compression a possibility. A foul odor of amniotic fluid indicates infection. Meconium in the amniotic fluid does not indicate CNS involvement.

A woman at 10 weeks' gestation comes to the clinic for an evaluation. Which assessment finding should the nurse prioritize?

fundal height measures 18 cm A fundal height of 18 cm is larger than expected and should be further investigated for gestational trophoblastic disease (hydatidiform mole). One of the presenting signs is the uterus being larger than expected for date. Mild nausea would be a normal finding at 10 weeks' gestation. Blood pressure of 120/84 mm Hg would not be associated with hydatidiform mole and depending on the woman's baseline blood pressure may be within acceptable parameters for her. Bright red spotting might suggest a spontaneous abortion (miscarriage).

A client has come to the office for a prenatal visit during her 22nd week of gestation. On examination, it is noted that her blood pressure has increased to 138/90 mm Hg. Her urine is negative for proteinuria. The nurse recognizes which factor as the potential cause? preeclampsia chronic hypertension gestational hypertension HELLP

gestational hypertension

A novice nurse asks to be assigned to the least complex antepartum client. Which condition would necessitate the least complex care requirements? preeclampsia abruptio placenta gestational hypertension placenta previa

gestational hypertension

A novice nurse asks to be assigned to the least complex antepartum client. Which condition would necessitate the least complex care requirements?

gestational hypertension Hypertensive disorders represent the most common complication of pregnancy. Gestational hypertension is elevated blood pressure without proteinuria, other signs of preeclampsia, or preexisting hypertension. Placental abruption (abruptio placentae), a separation of the placenta from the uterine wall; placenta previa (placenta covering the cervical os); and preeclampsia are high-risk, potentially life-threatening conditions for the fetus and mother during labor and birth.

A novice nurse asks to be assigned to the least complex antepartum client. Which condition would necessitate the least complex care requirements? preeclampsia placental abruption (abruptio placentae) placenta previa gestational hypertension

gestational hypertension Explanation: Hypertensive disorders represent the most common complication of pregnancy. Gestational hypertension is elevated blood pressure without proteinuria, other signs of preeclampsia, or preexisting hypertension. Placental abruption (abruptio placentae), a separation of the placenta from the uterine wall; placenta previa (placenta covering the cervical os); and preeclampsia are high-risk, potentially life-threatening conditions for the fetus and mother during labor and birth.

The nurse is assessing a client at 12 weeks' gestation at a routine prenatal visit who reports something doesn't feel right. Which assessment findings should the nurse prioritize?

gestational hypertension, hyperemesis gravidarum, absence of FHR Explanation: The early development of gestational hypertension/preeclampsia, hyperemesis gravidarum, and the absence of FHR are suspicious for gestational trophoblastic disease. The elevated levels of hCG lead to the severe morning sickness. There is no fetus, so FHR, quickening, and evidence of a fetal skeleton would not be seen. The abdominal enlargement is greater than expected for pregnancy dates, but hCG, not hPL, levels are increased.

The nurse is assessing a client at 12 weeks' gestation at a routine prenatal visit who reports something doesn't feel right. Which assessment findings should the nurse prioritize? elevated hCG levels, enlarged abdomen, quickening vaginal bleeding, increased hPL levels visible fetal skeleton on ultrasound, absence of quickening, enlarged abdomen gestational hypertension, hyperemesis gravidarum, absence of FHR

gestational hypertension, hyperemesis gravidarum, absence of FHR Explanation: The early development of gestational hypertension/preeclampsia, hyperemesis gravidarum, and the absence of FHR are suspicious for gestational trophoblastic disease. The elevated levels of hCG lead to the severe morning sickness. There is no fetus, so FHR, quickening, and evidence of a fetal skeleton would not be seen. The abdominal enlargement is greater than expected for pregnancy dates, but hCG, not hPL, levels are increased.

A pregnant client has been admitted with reports of brownish vaginal bleeding. On examination there is an elevated hCG level, absent fetal heart sounds, and a discrepancy between the uterine size and the gestational age. The nurse interprets these findings to suggest which condition?

gestational trophoblastic disease The client is most likely experiencing gestational trophoblastic disease or a molar pregnancy. In gestational trophoblastic disease or molar pregnancy, there is an abnormal proliferation and eventual degeneration of the trophoblastic villi. The signs and symptoms of molar pregnancy include brownish vaginal bleeding, elevated hCG levels, discrepancy between the uterine size and the gestational age, and absent fetal heart sounds. Abruption of placenta is characterized by premature separation of the placenta. Ectopic pregnancy is a condition where there is implantation of the blastocyst outside the uterus. In placenta previa the placental attachment is at the lower uterine segment.

A pregnant client has been admitted with reports of brownish vaginal bleeding. On examination, there is an elevated human chorionic gonadotropin (hCG) level, absent fetal heart sounds, and a discrepancy between the uterine size and the gestational age. The nurse interprets these findings to suggest which condition? -ectopic pregnancy -placenta previa -gestational trophoblastic disease -placental abruption (abruption placentae)

gestational trophoblastic disease The client is most likely experiencing gestational trophoblastic disease, or a molar pregnancy. In gestational trophoblastic disease, there is an abnormal proliferation and eventual degeneration of the trophoblastic villi. The signs and symptoms of molar pregnancy include brownish vaginal bleeding, elevated hCG levels, discrepancy between the uterine size and the gestational age, and absent fetal heart sounds. Placental abruption is characterized by premature separation of the placenta. Ectopic pregnancy is a condition where there is implantation of the blastocyst outside the uterus. In placenta previa, the placental attachment is at the lower uterine segment.

A client has been admitted with placental abruption (abruptio placentae). She has lost 1,200 ml of blood, is normotensive, and ultrasound indicates approximately 30% separation. The nurse documents this as which classification of abruptio placentae? -grade 2 -grade 1 -grade 3 -grade 4

grade 2 The classifications for placental abruption (abruptio placentae) are: grade 1 (mild) - minimal bleeding (less than 500 ml), 10% to 20% separation, tender uterus, no coagulopathy, signs of shock or fetal distress; grade 2 (moderate) - moderate bleeding (1,000 to 1,500 ml), 20% to 50% separation, continuous abdominal pain, mild shock, normal maternal blood pressure, maternal tachycardia; grade 3 (severe) - absent to moderate bleeding (more than 1,500 ml), more than 50% separation, profound shock, dark vaginal bleeding, agonizing abdominal pain, decreased blood pressure, significant tachycardia, and development of disseminated intravascular coagulopathy. There is no grade 4.

A client has been admitted with placental abruption (abruptio placentae). She has lost 1,200 ml of blood, is normotensive, and ultrasound indicates approximately 30% separation. The nurse documents this as which classification of abruptio placentae?

grade 2 Explanation: The classifications for placental abruption (abruptio placentae) are: grade 1 (mild) - minimal bleeding (less than 500 ml), 10% to 20% separation, tender uterus, no coagulopathy, signs of shock or fetal distress; grade 2 (moderate) - moderate bleeding (1,000 to 1,500 ml), 20% to 50% separation, continuous abdominal pain, mild shock, normal maternal blood pressure, maternal tachycardia; grade 3 (severe) - absent to moderate bleeding (more than 1,500 ml), more than 50% separation, profound shock, dark vaginal bleeding, agonizing abdominal pain, decreased blood pressure, significant tachycardia, and development of disseminated intravascular coagulopathy. There is no grade 4.

A client in her 20th week of gestation develops HELLP syndrome. What are features of HELLP syndrome? Select all that apply.

hemolysis elevated liver enzymes low platelet count Explanation: The HELLP syndrome is a syndrome involving hemolysis (microangiopathic hemolytic anemia), elevated liver enzymes, and a low platelet count. Hyperthermia and leukocytosis are not features of HELLP syndrome.

A client is suspected of having a ruptured ectopic pregnancy. Which assessment would the nurse identify as the priority? infection edema jaundice hemorrhage

hemorrhage

A nurse is providing a dietary instruction to a client with diabetes who has an uncontrolled blood sugar level. The nurse tells the client to avoid which of the following foods?

high fructose corn syrup People with diabetes should avoid fructose because it causes adverse effects on serum triglycerides and LDL cholesterol.

A nurse is providing care to a client who has been diagnosed with a common benign form of gestational trophoblastic disease. The nurse identifies this as: -hydatidiform mole. -ectopic pregnancy. -placenta accrete. -hydramnios.

hydatidiform mole. Gestational trophoblastic disease comprises a spectrum of neoplastic disorders that originate in the placenta. The two most common types are hydatidiform mole (partial or complete) and choriocarcinoma. Hydatidiform mole is a benign neoplasm of the chorion in which the chorionic villi degenerate and become transparent vesicles containing clear, viscid fluid. Ectopic pregnancy, placenta accreta, and hydramnios fall into different categories of potential pregnancy complications.

A nurse is providing care to a client who has been diagnosed with a common benign form of gestational trophoblastic disease. The nurse identifies this as:

hydratiform mole Gestational trophoblastic disease comprises a spectrum of neoplastic disorders that originate in the placenta. The two most common types are hydatidiform mole (partial or complete) and choriocarcinoma. Hydatidiform mole is a benign neoplasm of the chorion in which the chorionic villi degenerate and become transparent vesicles containing clear, viscid fluid. Ectopic pregnancy, placenta accreta, and hydramnios fall into different categories of potential pregnancy complications.

A woman is admitted with a diagnosis of ectopic pregnancy. For which procedure should the nurse prepare? bed rest for the next 4 weeks intravenous administration of a tocolytic immediate surgery internal uterine monitoring

immediate surgery

A woman is admitted with a diagnosis of ectopic pregnancy. For which procedure should the nurse prepare?

immediate surgery Ectopic pregnancy means an embryo has implanted outside the uterus, usually in the fallopian tube. Surgery is usually necessary to remove the growing structure before the tube ruptures or to repair the tube if rupture has occurred already.

A 24-year-old woman presents with vague abdominal pains, nausea, and vomiting. An urine hCG is positive after the client mentioned that her last menstrual period was 2 months ago. The nurse should prepare the client for which intervention if the transvaginal ultrasound indicates a gestation sac is found in the right lower quadrant?

immediate surgery The client presents with the signs and symptoms of an ectopic pregnancy, which is confirmed by the transvaginal ultrasound. Ectopic pregnancy means an embryo has implanted outside the uterus. Surgery is necessary to remove the growing structure before damage can occur to the woman's internal organs. Bed rest, a tocolytic, and internal uterine monitoring will not correct the situation. The growing structure must be removed surgically.

A client comes to the emergency department with moderate vaginal bleeding. She says, "I have had to change my pad about every 2 hours and it looks like I may have passed some tissue and clots." The woman reports that she is 9 weeks' pregnant. Further assessment reveals the following: Cervical dilation Strong abdominal cramping Low human chorionic gonadotropin (hCG) levels Ultrasound positive for products of conception The nurse suspects that the woman is experiencing which type of spontaneous abortion?

inevitable Based on the assessment findings, the woman is likely experiencing an inevitable abortion characterized by vaginal bleeding, rupture of membranes, cervical dilation, strong abdominal cramping, possible passage of products of conception, and ultrasound and hCG levels indicating pregnancy loss. A threatened abortion is characterized by slight vaginal bleeding, no cervical dilation or change in cervical consistency, mild abdominal cramping, close cervical os, and no passage of fetal tissue. An incomplete abortion is characterized by intense abdominal cramping, heavy vaginal bleeding and cervical dilation with passage of some products of conception. A complete abortion is characterized by a history of vaginal bleeding and abdominal pain along with passage of tissue and subsequent decrease in pain and decrease in bleeding.

The nurse is caring for a client with type 1 diabetes. Because the client is at risk for hypoglycemia, the nurse teaches the client to which of the following?

keep a source of carbohydrate on the nightstand in case hypoglycemia develops during the night An appropriate source of carbohydrate should be on the nightstand in case hypoglycemia develops during the night. The client should avoid extra physical activity at night, as this can cause hypoglycemia. The nurse would not instruct a client to skip insulin intake. Monitoring of the signs and symptoms of hypoglycemia should be consistent. If left unattended, hypoglycemia could cause convulsions, loss of consciousness, or other severe health problems.

The nurse is caring for a pregnant client with severe preeclampsia. Which nursing intervention should a nurse perform to institute and maintain seizure precautions in this client?

keep the suction equipment readily available The nurse should institute and maintain seizure precautions such as padding the side rails and having oxygen, suction equipment, and call light readily available to protect the client from injury. The nurse should provide a quiet, darkened room to stabilize the client. The nurse should maintain the client on complete bed rest in the left lateral lying position and not in a supine position. Keeping the head of the bed slightly elevated will not help maintain seizure precautions.

Which measure would the nurse include in the plan of care for a woman with prelabor rupture of membranes if her fetus's lungs are mature?

labor induction Explanation: With prelabor rupture of membranes (PROM) in a woman whose fetus has mature lungs, induction of labor is initiated. Reducing physical activity, observing for signs of infection, and giving corticosteroids may be used for the woman with PROM when the fetal lungs are immature.

Some women experience a rupture of their membranes before going into true labor. A nurse recognizes that a woman who presents with PPROM has completed how many weeks of gestation?

less than 37 weeks

Some women experience a rupture of their membranes before going into true labor. A nurse recognizes that a woman who presents with preterm premature rupture of membranes (PPROM) has completed how many weeks of gestation?

less than 37 weeks Explanation: Preterm premature rupture of membranes (PPROM) is defined as the rupture of the membranes prior to the onset of labor in a woman who is less than 37 weeks' gestation. PROM (premature rupture of membranes) refers to a woman who is beyond 37 weeks' gestation, has presented with spontaneous rupture of the membranes, and is not in labor.

Some women experience a rupture of their membranes before going into true labor. A nurse recognizes that a woman who presents with preterm premature rupture of membranes (PPROM) has completed how many weeks of gestation? less than 37 weeks less than 38 weeks less than 39 weeks less than 40 weeks

less than 37 weeks Explanation: Preterm premature rupture of membranes (PPROM) is defined as the rupture of the membranes prior to the onset of labor in a woman who is less than 37 weeks' gestation. PROM (premature rupture of membranes) refers to a woman who is beyond 37 weeks' gestation, has presented with spontaneous rupture of the membranes, and is not in labor.

What would be the physiologic basis for a placenta previa? a loose placental implantation low placental implantation a placenta with multiple lobes a uterus with a midseptum

low placental implantation

What would be the physiologic basis for a placenta previa? -a loose placental implantation -low placental implantation -a placenta with multiple lobes -a uterus with a midseptum

low placental implantation The cause of placenta previa is usually unknown, but for some reason the placenta is implanted low instead of high on the uterus.

A nurse is reviewing an article about preterm prelabor rupture of membranes. Which factors would the nurse expect to find placing a woman at high risk for this condition? Select all that apply. low socioeconomic status single gestations smoking high body mass index urinary tract infection

low socioeconomic status smoking urinary tract infection

The nurse is preparing the plan of care for a woman hospitalized for hyperemesis gravidarum. Which interventions would the nurse most likely include? Select all that apply. -maintaining NPO status for the first day or two -preparing the woman for insertion of a feeding tube -administering antiemetic agents -obtaining baseline blood electrolyte levels -monitoring intake and output

maintaining NPO status for the first day or two administering antiemetic agents obtaining baseline blood electrolyte levels monitoring intake and output When hospitalization is necessary, oral food and fluids are withheld to allow the gut to rest. Antiemetic agents are ordered to help control nausea and vomiting. The woman is likely to be dehydrated, so the nurse would obtain baseline blood electrolyte levels and administer intravenous fluid and electrolyte replacement therapy as indicated. Once the nausea and vomiting subside, oral food and fluids are gradually reintroduced. Total parenteral nutrition or a feeding tube is used to prevent malnutrition only if the client does not improve with these interventions.

Which medication will the nurse anticipate the health care provider will prescribe as treatment for an unruptured ectopic pregnancy? -oxytocin -promethazine -ondansetron -methotrexate

methotrexate Methotrexate, a folic acid antagonist that inhibits cell division in the developing embryo, is most commonly used to treat ectopic pregnancy. Oxytocin is used to stimulate uterine contractions and would be inappropriate for use with an ectopic pregnancy. Promethazine and ondansetron are antiemetics that may be used to treat hyperemesis gravidarum. Page 663

Which medication will the nurse anticipate the health care provider will prescribe as treatment for an unruptured ectopic pregnancy? oxytocin promethazine ondansetron methotrexate

methotrexate Explanation: Methotrexate, a folic acid antagonist that inhibits cell division in the developing embryo, is most commonly used to treat ectopic pregnancy. Oxytocin is used to stimulate uterine contractions and would be inappropriate for use with an ectopic pregnancy. Promethazine and ondansetron are antiemetics that may be used to treat hyperemesis gravidarum.

Which medication will the nurse anticipate the health care provider will prescribe as treatment for an unruptured ectopic pregnancy?

methotrexate Methotrexate, a folic acid antagonist that inhibits cell division in the developing embryo, is most commonly used to treat ectopic pregnancy. Oxytocin is used to stimulate uterine contractions and would be inappropriate for use with an ectopic pregnancy. Promethazine and ondansetron are antiemetics that may be used to treat hyperemesis gravidarum.

During a routine prenatal visit, a client is found to have proteinuria and a blood pressure rise to 140/90 mm Hg. The nurse recognizes that the client has which condition?

mild preeclampsia A woman is said to have gestational hypertension when she develops an elevated blood pressure (140/90 mm Hg) but has no proteinuria or edema. If a seizure from gestational hypertension occurs, a woman has eclampsia, but any status above gestational hypertension and below a point of seizures is preeclampsia. A woman is said to be mildly preeclamptic when she has proteinuria and a blood pressure rise to 140/90 mm Hg, taken on two occasions at least 6 hours apart. A woman has passed from mild to severe preeclampsia when her blood pressure rises to 160 mm Hg systolic and 110 mm Hg diastolic or above on at least two occasions 6 hours apart at bed rest (the position in which blood pressure is lowest) or her diastolic pressure is 30 mm Hg above her prepregnancy level. Marked proteinuria, 3+ or 4+ on a random urine sample or more than 5 g in a 24-hour sample, and extensive edema are also present. A woman has passed into eclampsia when cerebral edema is so acute a grand-mal seizure (tonic-clonic) or coma has occurred.

During a routine prenatal visit, a client is found to have proteinuria and a blood pressure rise to 140/90 mm Hg. The nurse recognizes that the client has which condition? -mild preeclampsia -gestational hypertension -severe preeclampsia -eclampsia

mild preeclampsia A woman is said to have gestational hypertension when she develops an elevated blood pressure (140/90 mm Hg) but has no proteinuria or edema. If a seizure from gestational hypertension occurs, a woman has eclampsia, but any status above gestational hypertension and below a point of seizures is preeclampsia. A woman is said to be mildly preeclamptic when she has proteinuria and a blood pressure rise to 140/90 mm Hg, taken on two occasions at least 6 hours apart. A woman has passed from mild to severe preeclampsia when her blood pressure rises to 160 mm Hg systolic and 110 mm Hg diastolic or above on at least two occasions 6 hours apart at bed rest (the position in which blood pressure is lowest) or her diastolic pressure is 30 mm Hg above her prepregnancy level. Marked proteinuria, 3+ or 4+ on a random urine sample or more than 5 g in a 24-hour sample, and extensive edema are also present. A woman has passed into eclampsia when cerebral edema is so acute a grand-mal seizure (tonic-clonic) or coma has occurred. Page 684

A 28-year-old primigravida client with type 2 diabetes mellitus comes to the health care clinic for a routine first trimester visit reporting frequent episodes of fasting blood glucose levels being lower than normal, but glucose levels after meals being higher than normal. What should the nurse point out that these episodes are most likely related to?

normal response to the pregnancy This is a normal response to the pregnancy. During pregnancy, tissues become resistant to insulin to provide sufficient levels of glucose for the growing fetus. This can result in three normally occurring responses: blood glucose levels are lower than normal when fasting; blood glucose levels are higher than normal after meals; and insulin levels are increased after meals. The various hormones will prevent the mother from using most of the insulin produced to allow the extra glucose to get to the growing fetus.

A client with diabetes is having a problem with high lipid levels in the body. Which food item would the nurse encourage the client to eat?

nuts Coconut oil, butter, and meatballs contain high saturated fats, which have long been associated with increased risk of heart disease and stroke for people with diabetes. Nuts contain monounsaturated fatty acid, which is considered to be a "good" fat.

A pregnant client with severe preeclampsia has developed HELLP syndrome. In addition to the observations necessary for preeclampsia, what other nursing intervention is critical for this client?

observation for bleeding Because of the low platelet count associated with this condition, women with HELLP syndrome need extremely close observation for bleeding, in addition to the observations necessary for preeclampsia. Maintaining a patent airway is a critical intervention needed for a client with eclampsia while she is having a seizure. Administration of a tocolytic would be appropriate for halting labor. Monitoring for infection is not a priority intervention in this situation.

Which of the following would be the physiologic basis for a placenta previa? a) A loose placental implantation b) Low placental implantation c) A uterus with a midseptum d) A placenta with multiple lobes

ow placental implantation Correct Explanation: The cause of placenta previa is usually unknown, but for some reason the placenta is implanted low instead of high on the uterus.

A pregnant woman with preeclampsia is to receive magnesium sulfate IV. Which assessment should the nurse prioritize before administering a new dose?

patellar reflex A symptom of magnesium sulfate toxicity is loss of deep tendon reflexes. Assessing for the patellar reflex or ankle clonus before administration is assurance the drug administration will be safe. Assessing the blood pressure, heart rate, or anxiety level would not reveal a potential magnesium toxicity.

A nurse is caring for a client undergoing treatment for ectopic pregnancy. Which symptom is observed in a client if rupture or hemorrhaging occurs before the ectopic pregnancy is successfully treated? fetal distress tetanic contractions painless bright red vaginal bleeding phrenic nerve irritation

phrenic nerve irritation

A nurse is caring for a client undergoing treatment for ectopic pregnancy. Which symptom is observed in a client if rupture or hemorrhaging occurs before the ectopic pregnancy is successfully treated?

phrenic nerve irritation The symptoms if rupture or hemorrhaging occurs before successfully treating the pregnancy are lower abdomen pain, feelings of faintness, phrenic nerve irritation, hypotension, marked abdominal tenderness with distension, and hypovolemic shock. Painless bright red vaginal bleeding occurring during the second or third trimester is the clinical manifestation of placenta previa. Fetal distress and tetanic contractions are not the symptoms observed in a client if rupture or hemorrhaging occurs before successfully treating an ectopic pregnancy.

A nurse is providing care to a multiparous client. The client has a history of cesarean births. The nurse anticipates the need to closely monitor the client for which condition? -placenta accreta -placental abruption (abruptio placentae) -preeclampsia -oligohydramnios

placenta accreta Placenta accreta is a condition where the placenta attaches itself too deeply into the wall of the uterus. According to the literature, a cesarean birth increases the possibility of a future placenta accreta; the more cesarean births that are done, the greater the incidence. Placental abruption (abruptio placentae) occurs when the placenta becomes detached prematurely. Preeclampsia and oligohydramnios are nonrelated to previous cesarean births.

A 44-year old client has type 2 diabetes with cardiovascular complications and must follow a special diet. Which food is allowed in this diet?

plant based proteins Plant-based proteins such as soy and beans are healthy proteins sources that are recommended for diabetes. Intake of saturated-fat foods such as red meat and whole milk is linked to high cholesterol and an increased risk of coronary heart disease and stroke, and this will further aggravate the complications of diabetes. Starchy foods such as pasta are foods to avoid.

A 44-year-old client has lost several pregnancies over the last 10 years. For the past 3 months, she has had fatigue, nausea, and vomiting. She visits the clinic and takes a pregnancy test; the results are positive. Physical examination confirms a uterus enlarged to 13 weeks' gestation; fetal heart tones are heard. Ultrasound reveals that the client is experiencing some bleeding. Considering the client's prenatal history and age, what does the nurse recognize as the greatest risk for the client at this time? premature birth hypertension pregnancy loss preterm labor

pregnancy loss

A 44-year-old client has lost several pregnancies over the last 10 years. For the past 3 months, she has had fatigue, nausea, and vomiting. She visits the clinic and takes a pregnancy test; the results are positive. Physical examination confirms a uterus enlarged to 13 weeks' gestation; fetal heart tones are heard. Ultrasound reveals that the client is experiencing some bleeding. Considering the client's prenatal history and age, what does the nurse recognize as the greatest risk for the client at this time?

pregnancy loss The client's advanced maternal age (pregnancy in a woman 35 years or older) increases her risk for pregnancy loss. Hypertension, preterm labor, and prematurity are risks as this pregnancy continues. Her greatest risk at 13 weeks' gestation is losing this pregnancy.

A woman at 35 weeks' gestation with severe hydramnios is admitted to the hospital. The nurse recognizes that which concern is greatest regarding this client? development of eclampsia hemorrhaging preterm rupture of membranes followed by preterm birth development of gestational trophoblastic disease

preterm rupture of membranes followed by preterm birth

A woman in week 35 of her pregnancy with severe hydramnios is admitted to the hospital. The nurse recognizes that which concern is greatest regarding this client?

preterm rupture of membranes followed by preterm birth

A woman in week 35 of her pregnancy with severe hydramnios is admitted to the hospital. The nurse recognizes that which concern is greatest regarding this client? preterm rupture of membranes followed by preterm birth development of eclampsia hemorrhaging development of gestational trophoblastic disease

preterm rupture of membranes followed by preterm birth

A woman at 35 weeks' gestation with severe hydramnios is admitted to the hospital. The nurse recognizes that which concern is greatest regarding this client? -preterm rupture of membranes followed by preterm birth -development of eclampsia -hemorrhaging -development of gestational trophoblastic disease

preterm rupture of membranes followed by preterm birth Even with precautions, in most instances of hydramnios, there will be preterm rupture of the membranes because of excessive pressure, followed by preterm birth. The other answers are less concerning than preterm birth in this pregnancy.

A woman in week 35 of her pregnancy with severe hydramnios is admitted to the hospital. The nurse recognizes that which concern is greatest regarding this client?

preterm rupture of membranes followed by preterm birth Even with precautions, in most instances of hydramnios, there will be preterm rupture of the membranes because of excessive pressure, followed by preterm birth. The other answers are not as big of concerns as preterm birth, in this situation.

A woman at 35 weeks' gestation with severe hydramnios is admitted to the hospital. The nurse recognizes that which concern is greatest regarding this client?

preterm rupture of membranes followed by preterm birth Explanation: Even with precautions, in most instances of hydramnios, there will be preterm rupture of the membranes because of excessive pressure, followed by preterm birth. The other answers are less concerning than preterm birth in this pregnancy.

A woman at 35 weeks' gestation with severe hydramnios is admitted to the hospital. The nurse recognizes that which concern is greatest regarding this client?

preterm rupture of the membranes followed by preterm birth Even with precautions, in most instances of hydramnios, there will be preterm rupture of the membranes because of excessive pressure, followed by preterm birth. The other answers are less concerning than preterm birth in this pregnancy.

A 24-year-old client presents in labor. The nurse notes there is an order to administer Rho(D) immune globulin after the birth of her infant. When asked by the client the reason for this injection, which reason should the nurse point out?

prevent maternal D antibody formation Because Rho(D) immune globulin contains passive antibodies, the solution will prevent the woman from forming long-lasting antibodies which may harm a future fetus. The administration of Rho(D) immune globulin does not promote the formation of maternal D antibodies; it does not stimulate maternal D immune antigens or prevent fetal Rh blood formation.

A 24-year-old client presents in labor. The nurse notes there is an order to administer RhoGAM after the birth of her infant. When asked by the client the reason for this injection, which reason should the nurse point out?

prevent maternal D antibody formation.

A 24-year-old client presents in labor. The nurse notes there is an order to administer RhoGAM after the birth of her infant. When asked by the client the reason for this injection, which reason should the nurse point out? promote maternal D antibody formation. prevent maternal D antibody formation. stimulate maternal D immune antigens. prevent fetal Rh blood formation.

prevent maternal D antibody formation.

A pregnant client has an Rh-negative blood type. Following the birth of the client's infant, the nurse administers her Rho(D) immune globulin. The purpose of this is to: promote maternal D antibody formation. prevent maternal D antibody formation. stimulate maternal D immune antigens. prevent fetal Rh blood formation.

prevent maternal D antibody formation.

A pregnant client has an Rh-negative blood type. Following the birth of the client's infant, the nurse administers her Rho(D) immune globulin. The purpose of this is to: stimulate maternal D immune antigens. prevent maternal D antibody formation. promote maternal D antibody formation. prevent fetal Rh blood formation.

prevent maternal D antibody formation.

A 24-year-old client presents in labor. The nurse notes there is an order to administer Rho(D) immune globulin after the birth of her infant. When asked by the client the reason for this injection, which reason should the nurse point out? -promote maternal D antibody formation. -prevent maternal D antibody formation. -stimulate maternal D immune antigens. -prevent fetal Rh blood formation.

prevent maternal D antibody formation. Because Rho(D) immune globulin contains passive antibodies, the solution will prevent the woman from forming long-lasting antibodies which may harm a future fetus. The administration of Rho(D) immune globulin does not promote the formation of maternal D antibodies; it does not stimulate maternal D immune antigens or prevent fetal Rh blood formation.

A client for has an Rh-negative blood type. Following the birth of the client's infant, the nurse administers her Rho(D)immune globulin. The purpose of this is to:

prevent maternal D antibody formation. Because Rho(D) immune globulin contains passive antibodies, the solution will prevent the woman from forming long-lasting antibodies.

A client at 27 weeks' gestation is admitted to the obstetric unit after reporting headaches and edema of her hands. Review of the prenatal notes reveals blood pressure consistently above 136/90 mm Hg. The nurse anticipates the health care provider will prescribe magnesium sulfate to accomplish which primary goal?

prevent maternal seizures The primary therapy goal for any client with preeclampsia is to prevent maternal seizures. Use of magnesium sulfate is the drug therapy of choice for severe preeclampsia and is only used to manage and attempt to prevent progression to eclampsia. Magnesium sulfate therapy does not have as a primary goal of decreasing blood pressure, decreasing protein in the urine, or reversing edema.

The nurse is admitting a G3 P2 client at 38 weeks' gestation who arrived reporting painless bleeding from the vagina leading to the diagnosis of placenta previa. When questioned by the client as to what caused this, which most likely factor should the nurse point out in her answer?

previous cesarean birth The risk of placenta previa is greatly increased when a woman has had a previous cesarean delivery due to the scarring of the endometrial lining. Maternal age over 35 years, and not just more than 30 years, is considered another risk factor. Placenta previa is more common among those living in high altitudes not among those living in coastal areas. Obesity is not recognized as a potential risk for this condition. Other risk factors can include uterine insult or injury, cocaine use, prior placenta previa, infertility treatment, multiple gestations, previous induced surgical abortion, smoking, previous myomectomy to remove fibroids, short interval between pregnancies, hypertension, or diabetes.

The nurse is caring for a client with preeclampsia and understands the need to auscultate this client's lung sounds every two hours. Why would the nurse do this?

pulmonary edema In the hospital, monitor blood pressure at least every four hours for mild preeclampsia and more frequently for severe disease. In addition, it is important to auscultate the lungs every two hours. Adventitious sounds may indicate developing pulmonary edema.

When assessing a woman with an ectopic pregnancy, the nurse would suspect that the tube has ruptured based on which finding?

referred shoulder pain

A woman has presented to the emergency department with symptoms that suggest an ectopic pregnancy. Which finding would lead the nurse to suspect that the fallopian tube has ruptured?

referred shoulder pain Referred pain to the shoulder area indicates bleeding into the abdomen caused by phrenic nerve irritation when a tubal pregnancy ruptures. Vaginal spotting, nausea, and breast tenderness are typical findings of early pregnancy and an unruptured ectopic pregnancy.

The following hourly assessments are obtained by the nurse on a client with preeclampsia receiving magnesium sulfate: 97.3° F (36.2° C), pulse 88 beats/min, respirations 10 breaths/min, blood pressure 148/110 mm Hg. What other priority physical assessment by the nurse should be implemented to assess for potential toxicity?

reflexes Reflex assessment is part of the standard assessment for clients on magnesium sulfate. The first change when developing magnesium toxicity may be a decrease in reflex activity. The health care provider needs to be notified immediately. A change in lung sounds and oxygen saturation are not indicative of magnesium sulfate toxicity. Hourly blood draws to gain information on the magnesium sulfate level are not indicated.

The following hourly assessments are obtained by the nurse on a client with preeclampsia receiving magnesium sulfate: 97.3oF (36.2oC), HR 88, RR 12 breaths/min, BP 148/110 mm Hg. What other priority physical assessments by the nurse should be implemented to assess for potential toxicity? lung sounds oxygen saturation reflexes magnesium sulfate level

reflexes Explanation: Reflex assessment is part of the standard assessment for clients on magnesium sulfate. The first change when developing magnesium toxicity may be a decrease in reflex activity. The health care provider needs to be notified immediately. A change in lung sounds and oxygen saturation are not indicative of magnesium sulfate toxicity. Hourly blood draws to gain information on the magnesium sulfate level are not indicated.

The following hourly assessments are obtained by the nurse on a client with preeclampsia receiving magnesium sulfate: 97.3oF (36.2oC), HR 88, RR 12 breaths/min, BP 148/110 mm Hg. What other priority physical assessments by the nurse should be implemented to assess for potential toxicity?

reflexes Reflex assessment is part of the standard assessment for clients on magnesium sulfate. The first change when developing magnesium toxicity may be a decrease in reflex activity. The health care provider needs to be notified immediately. A change in lung sounds and oxygen saturation are not indicative of magnesium sulfate toxicity. Hourly blood draws to gain information on the magnesium sulfate level are not indicated.

A client at 37 weeks' gestation presents to the emergency department with a BP 150/108 mm Hg, 1+ pedal edema, 1+ proteinuria, and normal deep tendon reflexes. Which assessment should the nurse prioritize as the client is administered magnesium sulfate IV? -urine protein -ability to sleep -hemoglobin -respiratory rate

respiratory rate A therapeutic level of magnesium is 4 to 8 mg/dl (1.65 to 3.29 mmol/L). If magnesium toxicity occurs, one sign in the client will be a decrease in the respiratory rate and a potential respiratory arrest. Respiratory rate will be monitored when on this medication. The client's hemoglobin and ability to sleep are not factors for ongoing assessments for the client on magnesium sulfate. Urinary output is measured hourly on the preeclamptic client receiving magnesium sulfate, but urine protein is not an ongoing assessment.

A client at 37 weeks' gestation presents to the emergency department with a BP 150/108 mm Hg, 1+ pedal edema, 1+ proteinuria, and normal deep tendon reflexes. Which assessment should the nurse prioritize as the client is administered magnesium sulfate IV? urine protein ability to sleep hemoglobin respiratory rate

respiratory rate Explanation: A therapeutic level of magnesium is 4 to 8 mg/dl (1.65 to 3.29 mmol/L). If magnesium toxicity occurs, one sign in the client will be a decrease in the respiratory rate and a potential respiratory arrest. Respiratory rate will be monitored when on this medication. The client's hemoglobin and ability to sleep are not factors for ongoing assessments for the client on magnesium sulfate. Urinary output is measured hourly on the preeclamptic client receiving magnesium sulfate, but urine protein is not an ongoing assessment.

A client at 37 weeks' gestation presents to the emergency department with a BP 150/108 mm Hg, 1+ pedal edema, 1+ proteinuria, and normal deep tendon reflexes. Which assessment should the nurse prioritize as the client is administered magnesium sulfate IV?

respiratory rate A therapeutic level of magnesium is 4 to 8 mg/dl (1.65 to 3.29 mmol/L). If magnesium toxicity occurs, one sign in the client will be a decrease in the respiratory rate and a potential respiratory arrest. Respiratory rate will be monitored when on this medication. The client's hemoglobin and ability to sleep are not factors for ongoing assessments for the client on magnesium sulfate. Urinary output is measured hourly on the preeclamptic client receiving magnesium sulfate, but urine protein is not an ongoing assessment.

When assessing a pregnant woman with vaginal bleeding, the nurse would suspect a threatened abortion based on which finding?

slight vaginal bleeding

A 44-year-old client has lost several pregnancies over the last 10 years. For the past 3 months, she has had fatigue, nausea, and vomiting. She visits the clinic and takes a pregnancy test; the results are positive. Physical examination confirms a uterus enlarged to 13 weeks' gestation; fetal heart tones are heard. Ultrasound reveals that the client is experiencing some bleeding. Considering the client's prenatal history and age, what does the nurse recognize as the greatest risk for the client at this time? -premature birth -hypertension -spontaneous abortion (miscarriage) -preterm labor

spontaneous abortion (miscarriage) The client's advanced maternal age (pregnancy in a woman 35 years or older) increases her risk for spontaneous abortion (miscarriage). Hypertension, preterm labor, and prematurity are risks as this pregnancy continues. Her greatest risk at 13 weeks' gestation is losing this pregnancy.

A 44-year-old client has lost several pregnancies over the last 10 years. For the past 3 months, she has had fatigue, nausea, and vomiting. She visits the clinic and takes a pregnancy test; the results are positive. Physical examination confirms a uterus enlarged to 13 weeks' gestation; fetal heart tones are heard. Ultrasound reveals that the client is experiencing some bleeding. Considering the client's prenatal history and age, what does the nurse recognize as the greatest risk for the client at this time?

spontaneous abortion (miscarriage) Explanation: The client's advanced maternal age (pregnancy in a woman 35 years or older) increases her risk for spontaneous abortion (miscarriage). Hypertension, preterm labor, and prematurity are risks as this pregnancy continues. Her greatest risk at 13 weeks' gestation is losing this pregnancy.

A pregnant woman has arrived to the office reporting vaginal bleeding. Which finding during the assessment would lead the nurse to suspect an inevitable abortion?

strong abdominal cramping

A pregnant woman has arrived to the office reporting vaginal bleeding. Which finding during the assessment would lead the nurse to suspect an inevitable abortion? closed cervical os strong abdominal cramping no passage of fetal tissue slight vaginal bleeding

strong abdominal cramping

A pregnant woman has arrived to the office reporting vaginal bleeding. Which finding during the assessment would lead the nurse to suspect an inevitable abortion?

strong abdominal cramping Strong abdominal cramping is associated with an inevitable spontaneous abortion. Slight vaginal bleeding early in pregnancy and a closed cervical os are associated with a threatened abortion. With an inevitable abortion, passage of the products of conception may occur. No fetal tissue is passed with a threatened abortion.

A pregnant woman has arrived to the office reporting vaginal bleeding. Which finding during the assessment would lead the nurse to suspect an inevitable spontaneous abortion (miscarriage)? -strong abdominal cramping -slight vaginal bleeding -closed cervical os -no passage of fetal tissue

strong abdominal cramping Strong abdominal cramping is associated with an inevitable spontaneous abortion (miscarriage). Slight vaginal bleeding early in pregnancy and a closed cervical os are associated with a threatened abortion. With an inevitable abortion, passage of the products of conception may occur. No fetal tissue is passed with a threatened abortion.

A pregnant woman has arrived to the office reporting vaginal bleeding. Which finding during the assessment would lead the nurse to suspect an inevitable spontaneous abortion (miscarriage)?

strong abdominal cramping Explanation: Strong abdominal cramping is associated with an inevitable spontaneous abortion (miscarriage). Slight vaginal bleeding early in pregnancy and a closed cervical os are associated with a threatened abortion. With an inevitable abortion, passage of the products of conception may occur. No fetal tissue is passed with a threatened abortion.

A pregnant woman is diagnosed with abruptio placentae. When reviewing the woman's medical record, the nurse would expect which finding?

sudden dark vaginal bleeding

A pregnant woman is diagnosed with abruptio placentae. When reviewing the woman's medical record, the nurse would expect which finding?

sudden dark, vaginal bleeding The uterus is firm to rigid to the touch with abruptio placentae; it is soft and relaxed with placenta previa. Bleeding associated with abruptio placentae occurs suddenly and is usually dark in color. Bleeding also may not be visible. Bright red vaginal bleeding is associated with placenta previa. Fetal distress or absent fetal heart rate may be noted with abruptio placentae. The woman with abruptio placentae usually experiences constant uterine tenderness on palpation.

The nurse is caring for a woman at 32 weeks' gestation with severe preeclampsia. Which assessment finding should the nurse prioritize after the administration of hydralazine to this client?

tachycardia Hydralazine reduces blood pressure but is associated with adverse effects such as palpitation, tachycardia, headache, anorexia, nausea, vomiting, and diarrhea. It does not cause gastrointestinal bleeding, blurred vision (halos around lights), or sweating. Magnesium sulfate may cause sweating.

A pregnant woman at 12 weeks' gestation comes to the office reporting she has begun minimal fresh vaginal spotting. She is distressed because her primary care provider indicates after examining her that they will "wait and see." Which response would be most appropriate from the nurse in answering this client's concerns?

tell her that medications to prolong a 12 week pregnancy usually is not advised Because many early pregnancy losses occur as the result of chromosome abnormalities, an aggressive approach to prolong these is not usually recommended. It would not be appropriate for the nurse to suggest an over-the-counter tocolytic, nor to tell the client that the care provider meant something else such as maintaining strict bed rest. Advising the client to seek a second opinion would not change the end results.

A woman in week 16 of her pregnancy calls her primary care provider's office to report that she has experienced abdominal cramping, cervical dilation, vaginal spotting, and the passing of tissue. The nurse instructs the client to bring the passed tissue to the hospital with her. What is the correct rationale for this instruction?

to determine whether gestational trophoblastic disease is present Gestational trophoblastic disease is abnormal proliferation and then degeneration of the trophoblastic villi. The embryo fails to develop beyond a primitive start. Abnormal trophoblast cells must be identified because they are associated with choriocarcinoma, a rapidly metastasizing malignancy. This is why it is important for any woman who begins to miscarry at home to bring any clots or tissue passed to the hospital with her. The presence of clear fluid-filled cysts changes the diagnosis from a simple miscarriage to gestational trophoblastic disease. The client is not instructed to bring in passed tissue to determine whether infection is present or the fetus is viable or to determine the stage of development of the fetus.

A woman at 8 weeks' gestation is admitted for ectopic pregnancy. She is asking why this has occurred. The nurse knows that which factor is a known risk factor for ectopic pregnancy?

use of IUD for contraception Use of an IUD with progesterone has a known increased risk for development of ectopic pregnancies. The nurse needs to complete a full history of the client to determine if she had any other risk factors for an ectopic pregnancy. Adhesions, scarring, and narrowing of the tubal lumen may block the zygote's progress to the uterus. Any condition or surgical procedure that can injure a fallopian tube increases the risk. Examples include salpingitis, infection of the fallopian tube, endometriosis, history of prior ectopic pregnancy, any type of tubal surgery, congenital malformation of the tube, and multiple elective abortions. Conditions that inhibit peristalsis of the tube can result in tubal pregnancy. Hormonal factors may play a role because tubal pregnancy occurs more frequently in women who take fertility drugs or who use progesterone intrauterine contraceptive devices (IUDs). A high number of pregnancies, multiple gestation pregnancy, and the use of oral contraceptives are not known risk factors for ectopic pregnancy.

A woman at 8 weeks' gestation is admitted for ectopic pregnancy. She is asking why this has occurred. The nurse knows that which factor is a known risk factor for ectopic pregnancy? -high number of pregnancies -multiple gestation pregnancy -use of oral contraceptives -use of IUD for contraception

use of IUD for contraception Use of an IUD with progesterone has a known increased risk for development of ectopic pregnancies. The nurse needs to complete a full history of the client to determine if she had any other risk factors for an ectopic pregnancy. Adhesions, scarring, and narrowing of the tubal lumen may block the zygote's progress to the uterus. Any condition or surgical procedure that can injure a fallopian tube increases the risk. Examples include salpingitis, infection of the fallopian tube, endometriosis, history of prior ectopic pregnancy, any type of tubal surgery, congenital malformation of the tube, and multiple elective abortions. Conditions that inhibit peristalsis of the tube can result in tubal pregnancy. Hormonal factors may play a role because tubal pregnancy occurs more frequently in women who take fertility drugs or who use progesterone intrauterine contraceptive devices (IUDs). A high number of pregnancies, multiple gestation pregnancy, and the use of oral contraceptives are not known risk factors for ectopic pregnancy.

A woman at 8 weeks' gestation is admitted for ectopic pregnancy. She is asking why this has occurred. The nurse knows that which factor is a known risk factor for ectopic pregnancy?

use of IUD for contraception Use of an IUD with progesterone has a known increased risk for development of ectopic pregnancies. The nurse needs to complete a full history of the client to determine if she had any other risk factors for an ectopic pregnancy. Adhesions, scarring, and narrowing of the tubal lumen may block the zygote's progress to the uterus. Any condition or surgical procedure that can injure a fallopian tube increases the risk. Examples include salpingitis, infection of the fallopian tube, endometriosis, history of prior ectopic pregnancy, any type of tubal surgery, congenital malformation of the tube, and multiple elective abortions. Conditions that inhibit peristalsis of the tube can result in tubal pregnancy. Hormonal factors may play a role because tubal pregnancy occurs more frequently in women who take fertility drugs or who use progesterone intrauterine contraceptive devices (IUDs). A high number of pregnancies, multiple gestation pregnancy, and the use of oral contraceptives are not known risk factors for ectopic pregnancy.

A woman at 8 weeks' gestation is admitted for ectopic pregnancy. She is asking why this has occurred. The nurse knows that which factor is a known risk factor for ectopic pregnancy?

use of an IUD for contraception Use of an IUD with progesterone has a known increased risk for development of ectopic pregnancies. The nurse needs to complete a full history of the client to determine if she had any other risk factors for an ectopic pregnancy. Adhesions, scarring, and narrowing of the tubal lumen may block the zygote's progress to the uterus. Any condition or surgical procedure that can injure a fallopian tube increases the risk. Examples include salpingitis, infection of the fallopian tube, endometriosis, history of prior ectopic pregnancy, any type of tubal surgery, congenital malformation of the tube, and multiple elective abortions. Conditions that inhibit peristalsis of the tube can result in tubal pregnancy. Hormonal factors may play a role because tubal pregnancy occurs more frequently in women who take fertility drugs or who use progesterone intrauterine contraceptive devices (IUDs). A high number of pregnancies, multiple gestation pregnancy, and the use of oral contraceptives are not known risk factors for ectopic pregnancy.

A serum magnesium level of 6.5 mEq/L would fall

within the therapeutic range of 4 to 7 mEq/L


Ensembles d'études connexes

AP Gov. First Semester Final - Question Bank

View Set